Элементы дискретной математики в задачах

advertisement
Элементы дискретной математики
в задачах
А. А. Глибичук, А. Б. Дайняк, Д. Г. Ильинский,
А. Б. Купавский, А. М. Райгородский,
А. Б. Скопенков, А. А. Чернов
Обновляемая версия части книги:
http://www.mccme.ru/circles/oim/discrbook.pdf
Все авторы: Московский физико-технический институт. Д. Ильинский: ЦЭМИ РАН. А. Райгородский: Московский Государственный
Университет. А. Скопенков: Независимый Московский Университет.
Личные страницы:
http://dm.fizteh.ru/staff, www.mccme.ru/~skopenko.
2
Элементы дискретной математики в задачах
Оглавление
Введение . . . . . . . . . . . . . . . . . . . . . . . . . . . . .
1
Элементы комбинаторики . . . . . . . . . . . . . . . .
1.1
Подсчёт и комбинаторные тождества . . . . . .
1.2
Формула включений и исключений . . . . . . .
1.3
Принцип Дирихле . . . . . . . . . . . . . . . . .
1.4
Комбинаторика булева куба . . . . . . . . . . .
1.5
Обращение Мёбиуса . . . . . . . . . . . . . . . .
1.6
Подсчёт двумя способами . . . . . . . . . . . .
1.7
Комбинаторные покрытия. А.Я. Канель . . . .
1.8
Подсказки . . . . . . . . . . . . . . . . . . . . .
1.9
Указания . . . . . . . . . . . . . . . . . . . . . .
2
Основы теории графов . . . . . . . . . . . . . . . . . .
2.1
Основные определения . . . . . . . . . . . . . .
2.2
Перечисление деревьев . . . . . . . . . . . . . .
2.3
Графы с точностью до изоморфизма . . . . . .
2.4
Плоские графы . . . . . . . . . . . . . . . . . .
2.5
Эйлеровы пути и циклы . . . . . . . . . . . . .
2.6
Гамильтоновы пути и циклы . . . . . . . . . . .
2.7
Экстремальные задачи (теорема Турана) . . .
2.8
Теорема Менгера . . . . . . . . . . . . . . . . .
2.9
Метод минимального контрпримера. А. Канель
2.10 Степенн´ые последовательности. В.А. Волков,
М.Н. Вялый и А.Б. Скопенков . . . . . . . . . .
2.11 Подсказки . . . . . . . . . . . . . . . . . . . . .
2.12 Указания . . . . . . . . . . . . . . . . . . . . . .
3
Раскраски графов и многочлены . . . . . . . . . . . .
3
6
11
11
13
17
19
21
24
26
28
30
50
50
53
56
57
62
65
67
69
70
71
78
81
95
4
Элементы дискретной математики в задачах
4
5
6
7
3.1
Раскраски графов . . . . . . . . . . . . . . . . . 95
3.2
Хроматические число и индекс . . . . . . . . . 97
3.3
Хроматический многочлен и многочлен Татта
99
3.4
Подсказки . . . . . . . . . . . . . . . . . . . . . 101
3.5
Указания . . . . . . . . . . . . . . . . . . . . . . 102
Основы теории Рамсея . . . . . . . . . . . . . . . . . . 105
4.1
Двухцветные числа Рамсея . . . . . . . . . . . 105
4.2
Многоцветные числа Рамсея . . . . . . . . . . . 106
4.3
Числа Рамсея для гиперграфов . . . . . . . . . 108
4.4
Результаты рамсеевского типа . . . . . . . . . . 109
4.5
Числа Рамсея для подграфов . . . . . . . . . . 111
4.6
Подсказки . . . . . . . . . . . . . . . . . . . . . 112
4.7
Указания . . . . . . . . . . . . . . . . . . . . . . 115
Системы множеств (гиперграфы) . . . . . . . . . . . . 125
5.1
Пересечения подмножеств . . . . . . . . . . . . 125
5.2
Системы общих представителей . . . . . . . . . 126
5.3
Системы различных представителей . . . . . . 128
5.4
Перманент . . . . . . . . . . . . . . . . . . . . . 130
5.5
Размерность Вапника-Червоненкиса . . . . . . 131
5.6
Оценка Виссера мощности пересечений . . . . 133
5.7
Структуры на конечном множестве . . . . . . . 134
5.8
Подсолнухи . . . . . . . . . . . . . . . . . . . . . 139
5.9
Подсказки . . . . . . . . . . . . . . . . . . . . . 141
5.10 Указания . . . . . . . . . . . . . . . . . . . . . . 143
Аналитические и вероятностные методы . . . . . . . . 152
6.1
Асимптотики . . . . . . . . . . . . . . . . . . . . 152
6.2
Независимость и доказательства существования 155
6.3
Случайные графы . . . . . . . . . . . . . . . . . 171
6.4
Случайные графы. А.М. Райгородский . . . . . 175
6.5
Подсказки . . . . . . . . . . . . . . . . . . . . . 178
6.6
Указания . . . . . . . . . . . . . . . . . . . . . . 181
Алгебраические методы . . . . . . . . . . . . . . . . . . 192
7.1
Линейно-алгебраический метод в комбинаторике192
7.2
Матрицы Адамара . . . . . . . . . . . . . . . . 196
7.3
Подсказки . . . . . . . . . . . . . . . . . . . . . 198
7.4
Указания . . . . . . . . . . . . . . . . . . . . . . 199
ОГЛАВЛЕНИЕ
8
Теоремы об инцидентностях в геометрии . . . . . . . .
8.1
Задачи . . . . . . . . . . . . . . . . . . . . . . .
8.2
Подсказки . . . . . . . . . . . . . . . . . . . . .
8.3
Указания . . . . . . . . . . . . . . . . . . . . . .
9
Аддитивная комбинаторика . . . . . . . . . . . . . . .
9.1
Задачи . . . . . . . . . . . . . . . . . . . . . . .
9.2
Подсказки . . . . . . . . . . . . . . . . . . . . .
9.3
Указания . . . . . . . . . . . . . . . . . . . . . .
10 Графы: задачи для исследования . . . . . . . . . . . .
10.1 Степенные последовательности. А. Скопенков
10.2 Гамильтоновость (А. Веснин, А. Скопенков) .
10.3 Изоморфизмы графов. И.Н. Шнурников . . . .
10.4 Турниры. Д. Пермяков . . . . . . . . . . . . . .
Предметный указатель . . . . . . . . . . . . . . . . . . . . .
11 Программа курса ДА 2014-15 уч. года . . . . . . . . .
Литература . . . . . . . . . . . . . . . . . . . . . . . . . . . .
5
204
204
206
206
209
209
212
213
215
215
216
218
219
220
224
233
6
Элементы дискретной математики в задачах
Введение
Зачем эта книга?
Мы приводим подборки задач по комбинаторным разделам математики. Эти задачи подобраны так, что в процессе их решения
читатель (точнее, решатель) освоит основы важных теорий — как
классических, так и современных. Ср. [S4],[Z],[J]. Книга будет полезна руководителям и участникам кружков для старшеклассников и
младшекурсников (в частности, ориентированных на олимпиады).
Некоторые приводимые красивые задачи и важные темы малоизвестны в традиции кружков по математике, но полезны как для
математического образования, так и для подготовки к олимпиадам.
Решение этих задач (т. е. изучение соответствующих теорий)
будет полезно также всем, кто хочет стать математиком, специалистом по computer science или программистом, работающим в наукоёмких отраслях информационных технологий. Именно таких специалистов мы готовим на факультете инноваций и высоких технологий Московского Физико-Технического Института. Приведенные
задачи используются при изучении курсов дискретных структур и
дискретного анализа на этом факультете. Эти курсы читают А.Б.
Дайняк и А.М. Райгородский, а остальные авторы ведут семинары по этим курсам. Некоторые материалы основаны на занятиях,
проведенных А.Б. Скопенковым в Кировской летней математической школе, Московской выездной олимпиадной школе, а также на
кружках «Математический семинар» и «Олимпиады и математика».
Комбинаторика — один из самых красивых разделов современной математики. Постановки задач этого раздела зачастую доступны школьникам. А результаты, тем не менее, носят фундаментальный характер и важны как для развития других разделов математики, так и для приложений в информатике, биологии, экономике
и др. Мы постараемся рассказать о тех мощных современных методах, благодаря которым кристаллизуется серьезная научная дисциплина — комбинаторика. Среди этих методов, помимо более или
менее стандартных, вероятностный и линейно-алгебраический методы. Они лежат в основе самых продвинутых комбинаторных результатов, полученных за последние десятилетия.
ОГЛАВЛЕНИЕ
7
Параграфы второй половины книги дают экскурс в активно развивающиеся области математики. Хотя здесь изучаются только самые простые результаты и методы, они дают некоторое представление об основных направлениях научных исследований в соответствующих областях. Этому посвящены замечания, которые не используются ни в формулировках, ни в решениях задач. Важные
факты выделены словом «теорема» или «следствие».
Используемый материал.
Формулировки большинства задач доступны старшеклассникам,
интересующимся математикой; 1 мы приводим все определения, не
так часто изучаемые на кружках. Без определения используются
только простейшие определения и результаты теории чисел [O, §8§9], [Vi, §§1-3], [Z, §§3.1-3.4 и 3.7]. Если в некотором разделе для понимания условий или для решения задач нужны дополнительные
сведения (или консультация специалиста), то в начале соответствующего раздела приводятся ссылки.
При этом многие задачи трудны: для их решения нужно предварительно прорешать другие приведенные задачи на данную тему.
Как устроена книга.
Эту книгу не обязательно читать (точнее, прорешивать) подряд. Параграфы и разделы книги практически независимы друг от
друга (кроме разделов в §3 и §4, которые желательно прорешивать
подряд). Если в задаче одного из разделов все-таки используется
материал другого раздела, то либо эту задачу можно игнорировать,
либо посмотреть конеретно указанный материал другого раздела.
Основные обозначения приведены в конце введения. Основные понятия и обозначения теории графов введены в §2.1.
При этом параграфы расположены примерно в порядке возрастания сложности материала.
К важнейшим задачам приводятся подсказки, указания и решения. Подсказки и указания расположены в конце каждого парагра1
Часть материала (например, §1.1) на некоторых кружках и летних школах изучается даже 6-классниками. Однако приводимые подсказки, указания
и решения рассчитаны на читателей с некоторой математической культурой
(необходимой для освоения бо́льшей части книги). Разбирать эти решения с
6-классниками нужно по-другому, см., например, [GIF].
8
Элементы дискретной математики в задачах
фа. Однако к ним стоит обращаться после прорешивания каждой
задачи.
Общие замечания к формулировкам задач.
Задачи обозначаются жирными цифрами. Если в условии задачи написано «найдите», то нужно дать ответ без знака суммы
и многоточия. Если же условие задачи является формулировкой
утверждения, то в задаче требуется это утверждение доказать. Как
правило, мы приводим формулировку утверждения перед его доказательством. 2 В таких случаях для доказательства утверждения могут потребоваться следующие задачи. Это всегда явно оговаривается в подсказках, а иногда и прямо в тексте. (На занятии
задача-подсказка выдается только тогда, когда школьник или студент немного подумал над самой задачей.)
Большинство задач не оригинальны, но установить первоисточник не представляется возможным. Многие задачи взяты из [IKRS],
[Z], [L] и из неопубликованных материалов кафедры дискретной математики ФИВТ МФТИ.
О литературе.
В список литературы не вошли многие хорошие стандартные
учебники по комбинаторике и теории графов (ввиду необъятности
их количества). Мы цитировали те из них, которые по тем или иным
причинам чаще используем в преподавании. Мы цитировали всю
известную нам более продвинутую учебно-научную литературу. Но
этот список тоже не претендует на полноту, поскольку мы можем
не знать о некоторых публикациях.
В списке литературы [Ga, GKP, Gr, Har, Hal, K, M, R1, R2, R3,
R4, S1, S2, VS, 8] и [AM, BKS, BKKSS, I, Ja, J, KZP, KR, O, P, Vi, R5,
R6, S3, S5] — базовые учебники и статьи по темам этой книги и по
связанным темам, [AS, B, Bo, G, L, S7, So] — более продвинутая литература. Остальное — источники замечаний, основное содержание
2
Часто происходит обратное: формулировки красивых результатов и важных проблем, ради которых была придумана теория, приводятся только после
продолжительного изучения этой теории (или не приводятся совсем). Это способствует появлению представления о математике как науке, изучающей немотивированные понятия и теории. Такое представление принижает ценность математики.
ОГЛАВЛЕНИЕ
9
которых может быть не связано с этой книгой, и опубликованные
ранние версии отдельных частей книги.
Благодарности.
Мы благодарим за полезные замечания редакторов книги А.В.
Шаповалова и И.Д. Шкредова, а также А.А. Полянского, М.Б. Скопенкова, И.Н. Шнурникова и членов редколлегии сборника «Математическое Просвещение». Мы благодарим студентов за каверзные
вопросы и указания на неточности. Мы благодарим А.Ю. Веснина
за разрешение использовать рис. 8.
А.Б. Купавский поддержан грантом РФФИ 12-01-00683 и грантом Президента РФ МД-6277.2013.1. А.М. Райгородский поддержан
грантом РФФИ 12-01-00683, грантом Президента РФ МД-6277.2013.1
и грантом ведущих научных школ НШ-2519.2012.1. А.Б. Скопенков
частично поддержан грантом фонда Саймонса. А.А. Глибичук поддержан грантами Мол-а-вед No12-01-33080 и грантом РФФИ No1401-00332 А.
10
Элементы дискретной математики в задачах
Основные обозначения
• [x] — (нижняя) целая часть числа x.
• d | n — число n делится на число d (для целых d и n).
• Rn — множество {1, 2, . . . , n}.
• N — множество {1, 2, . . . , n, . . .} целых положительных чисел.
• R, Q, Z — множества всех действительных, рациональных и
целых чисел, соответственно.
• Z2 — множество {0, 1} остатков от деления на 2 с операциями
сложения и умножения по модулю 2.
• Zm — множество {0, 1, . . . , m − 1} остатков от деления на m
с операциями сложения и умножения по модулю m.
( )
n
•
— количество k-элементных подмножеств n-элементного
k
множества (другое обозначение: Cnk ).
( )
X
•
— множество всех k-элементных подмножеств множеk
ства X.
• |X| — число элементов во множестве X.
• A \ B = {x | x ∈ A и x ∈
/ B} — разность множеств A и B (не
путайте этот знак с /).
• A⊔B — дизъюнктное объединение множеств A и B. Оно равно
A ∪ B, если A ∩ B = ∅.
• A ⊂ B — «множество A содержится в множестве B». (В некоторых других книгах это обозначают A ⊆ B, а A ⊂ B означает
«множество A содержится в множестве B и не равно B».)
• фраза ‘обозначим x = a’ сокращается до x := a.
1. ЭЛЕМЕНТЫ КОМБИНАТОРИКИ
1
1.1
11
Элементы комбинаторики
Подсчёт и комбинаторные тождества
( ) (
)
( )
( )
n
n
n
n
1.1.1. (a)
=
. (b) Найдите сумму
+. . .+
.
k
n−k
0
n
) (
) ( )
n+1
n
n
=
+
, если 0 6
1.1.2. (a) Правило Паскаля.
k+1
k+1
k
k 6 n{− 1. (Подсказка
приведена
}
{
} после
{ }задачи 1.1.4.a.)
{ }
n+1
n
n
n
= (k + 1)
+
. Здесь
— количество
(b)
k+1
k+1
k
k
разбиений n-элементного множества на k частей (т. е. непустых подмножеств); разбиения считаются неупорядоченными, т. е. разбиение
множества {1, 2, 3} на части {1, 2} и {3} и разбиение того же множества на части {3} и {1, 2} считаются одинаковыми. Ср. с задачей
1.4.7.e.
{ }
n
Замечание. Числа
называются числами Стирлинга втоk
рого рода; подробнее о них см., например, [GKP, с. 287].
(
1.1.3. (a) Во скольких подмножествах множества R11 не найдётся
двух подряд идущих чисел?
(b) То же для трёх подряд идущих чисел.
( )
n
n!
n(n − 1) . . . (n − k + 1)
1.1.4. (a)
=
=
.
k
k!(n − k)!
k!
( )
∑n
n j n−j
n
(b) Бином Ньютона. (a + b) = j=0
a b .
j
Как решать задачи этого раздела? Мы предлагаем три метода,
которые продемонстрируем на примере трех доказательств правила
Паскаля 1.1.2.a.
(Большинство задач этого раздела решаются несколькими методами из трех предложенных Но, конечно, не каждый метод применим к каждой задаче. Обычно в указаниях для краткости приводится только один способ решения.)
12
Элементы дискретной математики в задачах
Первое доказательство: комбинаторные рассуждения. Неформально говоря, идея в следующем: чтобы выбрать k + 1 футболистов, нужно либо выбрать k + 1 полевых, либо вратаря и k полевых.
Приведем строгое изложение этой идеи.
Количество (k + 1)-элементных подмножеств
множества Rn+1 ,
( )
n
• содержащих число n + 1, равно
, так как такие подмноk
жества при выкидывании числа n + 1 становятся подмножествами
в Rn ;
(
)
n
• не содержащих число n + 1, равно
, так как такие
k+1
подмножества являются также подмножествами в Rn .
Другая запись этого решения. Определим отображение
)
) ( )
(
(
Rn+1
Rn ⊔ Rn
формулой f (A) := A\{n + 1}.
f:
→
k
k+1
k+1
Остаётся доказать, что это — биекция, т.е. взаимно-однозначное соответствие (например, определив явной формулой обратное отображение).
Второе доказательство: использование явной формулы 1.1.4.a.
Имеем
(
) ( )
n
n
n!
n!
+
=
+
=
k+1
k
(n − k − 1)!(k + 1)! (n − k)!k!
(
)
(
)
n!
1
1
n!
n+1
=
+
=
=
(n − k − 1)!k! k + 1 n − k
(n − k − 1)!k! (k + 1)(n − k)
(
)
(n + 1)!
n+1
=
=
.
(n − k)!(k + 1)!
k+1
Третье
использование бинома Ньютона 1.1.4.b.
( доказательство:
)
n+1
Число
является коэффициентом при xk+1 в многочлене
k+1
(1 + x)n+1 = (1 + x)n (1 + x) = x(1 + x)n + (1 + x)n .
(
)
n+1
Поэтому число
равно сумме коэффициентов при степенях
k+1
xk и xk+1 у многочлена (1 + x)n . Отсюда следует требуемое равенство.
1. ЭЛЕМЕНТЫ КОМБИНАТОРИКИ
13
1.1.5.( Найдите
) ( )суммы:
( )
n
n
n
(a)
−
+ . . . + (−1)n
;
0
1
n
( )
( )
( )
( )
n
1 n
1 n
1
n
(b)
+
+
+ ... +
;
0
2
1
3
2
n
+
1
n
( )
( )
( )
( )
n
n
n
n
(c)
+2
+3
+ ... + n
;
1
2
3
n
( ) (
)
(
)
n
n+1
n+m
(d)
+
+ ... +
;
k
k+1
k+m
( )2
( )2
n
n
(e)
+ ... +
;
0
n
( )( ) ( )(
)
( )( )
n m
n
m
n m
(f)
+
+ ... +
;
1 ) k−
k
0 ( )
(0 ) k (
(1
)
2n
2n − 1
2n − 2
n
(g)
−
+
− . . . + (−1)n
;
(0)
( 1
)
(2
)
( n
)
2n
2n − 1
2n − 2
n
(h)
+2
+4
+ . . . + 2n
.
n
n
n
n
1.1.6. Найдите
«явную» формулу
для
∑( n )
∑( n )
(a)
;
(b)
;
2k
4k
k>0
k>0
(c)
∑( n )
k>0
3k
.
В ответе используйте только целочисленные функции целочисленного аргумента.
1.2
Формула включений и исключений
Этот раздел посвящен доказательству и использованию формулы включения-исключения. Она позволяет отвечать на вопрос
’Сколько существует объектов с данными свойствами?’ во многих
непростых случаях. Потребуются базовые навыки решения задач по
комбинаторике. В частности, уметь приводить строгие доказательства с использованием взаимно-однозначных соответствий, правил
суммы и произведения. Например, полезно прорешать предыдущий
пункт.
В этом пункте использован материал Д.А. Пермякова.
Обозначим через φ(n) функцию Эйлера, т. е. количество чисел
от 1 до n, взаимно простых с числом n.
14
Элементы дискретной математики в задачах
1.2.1. (a) Найдите количество чисел, не превосходящих 1001 и не
делящихся ни на одно из чисел 7, 11, 13.
(b) Найдите φ(1), φ(p), φ(p2 ), φ(pα ), где p — простое число, α > 2.
(c) φ(n) = n(1− p11 ) . . . (1− p1s ), где n = pα1 1 ·. . .·pαs s — каноническое
разложение числа n.
1.2.2. (a) На полу комнаты площадью 24 м2 расположены три
ковра (произвольной формы) площади 12 м2 каждый. Тогда площадь пересечения некоторых двух ковров не меньше 4 м2 .
(b) На кафтане расположено пять заплат (произвольной формы).
Площадь каждой из них больше трех пятых площади кафтана. Тогда площадь общей части некоторых двух заплат больше одной пятой площади кафтана.
(c) То же, что в (b), если площадь каждой заплаты больше половины площади кафтана.
1.2.3. Сколькими способами можно переставить числа от 1 до n,
чтобы
(a) и 1, и 2 не оказалось на своем месте?
(b) ровно одно из чисел 1, 2 и 3 оказалось на своем месте?
(c) каждое из чисел 1, 2 и 3 оказалось не на своем месте?
(d) каждое из чисел 1, 2, 3 и 4 оказалось не на своем месте?
1.2.4. На полке стоят 10 различных книг.
(a) Сколькими способами их можно переставить так, чтобы ни
одна книга не осталась на своем месте?
(b) Количество таких перестановок книг, при которых на месте
остаётся ровно 4 книги, больше 50000.
В этом разделе предлагаются задачи следующего типа: дано
конечное множество U и набор свойств (подмножеств) Ak ⊂ U ,
k = 1, . . . , n. Требуется найти количество элементов, для которых
выполнено хотя бы одно из свойств Ak (т.е. |A1 ∪ . . . ∪ An |), либо количество элементов, для которых не выполнено ни одно из свойств
Ak (т.е. |U \ (A1 ∪ . . . ∪ An )|). Для этого используется два варианта
формулы включений и исключений (см. задачу 1.2.6.b). При этом
если во всех пересечениях множеств набора число элементов зависит только от количества пересекаемых множеств, формулу можно
упростить (см. задачу 1.2.6.a).
1. ЭЛЕМЕНТЫ КОМБИНАТОРИКИ
15
1.2.5. Рассмотрим подмножества A1 , A2 , A3 , A4 конечного множества U .
(a’) A1 ∪ A2 = (A1 \A2 ) ⊔ (A1 ∩ A2 ) ⊔ (A2 \A1 ).
(b’) |A1 ∪ A2 | = |A1 | + |A2 | − |A1 ∩ A2 |.
(c’) |A1 ∪ A2 ∪ A3 | =
= |A1 | + |A2 | + |A3 | − |A1 ∩ A2 | − |A2 ∩ A3 | − |A1 ∩ A3 | + |A1 ∩ A2 ∩ A3 |.
(a) Количество элементов в U , не принадлежащих ни одному из
подмножеств A1 , A2 , A3 , равно
|U |−|A1 |−|A2 |−|A3 |+|A1 ∩A2 |+|A2 ∩A3 |+|A1 ∩A3 |−|A1 ∩A2 ∩A3 |.
(b) Для k = 1, 2, 3, 4 обозначим
∑
Mk :=
|Ai1 ∩ Ai2 ∩ . . . ∩ Aik |.
16i1 <···<ik 64
Докажите, что количество элементов в A, не принадлежащих ни
одному из Ai , равно |U | − M1 + M2 − M3 + M4 .
(с) В условиях пункта (b) количество элементов, принадлежащих ровно одному из множеств Ai , равно M1 − 2M2 + 3M3 − 4M4 .
1.2.6. Формула включений и исключений. Рассмотрим подмножества
∩ A1 , . . . , An конечного множества U . Положим по определению
|
Aj | := U .
j∈∅
∩
(a) Пусть число α|S| := |
Aj | зависит только от размера |S|
j∈S
набора S ⊂ Rn индексов, а не от самого набора. Тогда
( )
n
∑
k+1 n
|A1 ∪ . . . ∪ An | =
(−1)
αk ,
k
k=1
|U \ (A1 ∪ . . . ∪ An )| =
(b) Обозначим Mk :=
гда
∑
Rn
k
S∈(
)
|
∩
j∈S
n
∑
k=0
( )
n
(−1)
αk .
k
k
Aj |. В частности, M0 := |U |. То-
|A1 ∪ . . . ∪ An | = M1 − M2 + M3 − . . . + (−1)n+1 Mn ,
|U \ (A1 ∪ . . . ∪ An )| = M0 − M1 + M2 + . . . + (−1)n Mn .
16
Элементы дискретной математики в задачах
(c) Неравенства Бонферрони. Для любого s
M1 −M2 +M3 −. . .−M2s 6 |A1 ∪. . .∪An | 6 M1 −M2 +M3 −. . .+M2s+1 ,
M0 − M1 + M2 − . . . + M2s 6 |U \ (A1 ∪ . . . ∪ An )| 6
6 M0 − M1 + M2 − . . . − M2s+1 .
(d) Число элементов, принадлежащих ровно r из подмножеств A1 , . . . , An ,
( )
n
∑
k−r k
равно
(−1)
Mk .
r
k=r
В задаче 1.2.7 предполагается, что ответ записывается в виде
суммы (аналогично формуле включений и исключений).
1.2.7. (a) Сколькими способами можно расселить 20 туристов по
5 различным домикам, чтобы ни один домик не оказался пустым?
(b) Сколько существует различных сюръекций f : Rk → Rn ?
1.2.8. По кругу стоят числа 1, 2, . . . , n. Найдите число способов выбрать k из них, чтобы никакие два выбранных не стояли рядом.
(b) Найдите число способов рассадить n пар враждующих рыцарей за круглый стол с нумерованными местами, чтобы никакие
два враждующих рыцаря не сидели рядом.
1.2.9. Куб с ребром длины 20 разбит на 8000 единичных кубиков,
и в каждом кубике записано число. Известно, что в каждом столбике из 20 кубиков, параллельном ребру куба, сумма чисел равна
1 (рассматриваются столбики всех трех направлений). В некотором кубике записано число 10. Через этот кубик проходит три слоя
1 × 20 × 20, параллельных граням куба. Найдите сумму всех чисел
вне этих слоев.
1.2.10.* Сколько существует шестизначных трамвайных билетов, в
которых нет двух семерок рядом и всего
(a) не более трех семерок? (b) не более четырех семерок? (c)
сколько угодно семерок?
1.2.11.* Докажите следующую формулу:
n! · x1 x2 . . . xn = (x1 + x2 + . . . + xn )n −
1. ЭЛЕМЕНТЫ КОМБИНАТОРИКИ
−
+
∑
∑
(xi1 + xi2 + . . . + xin−1 )n +
16i1 <i2 <...<in−1 6n
n
n−1
(xi1 + xi2 + . . . + xin−2 ) − . . . + (−1)
16i1 <i2 <...<in−2 6n
1.3
17
n
∑
xni .
i=1
Принцип Дирихле
1.3.1. (a) Если сумма n действительных чисел равна S, то найдется слагаемое, не большее S/n, а также слагаемое, не меньшее
S/n.
(b) Если сумма n целых чисел больше kn для некоторого целого
k, то найдется слагаемое, не меньшее k + 1.
(c) Если сумма n целых чисел меньше kn для некоторого целого
k, то найдется слагаемое, не большее k − 1.
Утверждение 1.3.1.a применяется при решении задач, см., например, задачу 1.3.9. Его ‘дискретный аналог’ утверждение 1.3.1.b
называют принципом Дирихле и часто формулируют так: при любом распределении nk+1 или более предметов по n ящикам в какомнибудь ящике окажется не менее k + 1 предмета. 3
1.3.2. В мешке лежат 32 красных шара, 29 зелeных шаров, 45 синих, 17 желтых и по 30 белых, чeрных и серых. Какое наименьшее
число шаров надо взять, чтобы среди них наверняка нашлись шары
(а) всех 9 цветов? (б) 7 цветов?
1.3.3. (a) Среди 7-значных чисел, заканчивающихся на 3 пятерки, существует не менее 1200 чисел, имеющих один и тот же остаток
от деления на 7.
(b) Для каждого 4-значного числа посчитали сумму цифр его квадрата. Докажите, что существует не менее 1200 чисел, для которых
посчитанные суммы будут давать одинаковый остаток при делении
на 7.
3
Методически более грамотно [MS, Словарик, раздел ‘оценка’] было бы назвать этот раздел ‘оценки от противного’. Однако мы выбрали название, по
которому большинство читателей смогут наиболее ясно представить себе содержание этого раздела.
18
Элементы дискретной математики в задачах
1.3.4. (a) Среди чисел, записываемых только единицами, есть число, которое делится на 1997.
(b) В строку записаны n целых чисел. Докажите, что из них можно выделить одно или несколько подряд идущих с суммой, кратной
n.
1.3.5. (a) Среди любых n действительных чисел найдутся два,
1
.
дробные части которых различаются не более чем на n−1
(b) В таблице 10×10 расставлены целые числа, причeм любые два
числа в соседних по стороне клетках отличаются не более чем на 5.
Докажите, что среди этих чисел найдутся два равных.
1.3.6. Теорема Дирихле. Дано произвольное иррациональное число α.
(a) Для произвольного натурального N найдутся такие взаимно
простые p, q ∈ Z, что 0 < q 6 N и
p
α − 6 1 .
q qN
(b) Существует бесконечно много пар взаимно простых чисел p, q ∈
Z, для которых
p
α − 6 1 .
q q2
Замечание. В формулировке утверждения 1.3.6.b можно избавиться от взаимной простоты, так как для каждой дроби pq , для
p
которой выполнено неравенство α − q 6 q12 , существует лишь ко
pk 1
нечное количество целых чисел k > 0 таких, что α − qk 6 (qk)
2.
1.3.7. Натуральные числа от 1 до 101 записаны в некотором порядке. Докажите, что в этой последовательности найдeтся либо возрастающая, либо убывающая подпоследовательность длины 11.
Замечание. В данном случае подпоследовательность — это то,
что получается из последовательности вычеркиванием некоторых
её членов.
1.3.8. Имеется 10 яблок, каждое из которых весит не более 100 г, и
две одинаковые тарелки. Докажите, что можно положить в тарелки
1. ЭЛЕМЕНТЫ КОМБИНАТОРИКИ
19
(a) несколько яблок так, чтобы веса в тарелках отличались меньше, чем на 1 г.
(b) по одинаковому количеству яблок так, чтобы веса в тарелках
отличались меньше, чем на 2 г
При этом на тарелках должно лежать хотя бы одно яблоко, но не
обязательно должны лежать все яблоки (и в пункте (a) не обязательно, чтобы на каждой тарелке лежало хотя бы одно яблоко).
1.3.9. Для любых n векторов v1 , . . . , vn длины 1 на плоскости существует такой набор ε1 , . . . , εn = ±1, что
∑
∑
√
√
(a) | nk=1 εk vk | 6 n, (b) | nk=1 εk vk | > n.
1.4
Комбинаторика булева куба
1.4.1. Расставьте на шахматной доске нескольких коней, чтобы
каждый бил четырёх других.
1.4.2. 33 буквы русского алфавита кодируются последовательностями из нулей и единиц.
(a) При каком наименьшей длине последовательности кодирование можно сделать однозначным?
(b) Если при получении сообщения возможна ошибка в не более
чем одном разряде, т. е. если коды различных букв должны отличаться по крайней мере в трёх разрядах, то 8 разрядов не хватит.
(c) Если возможна ошибка в не более чем двух разрядах, то 10
разрядов не хватит.
(d)* Найдите наименьшее число разрядов, достаточное для кодирования из (b).
( )
n
1.4.3. (a) При фиксированном n число
максимально при k =
k
[n/2].
(b) Best in their own ways. В математической олимпиаде участвовало k школьников. Выяснилось, что для любых двух школьников
A и B нашлась задача, которую решил A и не решил B, и задача,
которую решил B, но не решил A. Какое наименьшее возможное
количество задач могло быть при этом условии? Иными словами,
20
Элементы дискретной математики в задачах
найдите наименьшее возможное n, для которого найдётся такое семейство из k подмножеств n-элементного множества, что ни одно
из подмножеств семейства не содержится (собственно) в другом.
1.4.4. Имеется табло с n горящими лампочками. Каждый переключатель может быть подсоединён к некоторым лампочкам. При
нажатии на кнопку переключателя соединённые с ним лампочки
меняют свое состояние: горящие тухнут, а не горящие загораются.
Какое наименьшее число переключателей необходимо, чтобы можно было зажечь любой набор лампочек (не входящие в этот набор
лампочки гореть не должны)?
1.4.5. В первый день своего правления король организует партии
среди n своих подданных. На второй день советник приносит королю список фамилий некоторых подданных (в первый день этот
список неизвестен). На третий день король может выбрать несколько партий и отправить в тюрьму всех подданных, участвующих в
каждой из них. Какое наименьшее число партий необходимо организовать в первый день, чтобы в третий день заведомо можно
было отправить в тюрьму всех подданных из принесенного списка
(и только их)?
Замечание. Следующая важная конструкция полезна (хотя и
не обязательна) для решения вышеприведенных (и многих других)
задач. Нарисуем точки, соответствующие всем подмножествам множества Rn . При этом на k-й этаж поместим точки, соответствующие k-элементным множествам. Соединим стрелкой те из них, которые получаются друг из друга добавлением одного элемента. Тогда
соединяемые стрелкой точки лежат на соседних этажах. Полученный граф называется n-мерным кубом. Его вершины соответствуют
векторам из Zn2 .
Определение множества Zn2 приведено в начале §7.1. Подмножество L ⊂ Zn2 называется линейным подпространством, если x + y ∈
L для любых x, y ∈ L (не обязательно различных). Иными словами, линейное подпространство — такое семейство подмножеств
n-элементного множества, которое вместе с любыми двумя подмножествами содержит их симметрическую разность (т. е., сумму по
модулю 2).
1. ЭЛЕМЕНТЫ КОМБИНАТОРИКИ
21
1.4.6. (a) Любое линейное подпространство содержит нулевой набор (0, . . . , 0).
(b) Число элементов в любом линейном подпространстве является степенью двойки.
n количество линейных подпространств в
Обозначим через k Zn2 , состоящих из 2k элементов (такие линейные подпространства в
Zn2 называют k-мерными, ср. §7.1).
2 1.4.7. (a) Найдите для k = 0, 1, 2.
k
3 (b) Найдите для k = 0, 1, 2, 3.
k
n n n n n
=
= 1, (c) 1 = n − 1 = 2 − 1.
0 n n n =
.
(d) k n−k n n+1 n n−k
=
+2
(e) k .
k+1 k+1 n .
(f) Найдите 2 n .
(g) Найдите k Для решения этой задачи нужны некоторые понятия, приведенные в начале §7.1.
1.5
Обращение Мёбиуса
Под значком
∑
подразумевается сумма по всем натуральным
d|n
делителям числа n.
Определим функцию Мёбиуса µ(n) следующим образом:
22
Элементы дискретной математики в задачах
µ(n) =


1,



k


(−1) ,



0,




если n = 1;
если n является произведением
k различных простых делителей;
если n делится на p2
для некоторого простого числа p.
{
∑
1, n = 1;
1.5.1. (a)
µ(d) =
0, n ̸= 1.
d|n
(b) Найдите сумму значений функции Мёбиуса по тем и только
тем делителям числа n, в каноническое разложение которых входит
чётное количество простых множителей.
(c) (Формула обращения
∑ Мёбиуса.) Пусть f : N → R — произвольная функция, g(n) =
f (d). Тогда выполнена формула
d|n
f (n) =
∑
µ(d)g
d|n
(n)
d
∑ (n)
µ
=
g(d).
d
d|n
Функция Эйлера φ(n) определена в §1.2.
∑
1.5.2. (a) Найдите сумму
φ(d).
d|n
(b)
∑ µ(d)
d|n
d
=
(
1
1−
p1
)
(
1
... 1 −
ps
)
.
Замечание. Заметим, что, применяя формулу обращения Мёбиуса к функции φ(n), можно немного другим способом доказать
формулу для нахождения φ(n) (см. утверждение 1.2.1 c).
Действительно, используя утверждения 1.5.2.a и b, получаем:
(
)
(
)
∑ µ(d)
∑
n
1
1
=n 1−
... 1 −
.
φ(n) =
µ(d) = n ·
d
d
p1
ps
d|n
d|n
1. ЭЛЕМЕНТЫ КОМБИНАТОРИКИ
23
Обозначим через Tr (n) количество способов раскрасить карусель из n вагончиков в r цветов, т. е. число раскрасок вершин правильного n-угольника в r цветов, если раскраски, совмещающиеся
поворотом, неотличимы. При этом
• в раскраске могут быть использованы не все цвета.
• цвета различны: например, раскраски КККЖ и ЖЖЖК различны.
Приведем более формальное определение. Для любой раскраски карусели можно «разорвать» карусель между любыми двумя
вагончиками и записать получившуюся последовательность цветов
(раскраску поезда), начиная с места разрыва по часовой стрелке.
Например, следующие последовательности соответствуют одной и
той же раскраске карусели:
КЖЗС;
ЖЗСK;
ЗСКЖ;
СКЖЗ.
С другой стороны, из каждой последовательности цветов можно
получить раскраску карусели, «склеив» её начало и конец правильным образом.
Циклическим сдвигом последовательности (a1 , a2 , . . . , an ) называется последовательность (a2 , a3 , . . . , a1 ). Формально, раскраской
карусели (или, более учено, циклической последовательностью) называется класс эквивалентности последовательностей с точностью
до циклического сдвига. Итак, Tr (n) — количество циклических последовательностей длины n, элементы которых — числа 1, . . . , r.
1.5.3. (a) Найдите Tr (n) при n = 3, 4, 5, 6, 9.
n
n
(b) 22 −n−1 < T2 (2n ) < 22 −n .
Назовем периодом последовательности минимальное положительное число d, такое, что в результате d циклических сдвигов она
перейдет в себя. Аналогично определяется период карусели.
1.5.4. (a) Период последовательности делит её длину.
(b) Если d делит n, то количество последовательностей длины n
и периода d равно количеству последовательностей длины d и периода d.
1.5.5. Обозначим через Mr (n) количество последовательностей длины n и периода n, элементы которых — числа 1, . . . , r.
24
Элементы дискретной математики в задачах
(a) Найдите
∑
Mr (d).
d|n
(b) Выразите Tr (n) через все Mr (d), где d | n.
∑1∑
l
d.
(c) Tr (n) =
µ(d)r
l
(d) Tr (n)
l|n
= n1
d|l
∑
n
φ(d)r d . (Более простой способ доказательства
d|n
этой формулы приведен в §??.)
1.5.6. Найдите количество различных раскрасок карусели из n вагончиков в r цветов, в которых
(a) цвет s встречается ns раз для каждого s = 1, . . . , r. (Здесь в
качестве ответа принимается формула с суммированием по делителям, аналогичная 1.5.5.c.)
(b) присутствует ровно 4 цвета из r = 5 данных.
1.6
Подсчёт двумя способами
Мы приводим простейший вариант вероятностного метода в комбинаторике. Ср. §6.2, §6.3. Этот метод также применяется при решении задач 2.4.3, 2.6.5, 2.6.7, 2.7.2, 3.1.11.b, 4.1.5, 4.3.4 и некоторых
задач из §??.
Комбинаторные решения нижеприведенных задач можно изложить на вероятностном языке. Решения без явного построения вероятностного пространства могут привести к бессмыслице и ошибке. (Подумайте, например, с какой вероятностью случайный треугольник будет остроугольным.) Поэтому строгие решения на вероятностном языке должны начинаться с явного построения вероятностного пространства.
1.6.1. (a) Даны 21 девятиэлементных подмножеств 30-элементного
множества. Тогда какой-то элемент 30-элементного множества содержится по крайней мере в семи данных подмножествах.
(b) Комиссия собиралась 40 раз. На каждом заседании было ровно 10 человек, любые два не были вместе больше 1 раза. Тогда
в комиссии хотя бы 60 человек.
(c) В компании у любых двух знакомых друг с другом человек
есть ровно 5 общих знакомых (кроме них самих). Тогда количество
пар знакомых между собой людей в компании делится на 3.
1. ЭЛЕМЕНТЫ КОМБИНАТОРИКИ
25
(d) Обозначим через Pn (k) число перестановок множества натуральных чисел от 1 до n, оставляющих ровно k чисел на своем
n
∑
месте. Тогда
k · Pn (k) = n!.
k=0
1.6.2. Пусть F — любое семейство k-элементных подмножеств nэлементного множества.
(a) Если k > l и каждое l-элементное подмножество n-элементного
множества
содержится
в некотором подмножестве из F, то
( ) /(
)
n
k
|F| >
.
l
l
(b) Количество (k − 1)-элементных подмножеств n-элементного
множества, целиком содержащихся хотя бы в одном из подмножеств
k|F|
семейства F, не меньше
.
n−k+1
1.6.3. На планете Марс 100 государств объединены в блоки, в каждом из которых не больше 50 государств. Известно, что любые два
государства состоят вместе хотя бы в одном блоке. Найдите минимально возможное число блоков. (Ср. с задачей 1.6.2.a.)
1.6.4. Ровно 19 вершин правильного 97-угольника покрашено в белый цвет, остальные вершины покрашены в чёрный. Тогда число
равнобедренных одноцветных треугольников с вершинами в вершинах 97-угольника не зависит от способа раскраски. (Треугольник
одноцветный, если все его вершины или белые, или чёрные.)
1.6.5. Даны числа n > k и множество S из n точек на плоскости.
Если любые три точки из множества S не лежат на одной прямой
и для любой точки P ∈ S существуют хотя бы k различных
точек
√
1
из множества S, равноудаленных от P , то k < 2 + 2n.
1.6.6. В любом множестве из n различных натуральных чисел найдётся подмножество из более чем n/3 чисел, в котором нет трёх
чисел, сумма двух из которых равна третьему.
1.6.7. По каждому из 100 видов работ в фирме имеется ровно 8
специалистов. Каждому сотруднику нужно дать выходной в субботу или в воскресенье. Докажите, что это можно сделать так, чтобы
26
Элементы дискретной математики в задачах
и в субботу, и в воскресенье для каждого вида работ на работе был
специалист по нему.
Замечание. Понятно, что при данном числе k специалистов (в
задаче 1.6.7 k = 8) для малого числа видов работ так распределить
выходные всегда можно. А при большом числе l видов работ это
может уже не получиться. В следующей задаче мы находим асимптотическую оценку снизу для такого числа l.
Вот более ученая формулировка (обобщения) задачи 1.6.7. Имеется l = 2k−1 подмножеств некоторого множества, в каждом из
которых ровно k элементов. Тогда элементы этого множества можно раскрасить в два цвета так, чтобы никакое из l подмножеств не
было одноцветно. Ср. с задачей 6.2.1.a.
1.6.8. (a) Если для некоторого чётного n
(
(n/2) )l
1 − 2 (nk )
2n < 1,
k
то в n-элементном множестве найдется l таких k-элементных подмножеств, что при любой раскраске элементов этого множества в
два цвета хотя бы одно из этих l подмножеств одноцветно.
(b) Существует такое c > 0, что для любого k существует не более
чем ck 2 2k таких k-элементных подмножеств некоторого множества,
что при любой раскраске элементов этого множества в два цвета
одно из этих подмножеств одноцветно.
1.7
Комбинаторные покрытия. А.Я. Канель
Данная подборка посвящена прежде всего использованию покрытий как метода решения. В большинстве случаев рассматриваются связи между объектами и эти связи покрываются.
IMO-2001,3. 21 девочка и 21 мальчик участвовали в олимпиаде. Оказалось, что
каждый участник решил не более 6 задач;
для любых мальчика и девочки найдется задача, которую они
оба решили.
Докажите, что некоторую задачу решило не менее трех мальчиков и не менее трех девочек.
1. ЭЛЕМЕНТЫ КОМБИНАТОРИКИ
27
IMO-2005, 6. На математической олимпиаде участникам было
предложено 6 задач. Оказалось, что каждая пара задач была решена более чем 2/5 от общего числа участников, но никто не решил все
6 задач. Докажите, что найдутся по крайней мере два участника,
каждый из которых решил ровно 5 задач.
IMO-2002, 6. Пусть n > 3 и C1 , . . . , Cn — круги единичного
радиуса такие, что любая прямая пересекает не более трех из них.
∑
1
(n − 1)π
Докажите, что
6
.
4
16i<j6n |Oi Oj |
2003, 1. Пусть A есть 100-элементное подмножество множества S = {1, 2, . . . , 106 }. Докажите, что для некоторых натуральных
{t1 , . . . , t100 } ⊆ S множества Aj = {x + tj | x ∈ A}, j = 1, . . . , 100,
попарно не пересекаются.
2006, 2. Каждой стороне b выпуклого многоугольника P поставлена в соответствие наибольшая из площадей треугольников,
содержащихся в P , одна из сторон которых совпадает с b. Докажите, что сумма площадей, соответствующих всем сторонам многоугольника P , не меньше удвоенной площади многоугольника P .
Указания и решения
1.7.IMO-2001.3. Каждая задача рассматривается как ”связь”
между мальчиками и девочками. Свяжем с ней пару (a, b), где a
— число решивших мальчиков, а b — девочек. Если заключение
нашей задачи не выполнено, то для каждой задачи либо a 6 2,
либо b 6 2. Рассматриваемая задача покрывает
a × b пар
∑
∑ мальчик—
2
девочка (всего пар 21 ). Кроме того,
ai 6 6 · 21 и
bi 6 6 · 21.
Решение задачи завершается подсчетом.
Похожим образом решается ??.IMO-2005.6.
1.7.IMO-2002.6. Рассмотрите фазовое пространство прямых.
С использованием идеи фазового пространства рекомендуем познакомиться по решению задачи 4.6 (об альпинистах), опубликованном
в N6 Математического Просвещения, стр. 150-151, или по книге В.
И. Арнольд, ”Обыкновенные дифференциальные уравнения”, 1.1.2,
где разбирается задача Н. Н. Константинова о возах.
1.7.2003.1. Последовательность {tj } строится поэтапно, возможность построения осуществляется исходя из соображения покры-
28
Элементы дискретной математики в задачах
тия.
1.7.2006.2. Используйте шевеление покрытия: при движении
одной из вершин с постоянной скоростью все площади меняются
линейно. Подробнее эта идея изложена в статье А. Канель, А. Ковальджи ”Треугольники и катастрофы” (Квант, 1992 г., №11)
1.8
Подсказки
1.1.1)
. (a) Постройте
(
(
) биекцию (т.е. взаимно-однозначное соответствие)
Rn
Rn
→
между семействами k-элементных и (n − k)k
n−k
элементных подмножеств множества Rn .
{
1, n = 0;
1.1.5 . (a) Ответ:
0, n ̸= 0.
( )
(
)
n
1
n+1
1
=
.
(b) Докажите, что
k(+ 1) k ( n +)
1 k+1
n
n−1
(c) Докажите, что k
=n
.
k
k−1
(d) Используйте правило Паскаля (задача 1.1.2.a).
(f) Решается аналогично предыдущей задаче.
(g) Ответ:


 1, если n = 3k,
−1, если n = 3k + 1,


0, если n = 3k − 1.
1.1.6 . Будут полезны тригонометрическая форма комплексного числа
√
b
a+bi = r(cos φ+i sin φ), где r = a2 + b2 и φ−arctg ∈ {0, π}
a
и формула Муавра (cos φ + i sin φ)n = cos nφ + i sin nφ.
1.3.2 . (a) Ответ: 57.
1. ЭЛЕМЕНТЫ КОМБИНАТОРИКИ
29
1.4.1 . Сначала расставьте нескольких коней, чтобы каждый бил одного другого. Затем расставьте нескольких коней, чтобы каждый
бил двух других.
1.4.3 . (b) Ответ: наименьшее n = n(k), для которого
(
)
n
> k.
[n/2]
1.4.4 . Ответ: n.
1.4.7 . (a) Ответ: 1, 3, 1.
(b) Ответ: 1, 7, 7, 1.
(c) Ответ: (2n − 1)(2n−1 − 1)/3.
(d) Используйте ортогональное дополнение.
(f) Выбрать в n-мерном линейном пространстве над Z2 упорядоченную
пару
линейно
независимых
векторов
можно
n
n
(2 − 1)(2 − 2) способами.
(g) Выбрать в n-мерном линейном пространство над Z2 упорядоченный набор из k линейно независимых векторов можно (2n − 20 ) ·
(2n − 21 ) · . . . · (2n − 2k−1 ) способами.
1.5.2 . (a) Ответ:
∑
φ(d) = n.
d|n
1.5.3 . (a) Для раскраски α карусели рассмотрите все возможные
значения периода d(α) (он определен после задачи 1.5.3).
1.5.6 . (b) Выразите ответ через Tk (10), k = 1, 2, 3, 4.
1.6.3 . Сначала докажите, что каждая страна участвует не менее,
чем в трех блоках.
1.6.6 . Используйте то, что среди чисел k + 1, k + 2, . . . , 2k, 2k + 1 ни
одно не равно сумме двух других.
50
2
2.1
Элементы дискретной математики в задачах
Основы теории графов
Основные определения
Графом G = (V, E) называется конечное множество V = V (G),
некоторые двухэлементные подмножества (т. е. неупорядоченные
пары несовпадающих элементов) которого выделены. Множество
выделенных
( ) подмножеств обозначается E = E(G). Таким образом,
V
E⊂
.
2
Элементы данного множества V называются вершинами. Выделенные пары вершин называются рёбрами. Хотя эти пары неупорядоченные, в теории графов их традиционно обозначают круглыми
скобками. Вершина, принадлежащая ребру, называется его вершиной. Если вершины a и b соединены ребром, они называются соседними или смежными, а само ребро (a, b) называется проходящим
через вершину a и вершину b или инцидентным вершине a и вершине b.
Общепринятый термин для понятия графа, данного здесь —
граф без петель и кратных рёбер или простой граф.
Если не оговорено противное, то через n и e обозначаются количества вершин и рёбер рассматриваемого графа, соответственно.
Граф можно представлять себе как набор точек (например, на
плоскости), некоторые пары которых соединены ломаными. См.
рис. 2, 3, 4, 7, 8, 10 и 11 ниже. При этом только концы каждой
ломаной являются вершинами графа и каждая пара вершин не соединена более чем одной ломаной. Точки называются вершинами
графа, а ломаные — рёбрами. Ломаные могут пересекаться, но точки пересечения «не считаются», т.е. не являются вершинами.
Путем Pn называется граф с вершинами 1, 2, . . . , n и ребрами
(i, i + 1), i = 1, 2, . . . , n − 1. Циклом Cn называется граф с вершинами 1, 2, . . . , n и ребрами (1, n) и (i, i + 1), i = 1, 2, . . . , n − 1. (Не
путайте эти графы с путем в графе и циклом в графе, определенными ниже.) Граф с n вершинами, любые две из которых соединены
ребром, называется полным и обозначается Kn . Если вершины графа можно разделить на две части так, что нет рёбер, соединяющих
вершины из одной и той же части, то граф называется двудольным,
2. ОСНОВЫ ТЕОРИИ ГРАФОВ
51
а части называются долями. Через Km,n обозначается двудольный
граф с долями из m и из n вершин, в котором имеются все mn рёбер
между вершинами разных долей.
2.1.1. В любом графе есть двудольный подграф, содержащий не
менее половины рёбер графа.
Степенью deg v вершины v графа называется число выходящих
из нее рёбер. Изолированной вершиной называется вершина, из которой не выходит ни одного ребра.
Грубо говоря, подграф данного графа — это его часть. Формально, граф G называется подграфом графа H, если каждая вершина
графа G является вершиной графа H, и каждое ребро графа G
является ребром графа H. При этом две вершины графа G, соединённые ребром в графе H, не обязательно соединены ребром в
графе G.
k-кликой в графе называется его подграф с k вершинами, являющийся полным. Независимым множеством или антикликой в
графе называется набор его вершин, между которыми нет рёбер.
Путем в графе называется последовательность v1 e1 v2 e2 . . . en−1 vn ,
в которой для любого i ребро ei соединяет вершины vi и vi+1 . Число
n − 1 называется длиной пути. (Ребра e1 , e2 , . . . , en−1 не обязательно
попарно различны.)
Циклом в графе называется последовательность v1 e1 v2 e2 . . . en−1 vn en ,
в которой для любого i < n ребро ei соединяет вершины vi и vi+1 , а
ребро en соединяет вершины vn и v1 . Циклы считаются одинаковыми, если они отличаются циклическим сдвигом последовательности.
Число n называется длиной цикла. Несамопересекающимся называется цикл, для которого вершины v1 , v2 , . . . , vn попарно различны и
рёбра e1 , e2 , . . . , en попарно различны. Стандартный термин (менее
удобный для начинающего) — простой цикл.
2.1.2. (a) Любой цикл, не проходящий ни по одному ребру дважды, содержит несамопересекающийся цикл.
(b) Любой цикл нечётной длины содержит несамопересекающийся
цикл обход нечётной длины.
(c) Справедливо ли аналогичное утверждение для циклов чётной
длины, не проходящих ни по одному ребру дважды?
52
Элементы дискретной математики в задачах
(d) В графе есть несамопересекающийся цикл, проходящий через
рёбра a и b, а также есть несамопересекающийся цикл, проходящий
через рёбра b и c. Тогда есть несамопересекающийся цикл, проходящий через рёбра a и c.
Граф называется связным, если любые две его вершины можно
соединить путём, и несвязным иначе.
2.1.3. Если степень каждой из n вершин графа больше
то граф связен.
n
2
− 1,
Ясно, что «соединённость некоторым путём» является отношением эквивалентности на множестве вершин графа. Связной компонентой графа называется любой класс этой эквивалентности.
Рис. 1: Удаление ребра G − e, стягивание ребра G/e и удаление
вершины G − x
Определение операций удаления ребра и удаления вершины ясно из рис. 1. Операция стягивания ребра (рис. 1) удаляет из графа
это ребро и заменяет вершины A и B этого ребра на одну вершину
D, а все рёбра, выходящие из вершин A и B в некоторые вершины,
заменяет на рёбра, выходящие из вершины D в те же вершины. (Эта
операция отличается от стягивания ребра в мультиграфах, см. §2.5,
тем, что каждое получившееся ребра кратности больше 1 заменяется на ребро кратности 1.) Например, если граф — цикл с четырьмя
2. ОСНОВЫ ТЕОРИИ ГРАФОВ
53
вершинами, то при стягивании любого его ребра получится цикл с
тремя вершинами.
Ориентированным графом (без петель и кратных рёбер) G =
(V, E) называется конечное множество V = V (G), некоторые упорядоченные пары несовпадающих элементов которого выделены. Множество выделенных пар обозначается E = E(G). Таким образом,
E ⊂ {(x, y) ∈ V × V | x ̸= y}. Если выделены и пара (a, b), и пара
(b, a), то это ребро не называется кратным.
Ориентированный путь в ориентированном графе — такая последовательность вершин, что в каждую следующую вершину ведет
ориентированное ребро из предыдущей.
2.1.4. Пусть дан ориентированный граф G, у которого на каждом
ребре u написан вес f (u). (Этот вес можно понимать как работу,
которую нужно затратить для того, чтобы пройти по ребру от начала до конца.) Функция p : V (G) → R («потенциал») такая, что
f (x, y) = p(x) − p(y) для любого ребра u = (x, y), существует тогда
и только тогда, когда сумма весов рёбер любого ориентированного
цикла равна нулю (при прохождении ребра по циклу в направлении, противоположном ориентации, вес в сумму берётся с отрицательным знаком).
Турниром называется ориентированный граф, любые две вершины которого соединены ребром. (Т.е. для любых двух вершин
v, w турнира среди его ребер есть (v, w) или (w, v), но не оба ребра
сразу.)
Некоторые другие определения приведены в начале каждого
раздела.
2.2
Перечисление деревьев
Граф называется деревом, если он связен и не содержит несамопересекающихся циклов. Остовом графа называется любой его
подграф, являющийся деревом и содержащий все вершины графа.
2.2.1. (a) В любом дереве найдется лист, т.е. вершина степени 1.
(b) В любом дереве с n вершинами n − 1 ребро.
54
Элементы дискретной математики в задачах
(c) В любом дереве между любыми двумя вершинами существует единственный путь.
(b’) Граф с n вершинами является деревом тогда и только тогда,
когда он не содержит несамопересекающихся циклов и имеет n − 1
ребро.
(b”) Граф с n вершинами является деревом тогда и только тогда,
когда он связен и имеет n − 1 ребро.
(c’) Если в графе между любыми двумя вершинами существует
единственный путь, то граф является деревом.
(d) Последовательность из n натуральных чисел является последовательностью степеней вершин некоторого дерева тогда и только
тогда, когда сумма её членов равна 2n − 2.
Заметим, что графы ({1, 2, 3}, {{1, 2}}) и ({1, 2, 3}, {{1, 3}}) различны. Графом называется именно граф, а не класс изоморфизма
графов (определение изоморфизма приведено в начале §2.3). Или,
говоря неформально, вершины графов считаются занумерованными. Поэтому вместо слова ‘граф’ иногда употребляют термин ‘помеченный граф’.
2.2.2. Каких графов с данными n вершинами больше:
(a) имеющих изолированную вершину или не имеющих?
(b) связных или несвязных?
2.2.3. (a) Формула Кэли. Число деревьев с данными n вершинами
равно nn−2 .
(b) Если сумма целых положительных чисел d1 , . . . , dn равна 2n −
2, то число деревьев с данными n вершинами, у которых i-я верши(n − 2)!
на имеет степень di , равно
.
(d1 − 1)! · . . . · (dn − 1)!
∑ deg (1)−1
deg (n)−1
Иными словами, (x1 + . . . + xn )n−2 = T x1 T
· . . . · xn T
,
где сумма берётся по всем деревьям T с вершинами 1, 2, . . . , n, и
через degT (k) обозначена степень вершины k дерева T .
(c) * Пусть T1 , . . . , Tr — деревья, множества вершин которых не пересекаются. Сколько есть деревьев, множество вершин которых есть
объединение множества вершин этих r деревьев, и которые содержат T1 , . . . , Tr ?
2. ОСНОВЫ ТЕОРИИ ГРАФОВ
55
2.2.4. Код Прюфера сопоставляет дереву с вершинами 1, 2, . . . , n последовательность чисел от 1 до n по следующему алгоритму. Сначала код Прюфера — пустое слово. Пока количество вершин больше
двух,
1. Выбирается лист (см. задачу 2.2.1) v с минимальным номером.
2. В код Прюфера добавляется номер вершины, смежной с v.
3. Вершина v и инцидентное ей ребро удаляются из дерева.
Когда осталось две вершины, алгоритм завершает работу.
(a) Найдите код Прюфера дерева с вершинами 1, 2, . . . , 10 и рёбрами (8,9),(8,4),(4,10),(10,3),(3,5),(10,6),(10,1),(1,7),(1,2).
(b) Восстановите дерево по коду Прюфера 1,1,2,5,4,2,7.
(c) Код Прюфера определяет взаимно-однозначное соответствие
между множеством деревьев с данными n вершинами и множеством
слов длины n − 2 из этих вершин.
(d) В коде Прюфера вершина степени d встречается d − 1 раз.
2.2.5. Граф называется унициклическим, если он становится деревом после удаления некоторого ребра. (Или, эквивалентно, если он
связен и имеет ровно один — с точностью до циклического сдвига
и симметрии — несамопересекающийся цикл.)
(a) Каких графов больше, деревьев с данными 100 вершинами или
унициклических графов с данными 98 вершинами?
(b) Выразите число унициклических графов с данными n вершинами в виде суммы не более чем n слагаемых.
2.2.6. (a) В дереве нет непустых подграфов, у которых степень
каждой вершины чётная и положительная.
(b) Для графа G обозначим через h1 (G) число его подграфов без
изолированных вершин, у которых степень каждой вершины чётна. (Пустой подграф удовлетворяет этому условию.) Докажите, что
h1 (G) – степень двойки. Выразите h1 (G) через количества n вершин, e рёбер и k компонент связности графа.
Замечание. Такие подграфы называют циклами в смысле теории
гомологий (не путайте с циклами в смысле теории графов). Как
они возникают, написано, например, в [S3, §6].
(c) На рёбрах дерева стоят знаки + и −. Разрешается менять знаки на всех рёбрах, выходящих из одной вершины. Тогда из любой
расстановки можно получить любую другую.
56
Элементы дискретной математики в задачах
(d) Для графа G обозначим через h1 (G) наибольшее количество
расстановок знаков + и − на его рёбрах, ни одну из которых нельзя
получить из другого описанными выше операциями. Докажите, что
h1 (G) — степень двойки. Выразите h1 (G) через n, e и k.
Замечание. В теории когомологий такие узоры называют коциклами, а приведенное отношение эквивалентности на коциклах — когомологичностью. Как возникает когомологичность коциклов, написано, например, в [S7, §11.1, §11.2].
(e)* Докажите, что h1 (G) и h1 (G) не меняются при стягивании ребра, и выведите отсюда, что h1 (G) = h1 (G).
2.3
Графы с точностью до изоморфизма
Грубо говоря, графы изоморфны, если они одинаковы (при этом
их изображения на плоскости могут быть разными). Формально,
графы G1 и G2 называются изоморфными, если существует взаимно
однозначное отображение f : V (G1 ) → V (G2 ), удовлетворяющее
условию: вершины A, B ∈ V (G1 ) соединены ребром в том и только
в том случае, если вершины f (A), f (B) ∈ V (G2 ) соединены ребром.
l◦'
lll ''
l
l
ll
''
lll
l
◦>> ''
◦.. ◦
>> '
.. >>'
. ◦
◦
l◦'
lululuu ''
l
l
ll u
lll uuu > ''
l
◦.. ◦ uu◦ > ''
>> '
..
uu
>>'
. uuu
◦u
◦
=
◦
vv◦ ===
v
v
◦===
◦
vv
v
v
= vv
◦I
◦=
IIII vvvv ===
◦===
◦
vIvII
v
II v
= vv
◦
◦
◦
◦
Рис. 2: Какие из графов на рисунке изоморфны?
2.3.1. Какие из графов на рисунке 2 изоморфны?
2.3.2. Для произвольных k, l, m, n ∈ N найдите количество
(a) клик размера k в графе Kn ,
2. ОСНОВЫ ТЕОРИИ ГРАФОВ
57
(b) клик размера k в графе Km,n ,
(c) независимых множеств размера k в графе Kn ,
(d) независимых множеств размера k в графе Km,n ,
(e) подграфов в Kn , изоморфных Kk,l ,
(f) подграфов в Km,n , изоморфных Kk,l .
Будьте внимательны: эти задачи простые, но почти все требуют
разбора случаев.
2.3.3. Перечислите все попарно неизоморфные
(a) графы с четырьмя вершинами,
(b) связные графы с пятью вершинами и пятью рёбрами,
(c) несвязные графы с пятью вершинами.
2.3.4. Сколько существует попарно неизоморфных графов, имеющих 8 вершин и 25 рёбер?
2.3.5. Количество классов изоморфизма деревьев с n вершинами
(т. е. количество различных деревьев с n незанумерованными вершинами) меньше 4n .
2.4
Плоские графы
Плоским графом называется изображение графа на плоскости,
для которого любые два ребра пересекаются только по их общим
вершинам (в частности, если таких вершин нет, то не пересекаются).
Иногда такое изображение называют просто графом, но это неточно, поскольку один и тот же планарный граф можно изобразить
(без самопересечений) на плоскости разными способами.
Рис. 3: Различные изображения графа на плоскости
Плоский граф делит плоскость на части, называемые гранями
графа. Заметим, что одна из таких частей будет ‘бесконечной’.
58
Элементы дискретной математики в задачах
2.4.1. Дан плоский граф с треугольными гранями, имеющий более трех вершин. Удалили вершину вместе с выходящими из нее
рёбрами.
(a) Верно ли, что получившаяся грань ограничена несамопересекающимся циклом?
(b) Верно ли, что если выкинуть еще одну вершину, то все грани
опять будут ограничены несамопересекающимися циклами?
(c) Пусть в полученном графе степень каждой вершины не менее
3. Верно ли, что любую вершину нового графа можно удалить и
получить граф, все грани которого будут ограничены несамопересекающимися циклами?
2.4.2. (a) Формула Эйлера. Для любого связного плоского графа
с f гранями имеет место равенство n − e + f = 2.
(Доказательство см., например, в [P]. Далее этим результатом можно пользоваться без доказательства. Эта формула часто записывается в виде V − E + F = 2.)
(b) Найдите аналог формулы Эйлера для плоского графа с k компонентами связности.
K5
K3,3
Рис. 4: Непланарные графы
2.4.3. Применения формулы Эйлера.
(a) Ни один из графов K5 и K3,3 невозможно без самопересечений
нарисовать на плоскости.
(b) На плоскости отмечено n точек. Разрешается соединять некоторые две из них ломаной, не проходящей через другие точки. Два
игрока по очереди соединяют ломаной какие-то две еще не соединённые точки. При этом требуется, чтобы эти ломаные не самопересекались и не пересекались нигде, кроме отмеченных точек. Проиг-
2. ОСНОВЫ ТЕОРИИ ГРАФОВ
59
рывает тот, кто не может сделать ход. Для каких n при правильной
игре выигрывает тот, кто ходит первым?
(c) Перечислите все связные плоские графы (с точностью до изоморфизма), у которых степени всех вершин равны и «степени» всех
граней равны (т. е. граница каждой грани состоит из одного и того
же числа рёбер).
Замечание. Если пункты (a), (b) или (c) не получаются, решайте
следующие пункты. Определение изоморфизма см. в §2.3.
(d)* Выпуклых правильных многогранников (все грани — правильные многоугольники с одинаковым числом сторон, степени всех вершин равны) ровно 5 (с точностью до изоморфизма их графов). Конструкцию соответствующих многогранников нужно привести, она
не предполагается известной.
(e) Для любого плоского связного графа без петель и кратных
рёбер, имеющего более двух вершин, 2e > 3f и e 6 3n − 6.
(f) В любом плоском графе есть вершина степени не более 5.
(g) Если каждая вершина плоского связного графа имеет степень
d, а граница каждой грани состоит из ровно k > 3 рёбер, то
1 1
1 1
+ = + .
d k
2 e
2.4.4. Картой на плоскости называется разбиение плоскости на
конечное число многоугольников (возможно, ‘бесконечных’). Раскраска карты называется правильной, если разные многоугольники,
имеющие общий граничный отрезок, имеют разные цвета. Докажите, что любую карту можно правильно раскрасить в
(a) 6 цветов; (b) * 5 цветов; (c)** [Всё равно не докажете.]
Замечание. Используя конструкцию двойственного графа, можно доказать, что правильная раскрашиваемость любой карты на
плоскости в d цветов равносильна правильной раскрашиваемости
любого плоского графа в d цветов (§3.1).
Граф называется планарным, если его можно без самопересечений нарисовать на плоскости. Ясно, что любой подграф планарного
графа планарен.
Операция подразделения ребра графа показана на рисунке.
60
Элементы дискретной математики в задачах
Рис. 5: Подразделение ребра
Два графа называются гомеоморфными, если от одного можно
перейти к другому при помощи операций подразделения ребра и
обратных к ним; или, эквивалентно, если существует граф, полученный из каждого из данных графов операциями подразделения
ребра.
Ясно, что гомеоморфные графы являются или не являются планарными одновременно.
Теорема Куратовского. Граф является планарным тогда и
только тогда, когда он не содержит подграфа, гомеоморфного графу K5 или K3,3 (рис. 4). (Доказательство этой теоремы см., например, в [S1].)
2.4.5. Придумайте алгоритм
(a) распознавания планарности графа (здесь можно использовать
без доказательства теорему Куратовского);
(b)* рисования без самопересечений заведомо планарного графа на
плоскости.
Найдите асимптотику сложности вашего алгоритма в зависимости от числа n рёбер графа. т. е. асимптотику максимума по графам
с n рёбрами от числа шагов в алгоритме, примененному к данному
графу. См. «определение» нахождения асимптотики в §6.1.
Теорема Хопкрофта-Тарджана. Существует линейный по
количеству рёбер алгоритм распознавания планарности графа.
2.4.6. (a) Теорема Фари. Плоский граф можно нарисовать без самопересечений на плоскости так, что все рёбра будут отрезками.
(b) Дан невыпуклый многоугольник с непрозрачными сторонами.
Назовем вершину A видимой из точки X, если внутренность отрезка AX не пересекается с границей многоугольника. Назовем ядром
многоугольника множество его внутренних точек, из которых видны все вершины многоугольника. Докажите, что если точку из ядра
2. ОСНОВЫ ТЕОРИИ ГРАФОВ
61
соединить отрезками с произвольно выбранными несколькими (не
менее чем с двумя, и не обязательно со всеми) вершинами исходного
многоугольника, то многоугольник разобьется на многоугольники
с непустыми ядрами.
Тор и лента Мёбиуса изображены на рис. 6. Эти фигуры предполагаются прозрачными, т. е. точка (или подмножество), «лежащая
на одной стороне поверхности», «лежит и на другой стороне». Это
аналогично тому, что при изучении геометрии мы говорим, например, о треугольнике на плоскости, а не о треугольнике на верхней
(или нижней) стороне плоскости.
Рис. 6: Тор, лист Мёбиуса и цилиндр
2.4.7. Нарисуйте без самопересечений на торе граф
(5) K5 ; (33) K3,3 ; (6) K6 ; (34) K3,4 ; (7) K7 ;
(44) K4,4 .
2.4.8. Нарисуйте без самопересечений на ленте Мёбиуса граф
(5) K5 ; (33) K3,3 ; (6) K6 ; (34) K3,4 .
2.4.9. Картой на торе называется разбиение тора на конечное число (криволинейных и изогнутых) многоугольников. Раскраска карты на торе называется правильной, если разные многоугольники,
имеющие общую граничную кривую, имеют разные цвета. Любую
ли карту на торе можно правильно раскрасить в
(5) 5 цветов; (6) 6 цветов; (7) 7 цветов?
Подробнее см. [S3, §2].
Замечание. Любой связный граф с g рёбрами можно так нарисовать «на сфере с g ручками» (т.е. внутри правильного 2g-угольника,
диаметрально противоположные стороны которого «склеены»), что
62
Элементы дискретной математики в задачах
некоторые рёбра являются отрезками, а остальные рёбра являются объединениями двух непересекающихся отрезков, у каждого из
которых один конец — вершина графа, а другой конец лежит на
стороне 2g-угольника.
2.5
Эйлеровы пути и циклы
Мультиграфом (или графом с петлями и кратными рёбрами)
называется квадратная таблица из целых неотрицательных чисел,
симметричная относительно главной диагонали. (Мы не используем более правильную, но более громоздкую, терминологию: мультиграф — граф с кратными рёбрами, псевдограф — графом с петлями, псевдомультиграф — граф с петлями и кратными рёбрами.)
При этом число, стоящее на пересечении i-й строки и j-го столбца, интерпретируют как число рёбер (или кратность ребра) между
вершинами с номерами i и j при i ̸= j и как число петель в вершине с номером i при i = j. Ребро называется кратным, если его
кратность больше единицы.
Степенью вершины мультиграфа называется число выходящих
из нее рёбер. При этом ребро кратности k, соединяющее вершину с
другой вершиной, ‘вносит вклад’ k в степень, а петля кратности k
‘вносит вклад’ 2k в степень.
Ориентированным мультиграфом (или ориентированным графом с петлями и кратными рёбрами) называется квадратная таблица из целых неотрицательных чисел. Если в некоторой клетке
(неважно, диагональной или нет) стоит число, большее 1, то говорят, что ориентированный мультиграф имеет кратные рёбра.
Читатель легко сообразит, как определить (ориентированный)
путь в (ориентированном) мультиграфе, а также как изображать с
самоперечечениями на плоскости (ориентированные) мультиграфы.
2.5.1. Сколько всего мультиграфов с данными n вершинами
(a) ориентированных без кратных рёбер, но, возможно, с петлями?
(b) неориентированных без петель, но, возможно, с кратными рёбрами?
2.5.2. Сколько всего мультиграфов с данными n вершинами, имеющих k рёбер и
2. ОСНОВЫ ТЕОРИИ ГРАФОВ
63
(a) неориентированных без петель и кратных рёбер?
(b) неориентированных, у которых допускаются кратные рёбра и
петли?
Эйлеров цикл (путь) в мультиграфе — цикл (путь), проходящий
по каждому ребру мультиграфа ровно один раз.
2.5.3. (a) В связном мультиграфе есть эйлеров цикл тогда и только тогда, когда степень каждой его вершины чётна.
(b) В связном мультиграфе есть эйлеров цикл тогда и только тогда, когда множество его ребер распадается на несамопересекающиеся циклы.
(c) При каком условии в мультиграфе существует эйлеров путь?
(d) При каком условии в ориентированном мультиграфе существует ориентированный эйлеров цикл?
(e) При каких n граф Kn имеет эйлеров цикл?
(f) То же для графа Km,n .
Входящей степенью вершины ориентированного мультиграфа
называется число входящих в нее рёбер (с учетом кратности). Аналогично определяется исходящая степень. При петля кратности k
‘вносит вклад’ k и во входящую, и в исходящую степень.
2.5.4. (a) Если количество вершин нечётной степени в связном
графе равно 2k, то множество его рёбер можно представить в виде
объединения k путей, никакой из которых не проходит ни по какому
ребру дважды и никакие два из которых не имеют общих рёбер.
(b) На рёбрах графа, у которого степень каждой вершины чётна,
можно поставить стрелки так, что у каждой вершины входящая
степень будет совпадать с исходящей.
(c) Все рёбра связного графа раскрашены в два цвета. Из каждой
вершины выходит поровну рёбер обоих цветов. Тогда из любой вершины до любой другой можно добраться, каждый раз меняя цвет
ребра.
(d) В нарисованном на плоскости без самопересечений связном
графе есть эйлеров цикл тогда и только тогда, когда грани можно раскрасить в 2 цвета правильно, т.е. так, что при переходе через
каждое ребро цвет меняется.
64
Элементы дискретной математики в задачах
2.5.5. Математик забыл трёхзначный код своего замкá. Замок открывается, если три цифры кода набраны подряд (даже если перед
этим были набраны другие цифры). Математик набирает одну цифру в секунду; набранная цифра добавляется в конец. Докажите, что
математик сможет открыть замок за
(a) 29 секунд, если в коде могут быть использованы только цифры
1, 3 и 7;
(b) 1002 секунды, если в коде могут быть использованы только
десять цифр.
(c) Сформулируйте и докажите правило «0 < 1 < 2 . . . < 8 < 9»
открытия замка за 1002 секунды.
Последовательность де Брёйна (П. д. Б.) с параметрами n и k
— последовательность, элементы которой принадлежат заданному
множеству из k элементов (обычно — {0, 1, . . . , k − 1}), причём все
её подпоследовательности длины n различны, и среди этих подпоследовательностей встречаются все k n возможных последовательностей. (Таким образом, длина П. д. Б. равна k n + n − 1.)
(Также П. д. Б. называют бесконечную периодическую последовательность с периодом k n , каждая подпоследовательность которой
длины k n + n − 1 является П. д. Б. с параметрами n и k.)
2.5.6. Постройте последовательность де Брёйна с параметрами k =
2 (‘двоичную’) и
(a) n = 3, начинающуюся с 111;
(b) n = 4, начинающуюся с 1011;
(c) n = 4, заканчивающуюся на 1010.
2.5.7. Правило «0 лучше 1». Рассмотрим последовательность из 0
и 1, построенную по следующим правилам. Она начинается с k единиц. Дальше мы пишем 1, только если при написании 0 не все подпоследовательности длины k новой последовательности различны.
Если даже при написании 1 не все подпоследовательности длины
k новой последовательности различны, то заканчиваем написание
последовательности. Докажите, что таким образом получится последовательность де Брёйна.
2.5.8. Дан связный ориентированный мультиграф с n вершинами.
Входящая степень dk каждой вершины k равна исходящей.
2. ОСНОВЫ ТЕОРИИ ГРАФОВ
65
(a) Существует дерево, содержащее все вершины этого мультиграфа, все рёбра которого направлены в сторону вершины 1.
(b) Фиксируем дерево T из (a). Будем обходить этот граф (по
стрелкам), проходя по каждому ребру не более одного раза. Сначала выйдем из вершины 1 в произвольном направлении. Далее, пусть
мы пришли в некоторую вершину v. Выходим из нее по любому ребру, не принадлежащему T , если это возможно. А если невозможно,
то выходим из нее по ребру, принадлежащему T (такое ребро единственно). Докажите, что движение закончится в вершине 1, и что
в результате получится ориентированный эйлеров цикл.
(c) Число ориентированных эйлеровых циклов в этом мультиграфе кратно числу (d1 − 1)! · . . . · (dn − 1)!.
2.6
Гамильтоновы пути и циклы
Гамильтонов путь (цикл) в графе — путь (цикл), проходящий
через каждую вершину ровно по одному разу.
2.6.1. Грани гамильтонова плоского графа можно правильно раскрасить в 4 цвета.
Напомним (§2.1), что длина пути — число его ребер (а не вершин).
2.6.2. (a) Если граф связен и 2e > n2 − 3n + 6, то в нём есть
гамильтонов цикл.
(b) Теорема Дирака. Граф, сумма степеней любых двух несмежных вершин которого не меньше n, имеет гамильтонов цикл.
(c) Лемма Дирака. Если a0 . . . as — максимальный из путей в графе, проходящих по каждой своей вершине только один раз, s > 3
и deg a0 + deg as > s, то в этом графе есть несамопересекающийся
цикл длины s.
(d) Если в связном графе есть несамопересекающийся цикл длины
s < n, то в этом графе есть путь длины s, проходящий по каждой
своей вершине только один раз.
(e) Граф, сумма степеней любых двух несмежных вершин которого не меньше n − 1, имеет гамильтонов путь.
66
Элементы дискретной математики в задачах
2.6.3. Пусть для некоторых графа и целого k > 2 среди любых
k + 1 вершин графа есть ребро и после удаления любого набора
из k − 1 вершины граф остается связным. Тогда в этом графе есть
гамильтонов цикл.
2.6.4. Пусть среди любых k + 1 вершин графа есть ребро и после
удаления любого набора из k − 1 вершины граф остается связным.
(a) В этом графе есть хотя бы один несамопересекающийся цикл.
(b) Обозначим через v1 , . . . , vs максимальный несамопересекающийся цикл в этом графе. Обозначим через W любую компоненту связности графа, полученного удалением вершин этого цикла
из исходного графа. Обозначим через X множество вершин цикла,
соседних с W .
Тогда |X| > k.
(c) Вершины vi , vi+1 не лежат одновременно в X.
(d) Если vi , vj ∈ X, то в графе нет ребра vi+1 vj+1 .
1
6
8
7
13
9
2
14
16
5
12
15
11
3
10
4
Рис. 7: Есть ли в этом графе гамильтонов путь?
2.6.5. (a) Есть ли гамильтонов путь в графе на рисунке 7?
(b) Есть ли гамильтонов цикл в графе на рисунке 8?
(c) Для каких n есть гамильтонов цикл в графе, вершинами которого являются 3-элементные подмножества n-элементного множества, и два подмножества соединены ребром, если они пересекаются
ровно по одному элементу?
2. ОСНОВЫ ТЕОРИИ ГРАФОВ
67
Рис. 8: Граф многогранника Гринбергса. Есть ли в нём гамильтонов
путь?
2.6.6. Максимальное число попарно непересекающихся
по рёбрам
[
]
n−1
гамильтоновых циклов в графе Kn равно
.
2
2.6.7. (a) В любом турнире имеется ориентированный гамильтонов путь.
(b) Для любого n существует турнир с n вершинами, в котором
имеется не менее n!/2n ориентированных гамильтоновых путей.
2.6.8. Рёберным графом графа G называется граф, вершины которого — рёбра графа G; две вершины рёберного графа соединены
ребром, если соответствующие рёбра графа G имеют общую вершину. Найдите в терминах графа G необходимое и достаточное условие
наличия гамильтонова цикла в его рёберном графе.
См. также [Ve].
2.7
Экстремальные задачи (теорема Турана)
2.7.1. Пункты этой задачи, кроме (b), являются различными версиями и частными случаями теоремы Турана.
Треугольником в графе называется цикл длины 3.
(a) Если граф не содержит треугольников, то e 6 n2 /4.
(b) Если e = [n2 /4] + 1, то в графе есть по крайней мере [n/2]
треугольников.
68
Элементы дискретной математики в задачах
(c) Если n = km и граф не содержит (k + 1)-клики, то 2e 6 k(k −
1)m2 . (Переходя к дополнительному графу, получаем, что если n =
km и граф не содержит (k + 1)-антиклики, то 2e > km(m − 1).)
(d) Если граф не содержит (k +1)-антиклики, то 2e > km(m−1)+
2mr, где m := [n/k] и r := k{n/k}.
2.7.2. (a) Если граф не содержит несамопересекающегося цикла
длины 4, то e < n3/2 .
(b) Если граф не содержит подграфа K3,2 , то e < 2n3/2 .
(c) Если граф не содержит подграфа K3,3 , то e < 2n5/3 .
(d)* Для любых целых s, t, 2 6 s 6 t, если граф не содержит
подграфа Ks,t , то e < tn2−1/s .
2.7.3. Для любых n точек на плоскости существует не более n диаметров, т. е. (неупорядоченных) пар точек, расстояние между которыми равно максимуму из всех возможных расстояний между
парами из этих n точек.
2.7.4. Для любых n точек A1 , . . . , An в Rd обозначим через D(A1 , . . . , An )
число (неупорядоченных) пар точек, расстояние между которыми
равно 1. Обозначим
En (d) = max{D(A1 , . . . , An ) : A1 , . . . , An ∈ Rd }.
Тогда:
(a) En (2) > n[log2 n]/4; (b) En (2) 6 2n3/2 ; (c) En (3) 6 2n5/3 ;
(n − 1)2
2(n + 4)2
(d)
6 En (4) 6
.
4
5
2.7.5. (a) Пусть V — 11q -элементное подмножество пространства
Rq (определение пространства Rq см. в главе 7) любое 10q -элементное подмножество которого содержит две точки x, y на расстоянии
1: |x − y| = 1. Докажите, что для достаточно большого q количество
единичных расстояний между точками множества V больше, чем
12q /2:
12q
1
|{(x, y) ∈ V × V : |x − y| = 1}| >
.
2
2
(b) Докажите, что в условиях предыдущего пункта можно заменить число 12q /2 на 12q .
2. ОСНОВЫ ТЕОРИИ ГРАФОВ
69
2.7.6. Можно рассмотреть обобщение задачи Турана (см. задачу
2.7.1), вместо клик заданного размера запретив другие подграфы.
Обозначим через exH (n) максимальное количество рёбер в графе
с n вершинами, не содержащем подграфов, изоморфных H. Например, exKk+1 (n) — это максимальное число рёбер в графе с n
вершинами, не содержащем (k + 1)-клики.
Докажите, что если H1 — подграф графа H2 , то exH1 (n) 6
exH2 (n).
См. также задачи 6.1.2 и 6.1.3.
2.8
Теорема Менгера
2.8.1. Из каждого связного мультиграфа можно удалить вершину
(вместе со всеми выходящими из нее рёбрами) так, что он останется
связным.
Граф или мультиграф называется двусвязным, если он отличен
от K2 и остается связным после удаления любой вершины.
2.8.2. (a) Частный случай вершинной теоремы Менгера. Любые
две различные вершины двусвязного мультиграфа, не соединённые
ребром, лежат на некотором несамопересекающемся цикле.
(b) Верно ли, что для любого пути P в мультиграфе, имеющем не
менее трёх вершин, найдётся другой путь в том же мультиграфе с
теми же концами, не пересекающийся с P нигде, кроме концов?
2.8.3. Частный случай рёберной теоремы Менгера. Если в мультиграфе есть хотя бы одно ребро и при удалении любого ребра найдётся путь между вершинами a и b, то в мультиграфе найдутся два
пути между вершинами a и b, не имеющие общих рёбер.
2.8.4. (a) Для любого ребра u двусвязного графа G, отличного от
K3 , хотя бы один из графов G − u и G/u двусвязен.
(b) Для любых двух вершин выпуклого многогранника существуют три непересекающиеся (нигде, кроме этих вершин) пути по его
рёбрам из одной вершины в другую. (Такие графы называют трёхсвязными.)
70
Элементы дискретной математики в задачах
(c) Для трёхсвязного графа G с ребром xy (вершины которого
— x, y) граф G/xy трёхсвязен тогда и только тогда, когда граф
G − x − y двусвязен.
2.8.5. (a) Теорема Уитни (вершинная). Мультиграф остается связным после удаления любых k−1 вершин тогда и только тогда, когда
любые две его вершины можно соединить k путями, пересекающимися только в этих двух вершинах.
(b) Теорема Менгера (вершинная). Если вершины a и b мультиграфа G, не соединённые ребром, остаются в одной компоненте связности после удаления любых k − 1 других вершин, то a и b можно
соединить k путями, пересекающимися только в этих двух вершинах.
2.8.6. Вершины A и B графа назовем эквивалентными, если существует такая последовательность вершин A = A0 , A1 , . . . , An = B,
что любые две соседние вершины Ai и Ai+1 можно соединить k путями, не имеющими общих промежуточных вершин. Тогда любые
две эквивалентные вершины можно соединить k путями, не имеющими общих рёбер.
2.9
Метод минимального контрпримера. А. Канель
При решении многих задач используется так называемый метод минимального контрпримера (разновидность принципа крайнего или метода спуска), который заключается в следующем. Предположим, что надо доказать, что объекта, удовлетворяющего некоторым свойствам, не существует. Предположим противное — тогда
найдется (в некотором смысле) минимальный контрпример. После
чего строят еще ’меньший’ контрпример и получают противоречие.
Понятие ’меньше’ подбирается в процессе доказательства. Особенно
распространен такой метод решения в задачах на графы.
Простейшие примеры — доказательства теорем Эйлера о плоских графах [?], Куратовского [?] и Менгера [?, ?].
Более содержательный пример — знаменитая теорема Дилуорса
о частично упорядоченных множествах.
2. ОСНОВЫ ТЕОРИИ ГРАФОВ
71
1. Множество A с отношением ≺ называется частично упорядоченным, если отношение ≺ удовлетворяет следующим свойствам:
(1) a ̸≺ a,
(2) a ̸≺ b либо b ̸≺ a и
(3)
если
a ≺ b и b ≺ c,
Если a ≺ b или b ≺ a, то элементы a и b называются сравнимыми.
Если же a ̸≺ b и b ̸≺ a, то они называются несравнимыми. Цепью называется множество попарно сравнимых элементов, а антицепью —
попарно несравнимых. Диаметром частично упорядоченного множества называется максимальный размер антицепи.
(a) Количество цепей, на которые можно разбить частично упорядоченное множество, не меньше его диаметра.
(b) Теорема Дилуорса. Количество цепей, на которые разбивается частично упорядоченное множество, совпадает с его диаметром.
3. (Задача зонального этапа Всероссийской олимпиады 1994 г.)
В каждый город ведет 3 дороги: красная, синяя и белая. В зависимости от цветов входящих дорог, считая по часовой стрелке, города
разделяются на два типа КСБ и КБС. Доказать, что разность количеств городов разных типов делится на 4.
4. (IMO, 2004, Shortlist) С конечным графом разрешается производить следующую операцию: выбрать произвольный цикл длины 4 и выбросить из него произвольное ребро. Дан полный граф с
n вершинами. (Граф называется полным, если любые две его вершины соединены ребром.) Какое минимальное число ребер можно
оставить с помощью этой операции?
5. (IMO, 2001, Shortlist) k-клика есть подмножество из k человек
попарно знакомых. Известно, что каждые две 3-клики имеют общего члена и нет 5-клик. Докажите, что удалив двух людей можно все
3-клики разрушить.
6. (IMO, 1992, Longlist) Дан граф G с n вершинами, m < n. Докажите, что G содержит подмножество из m + 1 вершины степени
которых отличаются не больше чем на m — 1.
2.10
Степенны́е последовательности. В.А. Волков, М.Н.
Вялый и А.Б. Скопенков
1. (a) При каких E и n существует граф с n вершинами и E
то a
72
Элементы дискретной математики в задачах
ребрами, каждая вершина которого имеет степень 3? (Такие графы
называют кубичными или правильными степени 3.)
(b) При каких n и k существует граф на n вершинах, степени
которых равны k?
2. Графом с петлями и кратными ребрами называется набор
точек среди которых некоторые пары, возможно совпадающих, вершин соединены линиями. Ребра, соединяющие вершину саму с собой, называются петлями, а несколько ребер, соединяющих одну и
ту же пару вершин — кратными ребрами.
Даны целые положительные числа n, d1 , . . . , dn .
(a) При каких условиях существует граф с n вершинами (возможно, имеющий петли и кратные ребра), из которых выходит d1 , . . . , dn
ребер, соответственно?
(b) То же, если граф не может иметь кратных ребер (но, возможно, имеет петли, даже кратные).
(c) То же, если граф не может иметь петель (но, возможно, имеет
кратные ребра).
(d)* То же, если граф не может иметь ни петель, ни кратных
ребер.
Основной вопрос (задача 2d): какие последовательности являются степенными, то есть представляют последовательность степеней
вершин некоторого графа без петель и кратных ребер? Невозрастающую последовательность из n положительных целых чисел будем
называть правильной, если первый ее член не превосходит n − 1, а
сумма всех членов четна.
3. Докажите, что невозрастающая степенная последовательность
является правильной.
4. Являются ли степенными последовательности
(a) (43 , 16 ), (b) (64 , 23 ), (c) (53 , 33 ), (d) (1810 , 123 , 68 ), (e)
(158 , 106 , 34 )?
(Мы используем экспоненциальную запись невозрастающих целочисленных последовательностей: ak означает, что k последовательных членов последовательности равны a.)
2. ОСНОВЫ ТЕОРИИ ГРАФОВ
73
5. Докажите, что для степенной последовательности d1 , . . . , dn
(∗)
k
∑
i=1
di 6 k(k−1)+
n
∑
min(k, di ) для любого k = 1, 2, . . . , n−1.
i=k+1
6. Пусть a, b, c, d — различные вершины графа, причем (ab),
(cd) — ребра, а (ac), (bd) — не ребра. Назовем обменом преобразование графа, состоящее в удалении ребер (ab), (cd) и добавлении
ребер (ac), (bd).
Пусть G1 , G2 — два графа с одинаковыми последовательностями степеней d1 > d2 > . . . > dn . Докажите, что обменами можно
перевести граф G1 в граф G2 .
7. (a) Приведите пример правильной, но не степенной, последовательности из n чисел, лежащих в промежутке [2004, n/2004].
(b) Докажите, что любая правильная последовательность из n
√
чисел, меньших n/2, — степенная.
8. Пусть последовательность d1 , d2 , . . . , dn правильная, выполнено условие (*) и не все di равны. Обозначим t = min{i : di > di+1 }.
Определим новую последовательность
{
di ,
i ̸= t, n,
c = (c1 , c2 , . . . , cn ) формулой ci :=
di − 1, i = t, n.
Докажите, что если последовательность
(a) c степенная, то и d степенная.
правильная.
(b) d правильная, то и c
9. В обозначениях предыдущей задачи докажите условие (*) с
заменой d на c
(a) при k > t; (b) при dk 6 k−1 6 t−2; (c) при dk = k 6 t−1;
(d) при dk > k + 1 6 t.
10. Докажите, что невозрастающая последовательность является степенной тогда и только тогда, когда сумма ее членов четна и
выполнено условие (*).
11.* (abcd) То же, что в задаче 2, для связных графов.
(a’b’c’d’) Сформулируйте и решите аналог задачи 2 для ориентируемых графов.
74
Элементы дискретной математики в задачах
Ответы, указания и решения
2.9.1. (a) Ясно, что цепь и антицепь могут пересекаться не более
чем по одному элементу. Поэтому количество цепей, на которые
можно разбить частично упорядоченное множество, не меньше его
диаметра.
(b) (Доказательство теоремы Дилуорса.) Рассмотрим минимальный контрпример. Пусть C — цепь диаметра D. Тогда каждый элемент множества M − D либо больше некоторого элемента
из D, либо строго его меньше. Таким образом, M −D есть несвязное
объединение двух частей M ′ (выше D) и M ′′ (ниже D).
Если M ′ ∪ D и M ′′ ∪ D разбиваются на d цепей, то и все M разбивается на d цепей (каждая цепь склеивается из верхней и нижней
половин).
Если
M ′ ̸= ∅ и M ′′ ̸= ∅,
то #(M ′ ∪D) < #M
и #(M ′′ ∪D) < #M.
Мы осуществили спуск. В противном случае имеется не более двух
максимальных антицепей: верхняя (если M ′ = ∅) и нижняя (если
M ′′ = ∅). Рассмотрим случай наличия только верхней антицепи
(случай наличия только нижней антицепи симметричен).
Пусть имеется единственная максимальная антицепь D и x ∈
D. Тогда M − {x} имеет диаметр d − 1 и в силу минимальномти
контрпримера разбивается на d − 1 цепь C1 , . . . , Cd−1 . Поэтому x ∪
{Ci }d−1
i=1 есть искомое разбиение на d цепей.
Пусть имеются две максимальные антицепи D1 и D2 . Легко показать, что найдется пара элементов x ∈ D1 и y ∈ D2 таких, что
x ≺ y. Тогда диаметр множества M −{x, y} строго меньше диаметра
M и доказательство завершается аналогично.
2.10.1. (a) Ответ: такой граф существует при (V, E) = (2k, 3k)
для произвольного целого k > 1.
Рассмотрим произвольный кубический граф: каждая его вершина имеет степень 3. Сумма степеней всех вершин есть 2E. Поэтому
3V = 2E. Тогда пары чисел (V, E) имеют вид (2k, 3k) для некоторого натурального k. Так как нет петель и кратных ребер, то k = 1
невозможно.
2. ОСНОВЫ ТЕОРИИ ГРАФОВ
75
При k > 1 условию удовлетворяют, например, 2k-угольник с
проведенными в нем диагоналями, соединяющими i-тые и (i + k)тые вершины.
(b) Ответ: такой граф существует при k ∈ {0, 1, . . . , n − 1} и
kn четном.
Для доказательства достаточности расположите вершины графа в вершинах правильного n-угольника. Соедините ребрами вершины, расстояние между которыми по окружности не превосходит
[k/2]. Для четного k построение графа закончено. Для нечетного k
число n четно, поэтому можно и нужно добавить большие диагонали.
Обозначим e := (d1 + · · · + dn )/2.
2.10.2. Ответы: (a,b) e целое. (c) e целое и di 6 e для любого
i. (d) См. задачи 5 и 10.
(с) Теорема. Граф без петель (но, возможно, с кратными ребрами и, возможно, не связный) с n вершинами степеней d1 , d2 , ..., dn >
0 существует тогда и только тогда, когда e := (d1 + · · · + dn )/2
целое и di 6 e для любого i.
Доказательство (предложено А. Руховичем). Необходимость
целочисленности e вытекает из того, что e равно числу ребер в
графе. Второе условие необходимо, поскольку в графе нет петель, а
значит степень каждой вершины не больше суммы степеней остальных вершин.
Докажем достаточность индукцией по e. База индукции: утверждение для e = 0 очевидно. Докажем шаг индукции. Пусть утверждение для e < k. Докажем, что оно верно и для e = k > 1. Из
k > 1 и условия di 6 e следует, что найдутся хотя бы две вершины
ненулевой степени. Можно считать, что d1 > d2 > ... > dn . Рассмотрим набор d1 − 1, d2 − 1, d3 , ..., dn . Условия теоремы для него
выполнены, поскольку сумма степеней вершин уменьшилась на 2,
а степень каждой вершины понизилась не более, чем на 1. Поэтому можно воспользоваться предположением индукции: существует
граф для набора d1 − 1, d2 − 1, d3 , ..., dn . В этом графе соединим ребром вершины 1 и 2. Поскольку эти вершины различны, то петель не
появилось. Следовательно, новый граф не содержит петель. Ясно,
что набор степеней его вершин — d1 , d2 , d3 , ..., dn . QED
76
Элементы дискретной математики в задачах
2.10.4. (a,c) да.
(b,d,e) нет.
2.10.5. Оцените сверху количество ребер, выходящих из первых
k вершин.
2.10.8. (a) Если хотим добавить ребро AB, а оно уже есть, то
возьмем вершину C, не соединенную с A. А потом возьмем вершину
D, соединенную с C, но не с B.
Доказательство С.А. Чоудама теоремы о степенных последовательностях (решение задач 9, 10). Докажем, что если последовательность d1 , d2 , . . . , dn неотрицательных чисел правильная и
k
∑
i=1
di 6 k(k − 1) +
n
∑
min(k, dj ),
j=k+1
для любого k = 1, 2, . . . , n − 1, то существует граф со степенями
d1 , d2 , . . . , dn .
Доказательство
ведем индукцией по сумме элементов последо∑
вательности
di . Случай, когда все di равны, рассмотрен в задаче 1b. Пусть теперь не все di равны. Можно считать, что dn > 0.
Обозначим t = min{i : di > di+1 }. Определим новую последовательность
{
di ,
i ̸= t, n,
c = (c1 , c2 , . . . , cn ) формулой ci :=
di − 1, i = t, n.
Обозначим
Sk =
казать неравенства
(∗)
k
∑
di ,
i=1
Sk′
Sk′
=
k
∑
ci . По задаче 8 достаточно до-
i=1
6 k(k − 1) +
n
∑
i=k+1
min{k, ci }.
При k > t
Sk′
= Sk −1 6 k(k−1)+
n
∑
i=k+1
min{k, di }−1 6 k(k−1)+
Пусть теперь k 6 t − 1. Тогда Sk′ = Sk = kdk .
n
∑
i=k+1
min{k, ci }.
2. ОСНОВЫ ТЕОРИИ ГРАФОВ
77
Для dk 6 k − 1 неравенство (*) тривиально.
Для dk = k
(
Sk′ −k(k−1)
(3)
(4)
= k −k(k−1) = k = dk+1 6 dk+1 +
2
n
∑
i=k+2
)
di − 2
(5)
=
n
∑
i=k+1
min{k, ci },
• третье равенство выполнено, поскольку k 6 t − 1;
• четвертое неравенство очевидно, если k + 2 < n; если же k +
2 = n, то d = ((n − 2)(n−1) , dn ) и dn > 2 в силу четности суммы
d1 + d2 + · · · + dn .
• пятое равенство выполнено, поскольку min{k, ci } = ci при i >
k + 1.
Случай dk > k+1. Если dn > k+1, то min{k, di } = min{k, ci } = k
при i > k + 1 и неравенство (*) следует из аналогичного для Sk .
Пусть теперь dn 6 k. Имеем
min{k, ci } = min{k, di } при k+1 6 i < n и
min{k, cn } = min{k, dn }−1.
В нашем случае Sk′ = Sk , поэтому достаточно показать, что
(∗∗)
Sk 6 k(k−1)+
n
∑
i=k+1
min{k, ci } = k(k−1)+
n
∑
i=k+1
min{k, di }−1.
Учитывая, что dk+1 = dk > k + 1, получаем
Sk+1
n
k+1 ∑
k + 1 (3)
= (k + 1)dk =
Sk 6 (k + 1)(k − 1) +
min{k, di } =
k
k
i=k+1
n
n
∑
(5)
k+1 ∑
= (k + 1)(k − 1) + (k + 1) +
min{k, di } > (k + 1)k +
min{k +
k
i=k+2
i=k+2
Неравенство (5) выполнено, так как при всех k + 2 6 i < n имеем
нестрогое неравенство и при i = n строгое. Значит, в (3) неравенство
строгое. Отсюда вытекает (**).
Комментарий. Можно также доказать, что если для правильной последовательности d1 , . . . , dn выполняются неравенства из задачи 5, то последовательность, полученная из d2 −1, d3 −1, . . . , dd1 +1 −
78
Элементы дискретной математики в задачах
1, dd1 +2 , dd1 +3 , . . . , dn−1 , dn расположением чисел по возрастанию, правильная и для нее выполняются аналогичные неравенства.
2.10.11. (abcd) То же, что в соответствующих пунктах задачи
2, с добавлением условия e > n − 1.
(c) Теорема. Связный граф без петель (но, возможно, с кратными ребрами) с n вершинами степеней d1 , d2 , ..., dn > 1 сушествует тогда и только тогда, когда выполнены следующие три условия:
(1) e := (d1 + d2 + ... + dn )/2 целое;
(2) di 6 e для любого i;
(3) n − 1 6 e.
Доказательство (предложено А. Руховичем). Для доказательства необходимости обозначим через e количество ребер в графе.
Условия (1) и (2) необходимы по задаче 2.c. Необходимость условия
(3) легко проверяется.
Для доказательства достаточности рассмотрим граф, полученный по задаче 2.c. Обозначим через c количество компонент связности этого графа.
Докажем, что если c > 1, то можно уменьшить количество компонент связности, не меняя степеней вершин. Ввиду условия (3)
e > n − 1 > n − c. Поэтому хотя бы в одной компоненте связности
есть цикл. Тогда можно взять ребро a1 a2 этого цикла и ребро b1 b2
из другой компоненты связности. Удалим эти ребра, и вместо них
добавим ребра a1 b1 и a2 b2 (ср. с задачей 6). Тогда степени вершин
сохранятся, а количество компонент связности уменьшится на 1.
Такими операциями можно понизить количество компонент связности до 1. Получится связный граф без петель с заданными степенями вершин. QED
2.11
Подсказки
2.1.1 . Обозначим данный граф через G. Посчитайте двумя способами количество таких пар (A, x), что A ⊂ V (G), x ∈ E(G) и ровно
один конец ребра x лежит в A.
2. ОСНОВЫ ТЕОРИИ ГРАФОВ
79
2.2.3 . Используйте взаимно-однозначное соответствие из следующей задачи 2.2.4.
2.3.1 . Чтобы доказать изоморфность, нужно привести изоморфизм:
указать, какая вершина первого графа соответствует какой вершине второго. А чтобы доказать неизоморфность, нужно придумать какой-то инвариант, который должен быть одинаковым у изоморфных графов, а у заданной пары графов различен. Для начала
в качестве инварианта можно попробовать взять количества вершин, рёбер, обход заданной длины, вершин заданной степени, наличие какого-то подграфа и т. д.
2.3.3 . В этой задаче нужно выработать какую-нибудь стратегию
перебора. В противном случае велики шансы либо забыть какойнибудь граф, либо привести два изоморфных графа. Выберите какойнибудь такой параметр (количество рёбер, количество несамопересекающихся циклов, длина минимального несамопересекающегося
цикла, количество компонент связности и т. д.), что в зависимости
от его значения множество всех графов разбивается на «не очень
большое» количество «хорошо обозримых» групп, и в каждой из
групп не слишком много графов. Тем самым вы сведете к минимуму вероятность ошибки, т.к. графы из разных групп заведомо
неизоморфны, а внутри одной группы легче отслеживать изоморфизм и «полноту».
2.4.2 . (b) n − e + f = 1 + k.
2.4.3 . (a) Пусть граф K5 нарисован на плоскости без самопересечений. Тогда по формуле Эйлера 5 − 10 + f = 2. Значит, f = 7.
Найдите соотношение между количествами граней и ребер.
(b) При n > 3 в конце игры все грани треугольные.
(c) Используйте (f) и (g). Предостережение: не забудьте доказать
изоморфность графов с одинаковыми степенями вершин и граней.
(d) К (c) нужно добавить конструкцию соответствующих многогранников.
80
Элементы дискретной математики в задачах
(e) Скольким граням принадлежит ребро? Какое наименьшее число рёбер может ограничивать грань?
2.4.6 . (a) Используйте (b). См. подробнее [P].
2.5.4 . (b) Нужно построить эйлеров цикл и ориентировать его.
(d) Используйте задачу 3.1.1 для «двойственного» графа.
2.5.5 . (a) Определим мультиграф де Брёйна для слов длины n из
k-буквенного алфавита. (Стандартный термин — граф де Брёйна.)
Его вершины — слова длины n−1 из k-буквенного алфавита. Ориентированные рёбра соответствуют словам (a1 , a2 , . . . , an−1 , an ); ориентированное ребро, соответствующее слову (a1 , a2 , . . . , an−1 , an ), ведет от вершины (a1 , a2 , . . . , an−2 , an−1 ) к вершине (a2 , a3 , . . . , an−1 , an )
(эти вершины могут совпадать).
Докажите существование эйлерова цикла в мультиграфе Де Брейна
для слов длины 3 из 10-буквенного алфавита. Можно также действовать аналогично решению задачи 2.5.7.
(b) Аналогично решению пункта (a).
(c) Аналогично решению пункта 2.5.7.
2.5.6 . (a) Аналогично решению задачи 2.5.5.a. Можно доказать и
использовать утверждение задачи 2.5.7.
2.5.7 . Используйте задачу 2.5.8.b для мультиграфа де Брёйна.
2.6.3 . См. задачу 2.6.4.
2.6.5 . Ответы:
(a) нет;
(b) нет;
(c) для n ̸∈ {4, 5}.
2.6.7 . (a) Индукция по количеству вершин графа.
(b) Посчитайте двумя способами число турнироперестановок,
т.е., пар (G, l), где G — турнир с n вершинами и l — гамильтонов
путь в нем.
2. ОСНОВЫ ТЕОРИИ ГРАФОВ
81
2.6.8 . Вот нужное условие: в графе G существует цикл, содержащий хотя бы по одной вершине из каждого ребра и не проходящий
ни по одному ребру дважды.
2.7.1 . (a) У концов любого ребра нет общих соседей, иначе есть
треугольник.
(b) Индукция по n с переходом от n к n+2. В доказательстве шага
рассмотрите случай, когда каждое ребро содержится в некотором
треугольнике, и противоположный случай.
(c) Обобщите рассуждения из (a).
(d) Аналогично решению пункта (c).
2.7.2 . Посчитайте двумя способами количество пар (a, {b, c}), где
a, b, c — вершины графа, b ̸= c, причем ab и ac — рёбра.
В (с) используйте неравенство между средним арифметическим
и средним кубическим. В (d) используйте неравенство между средним арифметическим и средним степенным порядка s.
2.7.4 . В этой и следующей задаче удобно использовать понятие дистанционного графа. Это граф, множество вершин которого — заданное множество точек, и в котором ребром соединены вершины
на расстоянии 1.
2.8.5 . (b) Пусть G — минимальный по числу рёбер контрпример к
теореме Менгера (b) для k = 3. Докажите, что вершины a и b оказываются в разных компонентах после удаления некоторых трёх
вершин x, y, z, две из которых соединены ребром.
2.12
Указания
2.1.2 . (b) Рассмотрим любую вершину, по которой цикл проходит
хотя бы дважды. Можно рассмотреть две части цикла: между первым и вторым прохождениями этой вершины и оставшуюся. Каждая из этих частей является циклом, и одна из них имеет нечётную
длину. Уменьшая так длину нечётного цикла, получим несамопересекающийся цикл.
3. РАСКРАСКИ ГРАФОВ И МНОГОЧЛЕНЫ
3
3.1
95
Раскраски графов и многочлены
Раскраски графов
Раскраска графа (т.е. вершин графа) в несколько цветов называется правильной, если концы любого ребра разноцветны.
3.1.1. Докажите, что следующие три условия эквивалентны:
• граф двудолен;
• граф можно правильно раскрасить в 2 цвета;
• граф содержит циклы только чётной длины.
3.1.2. (a) Если в графе степень каждой вершины не превосходит
d, то его можно правильно раскрасить в d + 1 цвет.
(b) Если в связном графе степень каждой вершины не превосходит d и есть вершина степени менее d, то его можно правильно
раскрасить в d цветов.
(c) Если в связном графе степень каждой вершины не превосходит
d, и есть вершина, после удаления которой граф перестает быть
связным, то граф можно правильно раскрасить в d цветов.
(d) Если связный граф G, имеющий более двух вершин, при удалении некоторого ребра распадается на два графа, каждый из которых можно правильно раскрасить в d цветов, то и исходный граф
можно правильно раскрасить в d цветов.
3.1.3. Если для некоторого k в графе с n вершинами среди любых
k + 1 вершин есть ребро, то граф невозможно правильно покрасить
менее, чем в n/k цветов.
3.1.4. В выпуклом многоугольнике провели несколько диагоналей,
не имеющих общих внутренних точек. Полученный плоский граф
можно правильно раскрасить в 3 цвета.
3.1.5. (a) В связном графе степень каждой вершины не превосходит трёх. Известно, что его можно правильно раскрасить в 3 цвета
так, чтобы соседи некоторой вершины были одноцветны. Добавили
одну вершину и выходящие из нее рёбра так, что по-прежнему степени всех вершин не превосходят трёх. Докажите, что полученный
граф можно правильно раскрасить в 3 цвета.
96
Элементы дискретной математики в задачах
(b) В связном графе степень каждой вершины не превосходит трёх.
Известно, что его можно правильно раскрасить в 3 цвета и при любой такой раскраске у каждой вершины есть разноцветные соседи.
Добавили одну вершину и выходящие из нее рёбра так, что попрежнему степени всех вершин не превосходят трёх и полученный
граф отличен от K4 . Докажите, что полученный граф можно правильно раскрасить в 3 цвета.
(c) Теорема Брукса. Если степень каждой вершины графа не превосходит d > 3 и нет (d + 1)-клики, то граф можно правильно раскрасить в d цветов.
3.1.6. Натуральные числа d, k, d1 , . . . , dk таковы, что d1 + d2 + . . . +
dk = d + 1 − k. Степень любой вершины графа не превосходит d.
Докажите, что вершины можно разбить на k групп так, что любая
вершина i-й группы соединена не более чем с di вершинами своей
группы.
3.1.7. Трем смышлёным девочкам Ире, Тане и Юле выдали по копии одного и того же графа. Юля и Таня раскрасили свои графы
правильно. Юля использовала меньше цветов, чем Таня, зато у Тани в каждый цвет покрашено не менее двух вершин. Докажите, что
Ира может правильно раскрасить свой граф, использовав не больше цветов, чем Юля, и чтобы в каждый цвет было покрашено не
менее двух вершин.
3.1.8. (a) Если граф невозможно правильно раскрасить в k − 1
цвет, то для любой его правильной раскраски в k цветов существует
путь, в котором встречается ровно по одной вершине каждого цвета.
(b) Если максимальный из путей в графе, проходящих по каждой
своей вершине только один раз, проходит через d вершин, то граф
можно правильно раскрасить в d цветов.
(c) Если максимальный нечётный обход в графе проходит через
d − 1 вершину, то граф можно правильно раскрасить в d цветов.
3.1.9. Ориентированный граф, из каждой вершины которого выходит не более d рёбер, можно правильно раскрасить в 2d + 1 цвет.
3.1.10. Имеется несколько цветов. Каждой вершине двудольного
графа с n 6 2k−1 вершинами сопоставлено не менее, чем k цветов. («Списки» цветов, сопоставленные разным вершинам, могут
3. РАСКРАСКИ ГРАФОВ И МНОГОЧЛЕНЫ
97
быть и одинаковыми, и различными.) Тогда существует правильная раскраска графа, приписывающая каждой вершине некоторый
сопоставленный ей цвет.
3.1.11. (a) Если степень каждой вершины графа не превосходит
d, то рёбра графа можно раскрасить в d + 1 цвет так, чтобы рёбра,
имеющие общий конец, были разноцветны.
(b) Существует такая раскраска рёбер графа K
в два
(m,n)(
) цвета, что
m n 1−ab
число одноцветных подграфов Ka,b не больше
2
.
a
b
3.2
Хроматические число и индекс
Хроматическим числом χ(G) графа G называется минимальное
количество цветов, в которые можно правильно покрасить вершины
графа G.
3.2.1. Если при удалении из графа любой вершины хроматическое
число уменьшается, то χ(G) 6 1 + [2e/n].
3.2.2. (a) На какое число может измениться хроматическое число
графа, если добавить к графу одно ребро? Или, формально, найдите все целые k, для которых существует граф G и его ребро u
такие, что χ(G) − χ(G − u) = k.
(b) χ(V, E1 ∪E2 ) 6 χ(V, E1 )χ(V, E2 ). (Напомним, см. §2.1, что через
(V, E) обозначается граф со множеством вершин V и множеством
ребер E.)
(c) Для любых r1 , r2 ∈ N постройте такие графы (V, E1 ) и (V, E2 ),
что χ(V, E1 ∪ E2 ) = χ(V, E1 )χ(V, E2 ), χ(V, E1 ) = r1 и χ(V, E2 ) = r2 .
3.2.3. Следующий алгоритм раскраски вершин графа называется
жадным. Сначала все вершины произвольно нумеруются. После
этого последовательно каждую вершину, начиная с первой, красим
в цвет с минимальным номером, отсутствующим среди уже покрашенных соседей этой вершины.
(a) Вершины произвольного графа G можно занумеровать так,
чтобы жадный алгоритм его раскраски использовал ровно χ(G)
цветов.
98
Элементы дискретной математики в задачах
(b) Для каждого целого k > 0 постройте такие двудольный граф и
нумерацию его вершин, что раскраска графа, построенная жадным
алгоритмом, отвечающим построенной нумерации, имеет не менее
k цветов.
Эта задача показывает, что «качество» раскраски, построенной
жадным алгоритмом, сильно зависит от упорядочения вершин.
Раскраска рёбер графа называется правильной, если любые два
ребра, имеющие общую вершину, окрашены в разные цвета. Хроматический индекс графа — минимальное число цветов, в которые
можно правильно раскрасить рёбра этого графа.
Рис. 10: Граф Петерсена
j◦TTTTT
TT
jjjj
j
j
◦''===
◦
'' ◦I ◦
◦
'' IIII uuuu IuIu
''
I
'' ◦uu ◦== =
◦
◦
1
=N=NN
◦GF
==NNN
p◦ppp◦
pp
◦@A
ED
p◦-p
p
pp - C
◦pNNN◦N=== -NN=  -◦ BC
2
j◦TT
jjjj TTTTT
j
j
◦''=== ◦
'' ◦ ◦.
.. uuu◦ ''
u
''
uu..
u
u
'' ◦
◦=== ◦
◦
3
Рис. 11: Исследуйте на планарность, найдите хроматическое число
и хроматический индекс графов
3.2.4. Исследуйте на планарность (§2.4), найдите хроматическое
число и хроматический индекс графов:
(a) графа Петерсена, изображённого на рисунке 10;
3. РАСКРАСКИ ГРАФОВ И МНОГОЧЛЕНЫ
99
(b) графов с рисунка 11.
3.3
Хроматический многочлен и многочлен Татта
Значением хроматической функции χG графа G в точке t называется количество правильных раскрасок графа в t цветов.
3.3.1. Найдите хроматическую функцию для (a) полного графа;
(b) графа, не имеющего ребер; (c) пути; (d) цикла; (e) дерева
с n вершинами.
3.3.2. (a) χG = χG−u − χG/u для любого ребра u графа G.
(b) Теорема Биркгофа–Уитни. Для каждого графа G существует
ровно один такой многочлен, что для любого t число χG (t) правильных раскрасок графа G в t цветов равно значению в точке t этого
многочлена.
Ввиду этой теоремы хроматическая функция называется хроматическим многочленом и считается определенной не только для целых
t > 0 (ср. с (e)).
(c) Степень хроматического многочлена χG равна n, старший коэффициент равен 1, второй коэффициент равен (−e), коэффициенты знакопеременны (т. е. коэффициент при tn−2k неотрицателен и
коэффициент при tn−2k+1 неположителен для любого целого k).
(d) Третий коэффициент хроматического многочлена графа, считая с самого старшего, однозначно определяется набором подграфов графа, содержащих 3 вершины.
(e) Число |χG (−1)| равно числу ациклических ориентаций графа
G, т. е. числу способов так расставить стрелки на его рёбрах, чтобы
полученный ориентированный граф не содержал ориентированных
циклов.
3.3.3. (a) Если хроматический многочлен графа равен t(t−1)n−1 ,
то граф — дерево.
(b) Не существует графа с хроматическим многочленом t4 − 3t3 +
3t2 .
3.3.4. (a) Найдите χG⊔H , зная χG и χH .
100
Элементы дискретной математики в задачах
(b) Путь из m вершин «прицепили» за один из концов к одной из
вершин графа G, содержащего n вершин. Выразите хроматический
многочлен полученного графа с m + n − 1 вершиной через χG .
(c) Если H, K — графы, на которые распадается связный граф G
при удалении его ребра, то tχG = (t − 1)χH χK .
3.3.5. Обозначим через χ′G = χ′G (t) количество правильных раскрасок рёбер графа G в t цветов.
(a) Функция χ′G является многочленом от t.
(b) Старший моном в χ′G равен te .
( v)
∑
(c) Коэффициент при te−1 в χ′G равен − v∈V (G) deg
.
2
Мостом называется ребро, при удалении которого количество
связных компонент графа увеличивается. Граф называется лесом,
если он не содержит несамопересекающихся циклов. Напомним, что
при стягивании ребра в мультиграфах, в отличие от графов, получившиеся ребра кратности больше 1 не заменяются на ребра кратности 1.
3.3.6. Для любого мультиграфа G выполнено TG = TG−u + TG/u ,
где u — ребро мультиграфа G, не являющееся ни петлей, ни мостом,
и TG — число
(a) остовных лесов (т. е. объединений остовов его компонент);
(b) таких наборов рёбер, что для любой компоненты связности
графа лежащие в ней рёбра из набора образуют связный подграф;
(c) подграфов, являющихся лесами.
3.3.7. Даны связный мультиграф и набор ребер в нем, не содержащий несамопересекающихся циклов и ни одного ребра некоторого
максимального дерева. В мультимножестве (т. е. в неупорядоченном наборе с кратностями) мультиграфов разрешается для любого
мультиграфа G и его ребра u, не являющегося ни петлёй, ни мостом,
заменять один мультиграф G на два мультиграфа G − u, G/u. Эта
замена применяется ко всем ребрам набора (точнее, к соответствующим им рёбрам мультиграфов, которые получены из исходного
при помощи этой операции). Тогда полученное мультимножество с
точностью до изоморфизмов входящих в него мультиграфов корректно определено , т. е. не зависит от порядка рёбер, к которым
мы применяем замены.
3. РАСКРАСКИ ГРАФОВ И МНОГОЧЛЕНЫ
101
3.3.8. Многочленом Татта мультиграфа G называется многочлен
T (x, y) от двух переменных, определённый рекуррентной формулой
TG = TG−u + TG/u , если u — не петля и не мост, и TG (x, y) = xi y j ,
если G имеет i мостов, j петель и не имеет других рёбер. Используя
без доказательства корректную определенность многочлена Татта,
выразите через него
(a’) хроматический многочлен;
(a,b,c) числа из задачи 3.3.6.
3.4
Подсказки
3.1.1 . Сначала докажите, что чётность любых двух путей, соединяющих фиксированные вершины a и b, одинаковая.
3.1.2 . (a), (b), (c) Примените индукцию по числу вершин.
3.1.3 . Среди одноцветных вершин нет рёбер.
3.1.11 . (b) Посчитаем двумя способами количество таких пар (A, x),
что A — раскраска, а x — подграф, изоморфный Ka,b и одноцветный
относительно A.
3.3.2 . (b) Используйте (a).
3.3.5 . Рассмотрите граф G′ — рёберный граф графа G. Вершины
графа G′ соответствуют рёбрам графа G. Пара вершин G′ смежна,
если соответствующие рёбра графа G имеют общий конец. Остаётся использовать свойства обычного хроматического многочлена
(задача 3.3.2).
3.3.6 . Аналогично теореме Биркгофа-Уитни (задача 3.3.2.b). Или
используйте задачи 3.3.7 и 3.3.8.
3.3.8 . Ответы: (a’) χG (t) = (−1)n−k tk TG (1 − t, 0), где k — число
компонент связности графа G.
(a) TG (1, 1). (b) TG (1, 2). (c) TG (2, 1).
4. ОСНОВЫ ТЕОРИИ РАМСЕЯ
4
4.1
105
Основы теории Рамсея
Двухцветные числа Рамсея
4.1.1. (33’) Среди пяти человек может не найтись ни трёх попарно
знакомых, ни трёх попарно незнакомых.
(33) Среди любых шести человек найдётся либо трое попарно
знакомых, либо трое попарно незнакомых.
(43) Среди любых десяти человек найдётся либо четверо попарно знакомых, либо трое попарно незнакомых.
(439) Среди любых девяти человек найдётся либо четверо попарно знакомых, либо трое попарно незнакомых.
(43’) Среди восьми человек может не найтись ни трёх попарно
знакомых, ни четверых попарно незнакомых.
(44) Среди любых 18 человек найдётся либо 4 попарно знакомых, либо 4 попарно незнакомых.
(35) Среди любых 14 человек найдётся либо 5 попарно знакомых, либо 3 попарно незнакомых.
Числом Рамсея R(m, n) называется минимальное из таких целых положительных чисел x, что выполнено любое из следующих
эквивалентных условий:
• среди любых x человек найдётся либо m попарно знакомых,
либо n попарно незнакомых.
• в любом графе с x вершинами найдётся либо m-клика, либо
n-антиклика.
• для любой раскраски рёбер графа Kx в синий и красный цвета
найдётся либо синяя m-клика, либо красная n-клика.
Например, очевидно, что R(1, n) = 1 и R(2, n) = n для любого
n. В задаче 4.1.1 доказано, что R(3, 3) = 6, R(3, 4) = 9, R(4, 4) 6 18
и R(3, 5) 6 14. Но не очевидно, что такое число существует для
любых m, n.
4.1.2. (a) Если числа R(m − 1, n) и R(m, n − 1) существуют, то
число R(m, n) существует и R(m, n) 6 R(m − 1, n) + R(m, n − 1).
Это утверждение обычно коротко записывают в виде «R(m, n) 6
R(m − 1, n) + R(m, n − 1)». Далее аналогичные утверждения записываются только в кратком виде.
106
Элементы дискретной математики в задачах
(
)
m+n−2
(b) R(m, n) 6
.
m−1
(c) R(m, n) 6 R(m − 1, n) + R(m, n − 1) − 1, если числа R(m − 1, n)
и R(m, n − 1) чётны.
(d) R(5, 5) 6 62.
4.1.3. (a) Если в графе с 13 вершинами нет ни треугольника, ни
5-антиклики, то степень каждой вершины равна 4.
(b) Если в графе с 18 вершинами нет ни треугольника, ни 6-антиклики,
то степень каждой вершины равна 5.
(Во избежание порочного круга, при решении этой и других задач
не используйте без доказательства ни равенства R(3, 6) = 18, ни
других фактов, которые не умеете доказывать.)
4.1.4. (a) R(4, 4) > 18. (b) R(3, 5) > 14.
4.1.5. (a) R(n, n) > (n − 1)2 .
(b) Теорема Эрдеша. R(n, n) & n2n/2 /e.
(Знак & определен в §6.1. Если Вам неохота туда лезть, то докажи(n−3)/2 , начиная с некоторого n.)
те, что R(n,
( n))> n2
n
r
(c) Если
< 2( 2 )−1 , то R(n, n) > r.
(n)
n
r
(d) Если
< s2( 2 )−1 , то R(n, n) > r − s.
n
( )
r 1−(n)
(e) R(n, n) > r −
2 2 − 1 для любого r.
n
4.1.6. В любом турнире с 4n вершинами можно выбрать вершины
A1 , . . . , An так, чтобы каждое ребро между ними было направлено
от большего номера к меньшему.
4.1.7. При любой раскраске рёбер графа Kn в два цвета в нём
найдётся гамильтонов цикл, состоящий из двух одноцветных путей
(цвета путей могут быть и одинаковы, и различны).
4.2
Многоцветные числа Рамсея
4.2.1. (a) На плоскости отметили 17 точек и соединили каждые
2 из них цветным отрезком: красным, желтым или зеленым. Тогда
есть одноцветный треугольник.
4. ОСНОВЫ ТЕОРИИ РАМСЕЯ
107
(b) Придумайте 9 точек на плоскости и раскраску в 3 цвета всех
соединяющих их отрезков, для которой нет одноцветного треугольника.
(c) То же, что в (b), для 16 точек.
Числом Рамсея R(m1 , . . . , mk ) называется минимальное из таких целых положительных чисел x, что для любой раскраски рёбер графа Kx в k цветов для некоторого i найдётся mi -клика i-ого
цвета (т.е. mi вершин, попарно соединённых рёбрами цвета i).
Например, очевидно, что R(1, m, n) = 1 и R(2, m, n) = R(m, n)
для любых m, n. В задаче 4.2.1 доказано, что R(3, 3, 3) 6 17, > 10
и > 17. Но не очевидно, что такое число существует для любых
m1 , . . . , mk .
4.2.2. (a) R(m, n, p) 6 R(R(m, n), p).
(b) R(m1 , m2 , . . . , mk ) 6
6 R(m1 −1, m2 , . . . , mk )+R(m1 , m2 −1, . . . , mk )+. . .+R(m1 , m2 , . . . , mk −1).
(c) Найдите оценку на R(m1 , m2 , . . . , mk ) через полиномиальные
коэффициенты.
4.2.3. Если рёбра графа K31 раскрашены в синий, белый и красный
цвета так, что нет ни синей 4-клики, ни белой 3-клики, ни красной
3-клики, то из каждой вершины выходит 14, 15 или 16 синих рёбер.
4.2.4. Теорема. Для любого целого m > 0 существует такое M >
0, что для любого простого числа p > M сравнение xm + y m ≡
z m (mod p) имеет ненулевое решение. (Доказать эту теорему вы
сможете после решения двух следующих задач.)
4.2.5. Сравнение xm + y m ≡ z m (mod p) имеет ненулевое решение
для (a) m = 2, p = 89; (b) m = 3, p = 89; (c) m = 4, p = 83;
(d) m = 3, p = 97; (e) m = 9, p = 97.
4.2.6. Теорема Шура. Для любой раскраски натурального ряда в
конечное число цветов найдётся одноцветное решение уравнения
x + y = z.
Более точно, для любого целого k > 0 существует такое целое
r > 0, что для любой раскраски первых r натуральных чисел в k
цветов найдётся одноцветное решение уравнения x + y = z.
108
Элементы дискретной математики в задачах
4.2.7. (a,b) Найдите нижние оценки на R(n, . . . , n), аналогичные
| {z }
утверждениям 4.1.5.ab.
4.3
k
Числа Рамсея для гиперграфов
4.3.1. Среди любых четырёх из 8000 студентов имеется слаженная
тройка (т. е. тройка, составляющая слаженную команду на олимпиаду по программированию). Докажите, что есть 5 студентов, любая
тройка из которых является слаженной.
4.3.2. (a) Среди любых 5 точек общего положения на плоскости
найдётся выпуклый 4-угольник.
(b) Найдётся 8 точек общего положения на плоскости, среди которых нет выпуклого 5-угольника.
(c) Среди любых 9 точек общего положения на плоскости найдётся выпуклый 5-угольник.
(d) Теорема Эрдеша-Секереша. Для некоторого n среди любых
n точек общего положения на плоскости найдётся выпуклый 10угольник. (Ср. с задачей 4.4.4.)
Числом Рамсея для гиперграфов Rl (m1 , . . . , mk ), m1 , . . . , mk > l,
называется минимальное из таких целых положительных чисел x,
что для любой раскраски всех l-элементных подмножеств x-элементного
множества в k цветов найдутся i и подмножество размера mi , у которого все l-элементные подмножества покрашены в i-й цвет. («Число Рамсея для гиперграфов» — единый термин, определённый выше; знание термина «гиперграф» не нужно для его понимания.)
Например, очевидно, что R2 (m1 , . . . , mk ) = R(m1 , . . . , mk ) и R3 (3, n) =
n. В задаче 4.3.1 требуется доказать, что R3 (5, 4) 6 8000. А при решении задачи 4.3.2.d требуется доказать, что R3 (10, 10) или R4 (5, 10)
существует.
4.3.3. (a) Число Rl (m1 , . . . , mk ) существует для любых m1 , . . . , mk .
(Это не очевидно!)
(b) Rl (m1 , . . . , mk ) 6 Rl (Rl (m1 , m2 ), m3 , . . . , mk ).
(c) Rl (m, n) 6 Rl−1 (Rl (m − 1, n), Rl (m, n − 1)) + 1.
4. ОСНОВЫ ТЕОРИИ РАМСЕЯ
109
( )
n
r
4.3.4. (a) Если
< 2( 3 )−1 , то R3 (n, n) > r.
n
2
(b) Найдётся такое число c > 0, что R3 (n, n) > 2cn .
(c) Найдите нижние оценки на Rl (n, . . . , n), аналогичные утвер| {z }
ждениям 4.1.5.ab (ср. с задачей 4.2.7).
Заметим, что Rl (n, . . . , n) > (l − 1)k k
| {z }
той n).
4.4
k
...k
(степенная башня высо-
k2k +1
Результаты рамсеевского типа
4.4.1. Верно ли, что для любой раскраски точек плоскости в два
цвета найдётся
(a) одноцветный равносторонний треугольник со стороной 1 или
√
3?
(b) одноцветный равносторонний треугольник
√ со√стороной 1?
(c) одноцветный треугольник со сторонами 2, 6, π?
4.4.2. При любой раскраске точек плоскости в три цвета найдутся
две точки одного цвета на расстоянии 1.
4.4.3. (a) Найдётся такое n, что для любой раскраски пространства Rn (определение см. в главе 7) в 9 цветов найдётся прямоугольник с одноцветными вершинами и сторонами 1 и 2.
(b) Верно ли, что для любого параллелограмма P с неперпендикулярными сторонами найдётся такое n, что для любой раскраски
точек пространства Rn в 4 цвета найдётся равный P параллелограмм с одноцветными вершинами?
4.4.4. Назовем m-чашкой (m-шапкой) подмножество из m точек
графика выпуклой вниз (вверх) функции. Обозначим через f (k, l)
минимальное число n, такое, что среди любых n точек на плоскости,
никакие три из которых не лежат на прямой, есть либо k-чашка,
либо l-шапка. (Не очевидно, что такое число существует. Поэтому
о формулах из этой задачи справедливо замечание, аналогичное
сделанному в задаче 4.1.2.a.)
110
Элементы дискретной математики в задачах
(a) Среди любых f (m, m) точек на плоскости найдётся выпуклый
m-угольник. (Ср. с задачей 4.3.2.)
(b) f (k, l) 6 f((k − 1, l) )
+ f (k, l − 1) − 1.
k+l−4
(c) f (k, l) 6
+ 1.
k
−
2
(
)
k+l−4
(d) f (k, l) =
+ 1.
k−2
4.4.5. (a) При любой раскраске чисел 1, . . . , 9 в 2 цвета найдётся
одноцветная трёхчленная арифметическая прогрессия.
(b) Аналог предыдущего пункта для чисел 1, . . . , 8 неверен.
(c) Существует такое целое W , что при любой раскраске чисел
1, . . . , W в 2 цвета найдётся либо трёхчленная арифметическая прогрессия первого цвета, либо четырёхчленная — второго.
(d) Существует такое целое W , что при любой раскраске чисел
1, . . . , W в 3 цвета найдётся одноцветная трёхчленная арифметическая прогрессия.
(e) То же, что в предыдущем пункте, для r цветов.
(f)* Теорема Ван дер Вардена. Для любых k, r при любой раскраске натурального ряда в r цветов найдётся одноцветная k-членная
арифметическая прогрессия.
4.4.6. (a) Из любых 5 точек на плоскости можно выбрать две такие
непересекающиеся пары точек, что отрезок, соединяющий точки
в первой паре, пересекает отрезок, соединяющий точки во второй
паре.
(a’) Для любых 5 точек общего положения на плоскости количество пересечений отрезков, не имеющих общих концов, каждый
из которых соединяет данные точки, нечетно.
(Набор точек на плоскости называется набором общего положения, если никакие 3 из них не лежат на одной прямой.)
(b) Теорема Конвея–Гордона–Закса для линейных вложений. Для
любых 6 точек общего положения в пространстве найдутся два зацепленных треугольника с вершинами в этих точках (т.е. таких,
что объединение сторон первого пересекает второй двумерный треугольник в единственной точке.)
(Набор точек в пространстве называется набором общего положения, если никакие 4 из них не лежат в одной плоскости.)
4. ОСНОВЫ ТЕОРИИ РАМСЕЯ
111
(c) Теорема ван Кампена–Флореса для линейных вложений. Из
любых 7 точек в четырехмерном пространстве можно выбрать две
такие непересекающиеся тройки точек, что образованные этими тройками двумерные треугольники пересекаются.
Подробнее см. [G].
4.5
Числа Рамсея для подграфов
4.5.1. Для любых графов G и H существует целое положительное
число x, для которого при любой раскраске рёбер графа Kx в два
цвета найдётся либо подграф первого цвета, изоморфный G, либо
подграф второго цвета, изоморфный H.
Наименьшее из таких чисел x обозначается R(G, H).
4.5.2. R(G, H) > (χ(G)−1)(c(H)−1)+1, где χ(G) — хроматическое
число графа G, c(H) — число вершин в наибольшей компоненте
связности.
4.5.3. Обозначим через Tm дерево на m вершинах.
(a) R(Tm , Kn ) = (m − 1)(n − 1) + 1.
(b) Если m − 1 делит n − 1, то R(Tm , K1,n ) = m + n − 1.
4.5.4. Обозначим через nK3 граф из n непересекающихся (по вершинам) треугольников.
(a) R(nK3 , nK3 ) > 5n.
(b) Ребра графа раскрашены в два цвета. Если есть два непересекающихся одноцветных треугольника, то среди их вершин есть
пять, на которых есть два одноцветных треугольника, пересекающихся по одной вершине (такая конструкция называется бантик ).
(c) R(nK3 , nK3 ) 6 5n + 1.
(d)* R(2K3 , 2K3 ) = 10.
(e)* R(nK3 , nK3 ) = 5n для любого n > 1.
4.5.5. Найдите R(K3 , Cn ), где Cn — цикл с n вершинами (2.1).
112
4.6
Элементы дискретной математики в задачах
Подсказки
4.1.2 . (b) Следует из рекуррентного соотношения (a).
(c) Пусть это неверно, т. е. существует граф с R(m−1, n)+R(m, n−
1) − 1 вершинами, в котором нет ни m-клики, ни n-антиклики.
(d) Используйте рекуррентные соотношения (a,c).
4.1.3 . (a) Используйте то, что R(3, 4) 6 9 (задача 4.1.1.(349)).
(b) Используйте то, что R(3, 5) 6 14 (задача 4.1.1.(35)).
4.1.4 . (a) Рассмотрите граф с 17 вершинами, в котором вершины
i, j соединены ребром тогда и только тогда, когда i − j − x2 делится
на 17 для некоторого целого x.
(b) Рассмотрите граф с 13 вершинами, в котором вершины i, j
соединены ребром тогда и только тогда, когда i − j − x3 делится на
13 для некоторого целого x.
4.1.5 . (b) Возьмите r := [n2n/2 /e] в (e). Используйте задачу 6.1.6.c
(ср. с формулой Стирлинга 6.1.6.e).
(c) Подсчёт двумя способами, ср. §1.6, или случайная раскраска,
§6.3.
(d) Аналогично решению пункта (c). После выбора раскраски, в которой одноцветных клик не более s, удалите из каждой одноцветной
клики по вершине.
(e) Следует из (d).
4.2.1 . (a) Из каждой точки выходит не менее 6 одноцветных отрезков.
(c) Из каждой точки выходит по 5 отрезков каждого цвета.
4.2.2 . Аналогично решению задачи 4.1.2.
4.2.5 . (a) Первый способ. Найти явное решение при помощи пифагоровых троек.
4. ОСНОВЫ ТЕОРИИ РАМСЕЯ
113
Второй способ. Построим граф с p вершинами, в котором вершины i, j соединены ребром тогда и только тогда, когда i − j —
квадратичный вычет по модулю p. В таком графе есть либо 3-клика,
либо 3-антиклика.
(b,c) Используйте малую теорему Ферма.
(d) Раскрасьте рёбра графа K96 в 3 цвета так, чтобы наличие
одноцветного треугольника означало наличие необходимой тройки
чисел.
( )
n
r
4.2.7 . (b) Сначала докажите, что если
< k ( 2 )−1 , то R(n, . . . , n) >
| {z }
n
k
r.
4.3.1 . Докажем, что можно выбрать 4 таких студентов, если всего
студентов 19 (а не 8000). Выберем одного из студентов и назовем
его Никитой. Двух студентов среди оставшихся подружим, если они
вместе с Никитой составляют слаженную тройку. Так как R(4, 4) 6
18, то среди оставшихся студентов найдутся либо четверо попарно
дружных, либо четверо попарно не дружных.
В первом случае, так как среди этих четверых есть слаженная
тройка, то эти трое вместе с Никитой составляют нужную четверку.
Во втором случае к любым трем из этих четверых можно добавить Никиту. Среди полученных четырёх студентов найдётся слаженная тройка. Такой тройкой может быть только тройка студентов, к которой мы добавили Никиту. Итак, четверо попарно не
дружных студентов составляют нужную четверку.
Пять таких студентов выбираются аналогично, используя неравенство R(19, 5) < 8000, вытекающее из задачи 4.1.2.b. (Более сильное неравенство R(19, 5) 6 6783 получается из задач 4.1.2.a,c.)
4.3.2 . (a) Несложный перебор, основанный на рассмотрении выпуклой оболочки точек.
(c) Более сложный перебор, основанный на рассмотрении выпуклой оболочки точек.
(d) Занумеруем n точек на плоскости. Назовем (неупорядоченную) тройку {i, j, k} точек, где i < j < k, положительной, если
114
Элементы дискретной математики в задачах
треугольник ijk (с упорядоченными вершинами) ориентирован положительно.
Другое решение. Посадим в каждую точку по студенту. Скажем, что
четверо студентов образуют команду по бриджу, если они сидят в
вершинах выпуклого 4-угольника.
4.3.3 . (a) Следует из (b) и (c) по индукции.
(b) Объединим первый и второй цвет в один цвет.
(c) Обозначим x := Rl−1 (Rl (m − 1, n), Rl (m, n − 1)) + 1. Пусть дана раскраска l-элементных подмножеств x-элементного множества
X в красный и синий цвета. Возьмём любую точку A множества.
Определим индуцированную раскраску (l − 1)-элементных подмножеств множества X − {A} формулой c(S) := c({A} ∪ S).
4.3.4 . Аналогично решениям задач 4.1.5 и 4.2.7.
4.4.1 . (a) Используйте ромб с углом π/3.
(b) Раскрасьте плоскость «полосами».
(c) Используйте пункт (a).
4.4.2 . В противном случае вершины любого равностороннего треугольника со стороной 1 разноцветны.
4.4.5 . (a) Разберите варианты раскраски чисел 4 и 6.
(d) Приведём доказательство для двух цветов. Оно более сложно,
чем решение пункта (a), зато обобщается на 3 цвета, если вместо
прямоугольника взять параллелепипед.
Расположим числа 1, . . . , 325 в таблицу 5 × 65: ajk := j + 5(k − 1),
j ∈ {1, 2, 3, 4, 5}, k ∈ {1, 2, . . . , 65}. Cреди первых 33 (5-элементных)
столбцов есть два одинаково раскрашенных. Пусть, например, это
1-й и 23-й столбцы. В каждом из них среди первых трёх элементов есть одноцветная арифметическая прогрессия длины 2. Пусть,
например, это 1-й и 3-й элементы. Пусть нет трёхчленной арифметической прогрессии. Тогда элементы 5 в обоих столбцах (1-м и
23-м) другого цвета. Рассматривая элемент 5 в 45-м столбце, получаем противоречие.
5. СИСТЕМЫ МНОЖЕСТВ (ГИПЕРГРАФЫ)
5
5.1
125
Системы множеств (гиперграфы)
Пересечения подмножеств
Когда речь идет о множестве подмножеств, употребляют синонимы ‘система’, ‘семейство’ или ‘набор’ подмножеств.
5.1.1. В любом семействе попарно пересекающихся подмножеств
n-элементного множества не более 2n−1 подмножеств.
5.1.2. Пусть 2 6 t 6 n − 2.
(a) Постройте семейство из 2n−t подмножеств n-элементного множества, любые два из которых пересекаются не менее, чем по t
элементам.
(b) Существует ли такое семейство из 2n−t + 1 подмножеств?
5.1.3. Теорема Эрдеша—Ко—Радо. Пусть F — любое семейство kэлементных подмножеств n-элементного множества.
(a) Если( 2k 6)n и любые два подмножества из F пересекаются,
n−1
то |F| 6
.
k−1
(b) Если 2k > n и объединение никаких двух
из F
( подмножеств
)
n−1
не есть все n-элементное множество, то |F| 6
.
k
5.1.4. Любое семейство из двадцати 5-элементных подмножеств 15элементного множества можно так разбить на 6 подсемейств, чтобы
любые два непересекающихся подмножества лежали бы в разных
подсемействах.
Замечание. В таких ситуациях (см. §7.1) обычно вместо разбиении на подсемейства говорят о раскраске в разные цвета. Тогда
вопреки наглядному представлению о раскраске красятся множества, но при этом не красятся их элементы.
5.1.5. Для l < k обозначим через M (n, k, l) минимальное количество таких k-элементных подмножеств множества Rn , что любое
l-элементное подмножество множества Rn целиком содержится хотя бы в одном
(n) из
(k)них. Например, задача 1.6.2.a утверждает, что
M (n, k, l) > l / l .
126
(a)
(b)
(c)*
(d)
5.2
Элементы дискретной математики в задачах
Найдите M (n, k, 1).
Найдите M (6k + 3, 3, 2).
Найдите M (n, 3, 2).
Докажите, что M (n, k, l) > nM (n − 1, k − 1, l − 1)/k.
Системы общих представителей
5.2.1. В группе студентов Яндекса 20 человек. Из них ровно 5 человек — специалисты по поиску в интернете, 5 — по борьбе со спамом
и т.д., всего 18 видов проблем (так что, очевидно, некоторые студенты являются специалистами по разным проблемам). Требуется
составить из этих студентов сильную команду разработчиков. При
этом хочется, чтобы для каждой проблемы в команде нашелся специалист по ней и чтобы размер команды был как можно меньше
(для экономии зарплаты).
(a) При любом раскладе получится набрать такую команду из семи человек.
(b) При некотором раскладе не получится набрать такую команду
из пяти человек.
Системой общих представителей (сокращённо с.о.п.) для набора M множеств называется такое множество A, что M ∩ A ̸= ∅
для любого M ∈ M.
5.2.2. Для набора {{1, 6}, {1, 2}, {2, 3}, {3, 4}, {4, 5}, {5, 6}} множеств
найдите (a) некоторую с.о.п.; (b) с.о.п. наименьшего размера.
5.2.3. (a) Найдите наименьший размер с.о.п. для набора всех kэлементных подмножеств множества Rn .
(b) Сколько для него имеется с.о.п. наименьшего размера?
Минимальная с.о.п. — с.о.п. наименьшего размера
для данного
(( Rn ) )
k
набора M. Назовем (n, s, k)-набором элемент из
, т.е. набор
s
k-элементных подмножеств множества Rn , в котором s множеств.
(Этот термин не общепринят.)
(
)
n−1
5.2.4. (a) Постройте (2n, 2
, k)-набор, для которой миниk−1
мальная с.о.п. состоит из двух элементов и единственна.
5. СИСТЕМЫ МНОЖЕСТВ (ГИПЕРГРАФЫ)
127
(b) При данных n, k найдите наибольшее s, для которого найдётся
(n, s, k)-набор, имеющая ровно две минимальные с.о.п.
5.2.5. Жадным алгоритмом называется следующий. Возьмем любой элемент, лежащий в максимальном количестве множеств данного набора. Добавим его в ‘пред-с.о.п’ и выкинем множества, которые
его содержат. Аналогично возьмем элемент, лежащий в максимальном количестве оставшихся множеств, и т.д.
Постройте набор множеств, наименьший размер с.о.п. которого
на k меньше размера любой из с.о.п., которые могут быть получены жадным алгоритмом, если (a) k = 1; (b) k = 2; (c) k
произвольно.
5.2.6. *
(a) Для любого (n, s, k)-набора найдется с.о.п. размера меньше
{
}
n n sk
n
G(n, s, k) := max
, ln
+ + 1.
k k
n
k
sk
< ek , то найдется (n, s, k)-набор,
n
n
sk
размер любой с.о.п. которого больше
ln .
64k
n
(c) Если
(( ) ( ) (
))
n
n
n−l
k 6n−l и G
,
,
6 s,
k
l
k
(b) Если n > 32k и 60 6
то найдется (n, s, k)-набор, размер любой с.о.п. которого больше l.
(d)( )
Для
достаточно больших n если k 2 l+kl2 < n1.8 , то k < n−l
( всех )
n
n−l
и
/
< 2ekl/n .
k
k
(e) Для всех достаточно больших n и k если 101 ln ln k < ln sk
n <
√
√
k < 4 n, то найдется (n, s, k)-набор, размер любой с.о.п. которого
n sk
больше 0, 99 ln .
k
n
(f) Если
( )((n) (n−l)) ((n))
n
k −
k
l 6n−k и
< k ,
l
s
s
то найдется (n, s, k)-набор, размер любой с.о.п. которого больше l.
128
5.3
Элементы дискретной математики в задачах
Системы различных представителей
5.3.1. (a) Лемма о паросочетаниях. Пусть есть несколько (конечное число) юношей и девушек. Каждый юноша любит некоторое (вполне возможно, нулевое) число девушек. Тогда всех юношей
можно женить на любимых ими девушках (так, чтобы брачные пары не пересекались) тогда и только тогда, когда для любого множества юношей число девушек, которых любит хотя бы один из них,
не меньше числа этих юношей.
(b) Теорема Холла. Пусть S1 , . . . , Sm — конечные множества. В
каждом из них можно выбрать по элементу xi ∈ Si так, чтобы
все xi были различны, тогда и только тогда, когда для каждого
k ∈ {1, . . . , m} объединение любых k из этих множеств имеет не
менее k элементов.
5.3.2. Какое минимальное количество рёбер можно удалить из графа Kn,n , чтобы не осталось паросочетаний (т. е. подграфа из n непересекающихся отрезков)?
Пусть дан набор M множеств. В каждом из множеств выбрали по элементу. Если все элементы различны, то такой набор назовем системой различных представителей (сокращенно с.р.п.).
Или, формально, системой различных представителей для набора M множеств называется упорядоченный набор различных элементов x(S) ∈ S, S ∈ M, т. е., такое инъективное отображение
x : M → ∪S∈M S, что x(S) ∈ S для любого S ∈ M.
(Упорядоченность набора важна для подсчёта количества с.р.п.
в задаче 5.3.5, а не для выяснения существования с.р.п.)
Например, теорема Холла утверждает, что у системы S1 , . . . , Sm
конечных множеств есть система различных представителей тогда
и только тогда, когда | ∪i∈I Si | > |I| для любого I ⊂ {1, . . . , m}.
5.3.3. Пусть для системы m-элементных множеств каждый элемент, входящий хотя бы в одно из них, входит ровно в l из них.
Тогда при m > l у этой системы множеств есть с.р.п.
5.3.4. С.р.п. поднабора можно дополнить до с.р.п. всего набора.
Вот более подробная формулировка. Из набора M множеств выбрано несколько подмножеств S1 , . . . , Sk . Допустим, что элементы
5. СИСТЕМЫ МНОЖЕСТВ (ГИПЕРГРАФЫ)
129
x1 , . . . , xk — это с.р.п. набора множеств S1 , . . . , Sk ∈ M. Если у всего
набора M есть с.р.п., то существует его с.р.п., содержащая элементы x1 , . . . , xk .
5.3.5. Обозначим через F (S1 , . . . , Sm ) количество с.р.п. у системы
{S1 , . . . , Sm }.
(a) Для любого ли k существует система S1 , . . . , Sm такая, что
F (S1 , . . . , Sm ) = k?
(b) Найдите все возможные значения F (S1 , S2 ) при условии |S1 | =
|S2 | = 5.
(c)* Найдите все возможные значения F (S1 , S2 , S3 ) при условии
|S1 | = |S2 | = |S3 | = 5.
5.3.6.
∪mПусть даны
∪m два разбиения множества S на m подмножеств:
S = i=1 Ai = i=1 Bi , m 6 |S|. Пусть выполнено одно из следующих условий.
(a) Для любого подмножества {i1 , . . . , ik } ⊂ {1, . . . , m} множество
Ai1 ∪ . . . ∪ Aik содержит не более k из множеств B1 , . . . , Bm .
(b) |A1 | = . . . = |Am | = |B1 | = . . . = |Bm |.
Тогда можно перенумеровать множества A1 , . . . , Am так, чтобы после перенумерации Ai ∩ Bi ̸= ∅ для любого i = 1, . . . , m.
5.3.7.*
Пусть даны
∪m два разбиения множества S на m подмножеств:
∪m
S = i=1 Ai = i=1 Bi , m 6 |S|. Пусть для любых подмножеств
I, J ⊂ {1, . . . , m} выполнено неравенство:
(

) 
∪
∪


A
∩
B
i
j > |I| + |J| − m.
i∈I
j∈J
Тогда можно перенумеровать множества A1 , . . . , Am так, чтобы после перенумерации нашлись попарно различные элементы xi ∈ Ai ∩
Bi , i = 1, . . . , m.
Такой набор x1 , . . . , xm называются общей системой различных
представителей наборов множеств A1 , . . . , Am и B1 , . . . , Bm .
5.3.8. (a) При каком условии на любовь юношей и девушек можно
распределить всех девушек по непересекающимся гаремам, в каждом из которых ровно по две жены?
130
Элементы дискретной математики в задачах
Или, формально, найдите необходимое и достаточное условие на
двудольный граф, при котором вершины можно занумеровать A1 , . . . , An
(в первой доле) и B1 , C1 , . . . , Bn , Cn (во второй доле) так, что есть
рёбра A1 B1 , A1 C1 , . . . , An Bn , An Cn .
(b) Пусть есть m юношей и несколько девушек, каждый юноша
любит не менее t девушек, причем всех юношей можно женить
на любимых ими девушках (так, чтобы брачные пары не пересекались), т. е. есть паросочетание. Тогда имеется не менее
{
t!,
t6m
t!/(t − m)!, t > m
способов переженить юношей на любимых ими девушках.
5.4
Перманент
Перманент квадратной матрицы A = (aij ) размера n × n определяется формулой
Per(A) :=
n
∑ ∏
ai,σ(i) ,
σ∈Σn i=1
где Σn есть множество всех перестановок n-элементного множества.
5.4.1. Найдите
(
) перманент матрицы
a b
(2)
;
c d
(k) 4 × 4, у которой k = 0, 1, 2, 3, 4 диагональных элементов —
нули, а все остальные (в т.ч. не диагональные) элементы — единицы;
(n) n × n, у которой на диагонали нули, а вне диагонали — единицы.
Подматрицей данной матрицы называется матрица, полученная из данной вычеркиванием некоторого количества строк и столбцов. Перманент прямоугольной матрицы A определяется как сумма перманентов всех квадратных подматриц максимального размера. Или, формулой, при m < n
Per(A) :=
m
∑∏
σ i=1
ai,σ(i) ,
5. СИСТЕМЫ МНОЖЕСТВ (ГИПЕРГРАФЫ)
131
где сумма берётся по всем m-элементным размещениям чисел от 1
до n. При m > n положим Per(A) := Per(AT ).
5.4.2. Найдите перманент матрицы m × n, состоящей из одних единиц.
5.4.3. (a) Перманент не меняется при перестановке строк.
(b) Формула разложения по строке. Если m 6 n, то для любого i
Per(A) =
n
∑
aij Per(Aij ),
j=1
где Aij — матрица, получаемая из исходной вычеркиванием i-ой
строки и j-ого столбца.
5.4.4. (a) Перманент матрицы n×n из нулей и единиц равен нулю
тогда и только тогда, когда есть нулевая подматрица s×t, где s+t =
n + 1.
(b) Для любых m 6 n перманент прямоугольной матрицы m × n
из нулей и единиц равен количеству с.р.п. (§5.3) для системы из m
подмножеств множества {1, . . . , n}, определяемых строками.
5.5
Размерность Вапника-Червоненкиса
5.5.1. (a) Математики Вася и Чарли играют. Сначала Чарли отмечает на плоскости k точек. Затем Вася красит некоторые из этих
точек. Если теперь Чарли сможет провести прямую, отделяющую
покрашенные точки от непокрашенных, то он выиграл, иначе —
проиграл. При каком наибольшем k Чарли может выиграть независимо от действий Васи?
(b) То же, но точки отмечаются в пространстве, и Чарли проводит
полуплоскость.
Пусть R ⊂ 2X — семейство подмножеств произвольного множества X. Размерностью Вапника-Червоненкиса V C(X, R) (или VCразмерностью) пары (X, R) называется максимальное n такое, что
существует n-элементное подмножество A ⊂ X, для которого любое
подмножество в A является пересечением A и некоторого подмножества из R. Такое подмножество A называется дробящимся системой R. Если такого n не существует, то полагают V C(X, R) := ∞.
132
Элементы дискретной математики в задачах
Естественные примеры, в том числе пример с бесконечностью,
приведены в следующих задачах.
5.5.2. (a) Найдите VC-размерность семейства всех (двумерных замкнутых) прямоугольников на плоскости со сторонами, параллельными осям координат.
(b) Теорема. VC-размерность семейства всех полупространств в
Rn равна n + 1.
(c) Теорема Радона. Любые n + 2 точки в Rn можно разбить на
два множества, выпуклые оболочки которых пересекаются.
5.5.3. Найдите VC-размерность следующих семейств множеств:
(a) {{1, . . . , k} | k ∈ N};
(b) {{k, k + 1, k + 2, . . .} | k ∈ N};
(c) {{k, 2k, 3k, . . .} | k ∈ N};
(d) {{k, k 2 , k 3 , . . .} | k ∈ N};
5.5.4. Найдите VC-размерность следующих конечных семейств:
(a) {1, 2, 3}, {4, 5, 6}, {1, 2, 4} {1, 2, 5}, {2, 3, 6}, {3, 4, 5}, {3, 4, 6}, {2, 4, 5, 6}.
(b) {1, 2, 3}, {3, 4, 5}, {1, 2, 4} {1, 2, 5}, {2, 4, 5}, {2, 3, 5}, {2, 6, 7}, {3, 4, 6, 7}.
(c) {1, 2, 3}, {4, 5, 6}, {7, 8, 9}, {1, 4, 7}, {2, 5, 8}, {3, 6, 9}, {1, 6, 9}.
(d) Можно ли добавить ещё одно множество к системам из предыдущих пунктов так, чтобы VC-размерность увеличилась на 1?
5.5.5. (a) Возможно ли равенство V C(R2 , R) = ∞ для некоторого
2
набора R ⊂ 2R ?
2
(b) То же для некоторого счётного набора R ⊂ 2R ограниченных
множеств.
5.5.6. В любом семействе VC-размерности d, в каждом множестве
которого не более r элементов, найдутся такие подмножества X и
Y , что (a) |X ∩ Y | 6 r − d; (b) |X ∩ Y | > d − 1.
5.5.7. Если R ⊂ 2Rn и |R| = n, то для любого k = 1, 2, . . . , n найдётся такое множество A, что |{R ∩ A | R ∈ R}| > k = |A| + 1.
5.5.8. Если R ⊂ 2Rn — семейство VC-размерности d, то существует
наследственное (т. е. содержащее с каждым множеством все его
подмножества) семейство R′ ⊂ 2Rn VC-размерности d, для которого
(a) |R′ | 6 |R|; (b) |R′ | > |R|.
5. СИСТЕМЫ МНОЖЕСТВ (ГИПЕРГРАФЫ)
133
5.5.9. Если R ⊂ 2Rn , то
( ) ( ) ( )
(
)
n
n
n
n
|R| 6
+
+
+ ... +
.
0
1
2
V C(Rn , R)
5.6
Оценка Виссера мощности пересечений
1. В парламенте из 100 000 депутатов образовано k комиссий по
2 000 человек в каждой.
(a) Докажите, что если k > 100, то какие-то две комиссии имеют
хотя бы 21 общего члена.
(b) Докажите, что если k > 5 000, то какие-то две комиссии
имеют хотя бы 29 общих членов.
(c) Докажите, что если k > 250 000, то какие-то две комиссии
имеют хотя бы 32 общих члена.
(d) Докажите, что если k > 2 · 5030 , то какие-то две комиссии
имеют хотя бы 40 общих членов.
Подмножество отрезка [0, 1] называется хорошим, если оно является объединением конечного числа непересекающихся интервалов. Длиной |E| хорошего множества E называется сумма длин его
интервалов.
1’. (a) Если E1 , . . . , Ek — непересекающиеся хорошие множества
длины 1/k каждое и E0 — хорошее множество длины 1/k, то |E0 ∩
Ej | > 1/k 2 для некоторого j > 1.
(b) Придумайте пример бесконечного семейства хороших множеств длины 1/2 каждое, чтобы длина пересечения любых двух из
них не превосходила бы 1/4.
(c) То же для множеств длины 1/k и длины пересечения не более
2
1/k .
2’. Лемма Виссера. (a) Если E1 , . . . , Ek , . . . ⊂ [0, 1] — хорошие
множества длины m каждое, то найдутся i, j, для которых |Ei ∩
Ej | > m2 /2.
(b) То же, |Ei ∩ Ej | > 0, 99m2 .
(c) То же для хороших (т.е. являющихся объединением конечного количества многоугольников без границы с непересекающимися
внутренностями) подмножеств единичного квадрата и их площади.
134
Элементы дискретной математики в задачах
Задача 1’ поясняет роль множителей 0.99 и m2 в лемме Виссера.
2. Теорема Пуанкаре-Каратеодори о возвращении множеств.
(a) Пусть E ⊂ [0, 1] — хорошее непустое множество и f : [0, 1] →
[0, 1] — взаимно-однозначное отображение, сохраняющее хорошие
подмножества и их длины. Тогда существуют сколь угодно большие
n, для которых |E ∩ f n (E)| > 0.
Здесь через f n обозначается n-я итерация отображения f : f n =
f ◦ f ◦ . . . ◦ f (n раз). Для доказательства необходимо также f n с
n < 0: f n = f −1 ◦ f −1 ◦ . . . ◦ f −1 (|n| раз).
(b) То же для взаимно-однозначного отображения f круга единичной площади в себя (например, любого движения этого круга),
сохраняющего хорошие подмножества и их площади.
3. Теорема Хинчина. В условиях теоремы Пуанкаре-Каратеодори
(a) существуют сколь угодно большие n, для которых |E∩f n (E)| >
0, 99|E|2 .
(b) существует такое (большое) L, что для любого целого M
число n из пункта (a) можно найти среди чисел M, M +1, . . . , M +L.
4. (a) Если депутатов 2a , в каждой комиссии 2a−1 депутатов, а
комиссий 2a , то не обязательно найдутся две комиссии, имеющие
более чем 2a−2 общих членов.
(b) Если депутатов 1600, в каждой комиссии 80 депутатов, а
комиссий 60, то не обязательно найдутся две комиссии, имеющие
хотя бы 5 общих членов.
(c) Если депутатов p(s − 1)2 (p простое), в каждой комиссии
p(s − 1) депутатов, а комиссий p2 + p, то не обязательно найдутся
две комиссии, имеющие хотя бы s общих членов.
Теорема Фрэнкла-Вильсона. В парламенте из R депутатов
образовано k комиссий по n человек в каждой. Если p > 2 простое,
R/4−1
R = 4pα , n = R/2 и k > 2CR−1 , то обязательно найдутся две
комиссии, имеющие ровно R/4 общих членов.
5.7
Структуры на конечном множестве
В п. 1.4 книги Элементы дискретной математики в задачах,
А.А. Глибичук, А.Б. Дайняк, Д.Г. Ильинский, А.Б. Купавский, А.М.
Райгородский, А.Б. Скопенков, А.А. Чернов, Изд-во МЦНМО, 2015
5. СИСТЕМЫ МНОЖЕСТВ (ГИПЕРГРАФЫ)
135
рассмотрены некоторые классические и, на первый взгляд, различные комбинаторные задачи. На самом деле они являются частными
случаями общей проблемы. Цель предлагаемого цикла задач — рассказать об этой и близких проблемах.
Благодарю Никокошева Илью за полезные обсуждения.
Введем определения, которые пояснят связь между вышеприведенными задачами, а также помогут решить их. Рассмотрим множество Un целых чисел от 1 до n. Обозначим через 2Un множество
всех его подмножеств.
Алгеброй на множестве Un называется семейство его подмножеств, которое вместе с любыми подмножествами A и B содержит также их объединение A ∪ B, пересечение A ∩ B и дополнение Ā := Un − A. Например, 2Un — алгебра на Un ; {∅, {1, 2, 3}} и
{∅, {1}, {2, 3}, {1, 2, 3}} — алгебры на U3 .
Базисом алгебры называется наименьшее (по включению) её
подсемейство {X1 , . . . , Xk } такое, что любой элемент алгебры можно выразить через X1 . . . Xk с помощью операций пересечения, объединения и дополнения. Задача 5a предыдущего раздела равносильна нахождению наименьшего числа множеств в базисе алгебры 2Un .
Задача A. 1) Найдите все алгебры на U1 , U2 и U3 .
2) Разбиением множества Un называется неупорядоченный набор {X1 , X2 , . . . Xk } подмножеств Xi ⊂ Un , для которого Un =
X1 ∪ X2 ∪ · · · ∪ Xk и Xi ∩ Xj = ∅ при любых i ̸= j. Установите
взаимно-однозначное соответствие между алгебрами на Un и разбиениями множества Un .
3) Количество элементов произвольной алгебры есть степень
двойки 2k .
4) Размеры разных базисов одной алгебры могут быть различны.
Линейным пространством на множестве Un называется семейство его подмножеств, которое вместе с любыми подмножествами
A и B содержит также их симметрическую разность A ⊕ B. Например, любая алгебра является линейным пространством; {∅},
{∅, {1, 2}}, {∅, {1}, {2}, {1, 2}} и {∅, {1, 3}, {2, 3}, {1, 2}} — линейные пространства на U3 . Определение базиса линейного пространства и его связь с задачей L предыдущего раздела аналогичны
136
Элементы дискретной математики в задачах
случаю алгебр.
Задача L. 1) Любое линейное пространство содержит ∅.
2) Найдите все линейные пространства на U1 , U2 и U3 .
3) В линейном пространстве не может быть ровно 3 элемента.
4) Количество элементов произвольного линейного пространства есть степень двойки 2k . Число k есть размер любого его базиса.
Оно называется размерностью линейного пространства. В частности, размеры любых двух базисов данного линейного пространства
одинаковы.
Топологией на множестве Un называется семейство его подмножеств, которое содержит ∅, Un и вместе с любыми подмножествами
A и B содержит также A∩B, A∪B. Например, любая алгебра является топологией; {∅, {1}, {1, 2, 3}} и {∅, {1}, {2}, {1, 2}, {1, 3}{1, 2, 3}}
— топологии на U3 . Определение базиса топологии и его связь с задачей 5b предыдущего раздела аналогичны случаю алгебр.
Задача T. 1) Найдите все топологии на U1 , U2 и U3 . Убедитесь,
что существует топология, число элементов которой не является
степенью двойки.
2) Любая ли топология является линейным пространством? Можно ли симметрическую разность выразить через пересечение и объединение?
Задача 2.* (a) Найдите рекуррентную формулу для числа NA (n)
всех алгебр (числа Белла).
(l) Для числа NL (n) линейных пространств докажите NL (2n) ∼
2
C · 2n .
(t) Найдите количество топологий на n-элементном множестве
(нерешенная задача).
Задача 3. Цепью топологий называется последовательность
топологий {∅, Un } = A0 ( A1 ( A2 · · · ( Ak = 2Un , между любыми двумя соседними членами которой нельзя вставить ещё одну
топологию (т.е. для любого i не существует топологии B, для которой Ai ( B ( Ai+1 ). Аналогично определяются цепи алгебр и
линейных пространств.
(1) Все цепи алгебр (линейных пространств) имеют одинаковую
длину (какую?).
5. СИСТЕМЫ МНОЖЕСТВ (ГИПЕРГРАФЫ)
137
(2) Приведите пример цепей топологий различной длины.
(3*,4) Найдите наибольшую длину DT+ (n) и наименьшую длину
DT− (n) цепей топологий.
Задача 4. (l) Шириной WL (n) (семейства всех линейных пространств на Un ) называется максимальное количество элементов
линейных пространств, ни одно из которых не содержится (собственно) в другом. Найдите WL (n).
2
(l’)* WL (2n) ∼ C1 ·2n .
(a) Найдите WA (n).
(t)* Найдите
WT (n) (нерешенная задача).
Задача 5. Базой на множестве Un называется семейство его подмножеств, которое содержит Un и вместе с любыми подмножествами A и B содержит также A ∩ B. Примеры баз: любая топология;
{{1, 2}, {2, 3}, {2}, U4 } — база на U4 .
Вообще, структурой на множестве Un называется семейство его
подмножеств, которое замкнуто относительно некоторого заданного
набора операций. Например, если заданный набор операций пуст,
то решетка структур совпадает с 2n -мерным кубом. Определение
базиса структуры и ее других характеристик аналогичны случаю
алгебр.
Решите для других типов структур аналоги приведенных выше
задач. Полный список различных типов структур назывется решеткой Поста.
Рисунок: алгебры на U3 и соответствующие разбиения
Указания.
A.1. Все алгебры на U2 нарисованы на рис. 1.a.
A.2. Разбиению Un = X1 ∪X2 ∪· · ·∪Xk поставьте в соответствие
семейство всех множеств, полученных объединением некоторых из
X1 , . . . , Xk . На рис. 1.b нарисованы разбиения множества U2 , соответствующие алгебрам с рис 1.a.
A.3. Следует из задачи A.2.
A.4. Рассмотрите алгебру 2U4 и два базиса, один из множеств
{1, 2} и {1, 3}, другой из множеств {1}, {2}, {3}.
L.4. Докажите существование какого-либо базиса.
Введем определения, которые пояснят связь между различными
пунктами задач 2, 3, 4 и 5, а также помогут решить их.
138
Элементы дискретной математики в задачах
Нарисуем все алгебры (на Un — эти слова мы дальше опускаем).
Проведем стрелку от алгебры A к алгебре B, если A ( B и между
ними нельзя вставить никакую другую алгебру. Полученный граф
называют решёткой алгебр. На рисунке нарисована решётка алгебр
на U3 (указаны разбиения, соответствующие алгебрам).
Разбиение H = H0 ⊔ H1 ⊔ · · · ⊔ Hm множества всех алгебр называется разбиением на этажи, если для любых двух соединенных
стрелкой алгебр A ⊂ B номер этажа алгебры A на единицу меньше
номера этажа алгебры B.
Ясно, что решётка алгебр допускает разбиение на этажи. (Какие
алгебры находятся на k-м этаже?)
Аналогично случаю алгебр вводятся понятия решётки линейных пространств и ее разбиения на этажи. Ясно, что решётка
линейных пространств допускает разбиение на этажи. (Какие линейные пространства находятся на k-м этаже?)
Аналогично случаю алгебр вводятся понятия решётки топологий и ее разбиения на этажи. Но решётка топологий не допускает
разбиения на этажи.
Ответы: DA (n) = DL (n) = n+1, DT+ (n) = 2n+2, DB (n) = 2n +1.
Указания и решения
Некоторые решения к п. 5.6 написаны А. Пахаревым.
5.6.1. (a) Если любые две комиссии имеют не более 20 общих
членов, то число всех парламентариев не меньше 100 · 2000 − (100 ·
99/2)·20 = 101 000 > 100 000 (это частный случай формулы включенийисключений). Полученное противоречие доказывает нужное утверждение.
(b) Примените рассуждение из пункта (a) к декартову квадрату парламента, т.е. парламентом будет множество пар депутатов, а
комиссиями множество пар депутатов из одной комиссии.
(с) То же для декартова куба.
40
(d) Найдите N такое, что N√ > 39.
2
5.6.2’. Аналогично задаче 5.6.1.
Комментарий: из нее сразу же вытекает, что в таком семействе
существуют 2n множеств с длиной пересечения сколь угодно близ-
5. СИСТЕМЫ МНОЖЕСТВ (ГИПЕРГРАФЫ)
139
n
кой к m2 . Интересно было бы доказать не для степеней двойки, а
для всех натуральных чисел.
5.6.3. (b) Отрицанием к утверждению задачи будет существование бесконечно больших интервалов из ”плохих“ целых чисел, т.е.
таких, которые не дают неравенства в нужную сторону. Пусть l1
— середина одного из них (под серединой будем иметь ввиду любое ближайшее к полусумме концов целое число). Далее, пусть l2 —
середина интервала с длиной, большей |2l1 |, и т.д. Тогда нетрудно
заметить, что при n > m число ln − lm содержится в интервале с
серединой ln , т.е. оно плохое. Но тогда |f ln (E) ∩ f lm (E) < 0, 99|E|2
для любых натуральных n и m, что противоречит задаче 2’b.
5.6.4. (b) Разобьем всех парламентеров на 400 = 20 · 20 четверок. Занумеруем четверки парами (i, j) целых чисел, 1 6 i, j 6 20.
Каждая комиссия будет объединением двадцати четверок. Первая
комиссия является объединением четверок (1, j), 1 6 j 6 20. Вообще, i-ая комиссия при 1 6 i 6 20 является объединением четверок
(i, j) по 1 6 j 6 20. А j-ая комиссия при 21 6 j 6 40 является объединением четверок (i, j − 20) по 1 6 i 6 20. И k-ая комиссия при
41 6 k 6 60 является объединением всех четверок (i, j), в которых
i − j − k делится на 20 и 1 6 i, j 6 20. Тогда любые две комиссии
имеют менее 5 (более точно, 0 или 4) общих членов.
(с) См. Квант, 1997, N2, стр. 24, решение задачи M1566 (этот
пример принадлежит Н. Б. Васильеву).
5.8
Подсолнухи
Подсолнухом с k лепестками и ядром Y называют такой набор
множеств {F1 , . . . , Fk }, что |Fi ∩ Fj | = Y при i ̸= j и все множества
Fi \ Y непусты. Например,
• попарно непересекающиеся множества образуют подсолнух с
пустым ядром.
• одномерные векторные подпространства (§7.1) образуют подсолнух с одноэлементным ядром.
• множества Xp всех рациональных дробей с фиксированным
простым знаменателем p образуют подсолнух с бесконечным ядром.
Большая часть задач этого раздела взята из книги [J].
140
Элементы дискретной математики в задачах
5.8.1. Найдите размер минимальной с.о.п. (§5.2) подсолнуха.
5.8.2. (a) Если |F| > s!(k − 1)s , то в F найдётся подсолнух с k
лепестками.
(b) Найдутся (k − 1)s подмножеств конечного множества, в каждом из которых s элементов и среди которых нельзя выбрать подсолнух с k лепестками.
Кроме подсолнухов, можно рассматривать также другие конфигурации множеств, задаваемые условиями на пересечения. Мы
предлагаем читателю самостоятельно приводить интересные примеры таких конфигураций.
Пусть, например, попарные пересечения не обязательно равны,
однако содержат одинаковое количество элементов. Слабой
∆-системой называется такой набор множеств S1 , . . . , Sk , что
|Si ∩Sj | одинаковы при всех i ̸= j. Очевидно, что подсолнух является слабой ∆-системой. Обратное неверно. Однако есть следующая
теорема.
Теорема Деза. Если F — слабая ∆-система из s-элементных
множеств и |F| > s2 − s + 2, то F — подсолнух.
Доказательство теоремы достаточно сложное.
Покажем, что приведенная оценка точна.
5.8.3. Для любого простого числа p существует слабая ∆-система
(p + 1)-элементных множеств, не являющаяся подсолнухом и состоящая из p2 + p + 1 множеств.
5.8.4. Общая часть множеств S1 , . . . , Sk — это объединение
∪
i̸=j (Si ∩ Sj ) всех их попарных пересечений.
(a) Если F — конечный набор s-элементных множеств и |F| >
(k − 1)s , то найдутся k множеств из F, в общей части которых
менее s элементов.
(b) Оценка |F| > (k − 1)s в пункте (a) точна.
5.8.5. Цветком с k лепестками и ядром Y называется такой набор
F множеств, что каждое из них содержит Y и не существует с.о.п.
из k − 1 элемента для набора {S \ Y : S ∈ F }.
(a) Если F — конечный набор s-элементных множеств и |F| >
(k −1)s , то F содержит цветок с k лепестками (и некоторым ядром).
5. СИСТЕМЫ МНОЖЕСТВ (ГИПЕРГРАФЫ)
141
(b) Оценка |F| > (k − 1)s из пункта (a) точна.
5.9
Подсказки
5.1.3 . (a) Среди n − k + 1 подмножеств
{1, 2, . . . , k}, {2, 3, . . . , k + 1}, . . . , {n − k + 1, n − k + 2, . . . , n}
не более k лежат в F.
5.2.2 . (a) Возьмите объединение.
5.2.6 . (a) Используйте жадный алгоритм.
2
(d) Используйте неравенство (1 − x)−1 6 ex+x для малых x.
(e) Используйте (c,d).
(f) Подсчет двумя способами (=вероятностный метод), §1.6.
5.3.1 . (b) Примените лемму о паросочетаниях. Нарисуем двудольный граф: вершины долей — это элементы и множества, ребро (x, S)
проведено тогда и только тогда, когда x ∈ S.
Несложно доказать утверждение и по индукции. Рассмотрите отдельно случай, когда найдётся подмножество I ⊂ {1, . . . , m} такое,
что |∪i∈I Si | = |I|.
Можно доказывать утверждение и в эквивалентной форме 5.4.4.a,
см. задачу 5.4.4.b.
5.3.3 . Примените теорему Холла 5.3.1.b.
5.3.4 . Достаточно доказать утверждение для |S| = k + 1. Для этого надо проверить, что условие невозможности дополнения с.р.п.
противоречит условию теоремы Холла для S.
5.3.5 . (a) Ответ: да.
(b) F (S1 , S2 ) = 25 − |S1 ∩ S2 |.
142
Элементы дискретной математики в задачах
(c)
F (S1 , S2 , S3 ) = 53 − 5(|S1 ∩ S2 | + |S2 ∩ S3 | + |S1 ∩ S3 |) + 2|S1 ∩ S2 ∩ S3 |.
5.4.1 . Можно применить задачу 5.4.3.
5.4.4 . Примените определение перманента.
5.5.2 . (a) Ответ: 4.
(b) Примените теорему Радона (c).
5.5.3 . Ответы: (a) 1; (b) 1; (c) ∞; (d) ∞.
5.5.4 . (a,b,c) Ответы: 2.
(d) Можно только в пункте (c).
5.5.5 . В обоих пунктах ответ — да.
5.5.7 . Можно использовать индукцию по k вверх.
5.5.8 . Можно считать, что максимальное дробящееся подмножество
— Rd .
5.5.9 . Для любых i ∈ Rn и M ⊂ Rn обозначим через Ri (M ) множество M \ {i}, если M \ {i} ̸∈ R, и M иначе. Т. е. мы уменьшаем множество, если уменьшенного нет в нашей системе. Обозначим
Ri := {Ri (M ) | M ∈ R}. Тогда |Ri | = |R|.
Утверждение. Если множество дробится системой Ri , то
оно дробится и системой R.
5.8.5 . (a) Решение аналогично решению задачи 5.8.2.a.
(b) Используйте решение задачи 5.8.2.b.
152
6
6.1
Элементы дискретной математики в задачах
Аналитические и вероятностные методы
Асимптотики
Если не оговорено противное, то o, O (рукописные обозначения: o, O), асимптотики и пределы рассматриваются при n → ∞.
f (n)
Запись f (n) ≪ g(n) означает, что f (n) = o(g(n)), т.е. lim
= 0.
n→∞ g(n)
Запись f (n) & g(n) означает, что f (n) > (1 + o(1))g(n).
Найти асимптотику для функции f (n) означает найти «явную»
f (n)
функцию a(n), для которой lim
= 1.
n→∞ a(n)
6.1.1. Найдите асимптотику для
(a,b,c) сумм из задачи 1.1.6;
(d) количества An подмножеств множества {1, 2, . . . , n}, не содержащих двух подряд идущих чисел;
(e)* То же, что в (d), для трёх подряд идущих чисел.
В ответе можно использовать функцию xP (a, b), которая по числам
a, b и многочлену P , имеющему единственный корень на отрезке
[a, b], выдает этот корень.
6.1.2. Найдите асимптотику наибольшего количества рёбер в графе
с n вершинами, не содержащем k-клики. Здесь k = kn = o(n).
6.1.3. Докажите следующие соотношения, предполагая в асимптотиках, что n → ∞, а k фиксировано. (Число exH (n) определено в
задаче 2.7.6.)
n2
k−1
(a) exPk (n) & k−2
2 n. (b) exC2k+1 (n) & 4 . (c) exK1,k (n) ∼ 2 n.
Замечание. Знаменитая теорема Эрдеша-Стоуна-Шимоновича
утверждает, что для любого фиксированного H такого, что χ(H) >
n2 χ(H) − 2
2, при n → ∞ выполнено exH (n) ∼
. (Для двудольных
2 χ(H) − 1
H известно лишь, что exH (n) = o(n2 ).) То есть, если мы запрещаем
графу иметь некоторый фиксированный подграф H, то доля рёбер, которые при этом можно провести, среди всевозможных рёбер
определяется хроматическим числом графа H. Удивительно, что
6. АНАЛИТИЧЕСКИЕ И ВЕРОЯТНОСТНЫЕ МЕТОДЫ
153
хроматическое число возникает в этой задаче! Доказательство теоремы можно прочесть по ссылке [1]. (Этой теоремой нельзя пользоваться при решении задачи 6.1.3.b.)
)n
(
6.1.4. (a) n2 2 + n1 = (2+o(1))n . По определению, это
( означает,
)n
2
что существует функция ψ(n) = o(1), для которой n 2 + n1
=
√ (
)n
(2 + ψ(n))n ; или, что то же самое, n n2 2 + n1 − 2 = o(1).
√ (
)n
(b) 3 n 2 + n1 = (2 + o(1))n .
√(
)
n
n
6.1.5. (a) Найдите асимптотику для
(ср. с задачами
[n/2]
1.4.3.b и 6.1.10.b).
(
)
2n
n
<
< 2n .
(b)
n+1
[n/2]
√( )
3m
(c) Найдите асимптотику для m
.
m
( )
33m
3m
(d)
< 22m
< 33m .
3m +)1
m
(
n
(e)
= (a−a (1 − a)a−1 + o(1))n .
[an]
n!
(f)
=
(e−a1 ln a1 −...−as ln as + o(1))n , где
[a1 n]! . . . [as n]!
a1 + . . . + as = 1.
6.1.6. (a) Найдите асимптотику
для ln(n!).
√
n
(b) Найдите асимптотику для n!.
(c) nn e−n+1 6 n! 6 nn+1 e−n+1 ;
√
(d) n! 6 nn e−n+1 n.
√
(e)* Формула Стирлинга. n! ∼ nn e−n 2πn, т. е. lim
n→∞
1.
n!
√
nn e−n 2πn
=
( )
n
nk
6.1.7. (a)
<
.
k
k!
(
( ))
(n − 1)(n − 2) . . . (n − k)
k
k(k + 1)
(b) ln
1
+
O
для k =
=
−
nk
2n
n
( )
kn < n/2. Это означает, что существует функция ψ(n) = O nk , для
154
Элементы дискретной математики в задачах
(n − 1)(n − 2) . . . (n − k)
k(k + 1)
=
−
(1 + ψ(n));
2n
nk
2n
(n − 1)(n − 2) . . . (n − k)
или, что то же самое, −1 −
ln
=
k
k(k
+
1)
n
( )
k
O
.
n( )
n
nk − k(k−1) +O(k3 /n2 )
(c)
=
e 2n
для k = kn < n/2.
k!
k
которой
ln
k(k−1)
3
2
(Сформулируйте
сами, что здесь означает e− 2n +O(k /n ) .)
( )
√
n
nk
(d)
∼
для k = kn = o( n).
k
k!
√
Неформально, это означает, что для k ≪ n вероятность выпадения ровно k орлов при n подбрасываниях монеты приближенно
nk −n
равна
2 . В неформальном замечании к этому и следующему
k!
пунктам достаточно интуитивного понимания того, что такое вероятность.
(
) ( )
2
2n
2n
− kn (1+o(1))
(e)
/
=e
для k = kn = o(n).
n−k
n
k2
(Сформулируйте сами, что здесь означает e− n (1+o(1)) .)
Неформально, это означает, что для k ≪ n вероятность Pk выпадения ровно n−k орлов при 2n подбрасываниях монеты приближенно
2
равна P0 e−k /n (нормальное распределение).
6.1.8. (a) Верно ли, что записи eo(n) и o (en ) «равнозначны»?
т. е., верно ли, что для любой функции f : Z → (0, +∞) условия
ln f (n)
lim
= 0 и lim f (n)e−n = 0 равносильны?
n→∞
n→∞
n
(b) Подберите функции f, g : Z → (0, +∞) такие, что f (n) ∼ g(n),
но ef (n) ̸= O(eg(n) ).
(c) Могут ли функции f, g : Z → (0, +∞) одновременно удовлетворять соотношениям f (n) = o(g(n)) и g(n) = o(f (n))?
(d) Могут ли функции f, g : Z → (0, +∞) одновременно удовлетворять соотношениям f (n) = O(g(n)) и g(n) = O(f (n))?
(e) Следует ли из двух соотношений из (d), что f (n) ∼ g(n)?
6.1.9. (a) Какая функция растет быстрее: x(x
x
x
найдите lim x(x ) (x!)(−2 ) .
x→∞
x)
x
или (x!)(2 ) ? т. е.
6. АНАЛИТИЧЕСКИЕ И ВЕРОЯТНОСТНЫЕ МЕТОДЫ
155
(b) Существует √ли функция ψ(n) = o(1), для которой
(2 + ψ(n))n 2−n e− n → ∞?
(Как в любой математической задаче, нужно обосновать ответ: привести пример такой функции или доказать её существование или
доказать, что такой функции не существует.)
В задачах 6.1.10.bcde и 6.1.11.cef, в отличие от остальных, можно
пользоваться без доказательства формулой Стирлинга 6.1.6.e.
6.1.10. Найдите
(
)
( 2)
( 2 ) асимптотику для
n
n
n
(a) ln
;
(b)
(c)
;
[n/2]
n
n
(
)
n
(d)*
(e) (2n−1)!! := (2n−1)·(2n−3)·. . .·3·1;
, α ∈ (0, 1);
[nα ]
n ( )2
n ( )4
∑
∑
n
n
(f)
;
(g)*
.
k
k
k=0
k=0
6.1.11. Найдите асимптотику функции s = s(n), заданной как
(a) ss = n;
3
(b) ss = n;
(c) s(n) := max{k
{ | k! 6 n};
}
(n)
m
(
)
(d) s(n) := min m ∈ N | m < 2 2 ;
(e) s(n) := min {m ∈ N | 2m /m > n} (функция 2m /m возникает
как сложность реализации
функций алгебры
логики);
{
}
( m )
(f) s(n) := min m ∈ N | [m/2] > n (ср. с задачей 1.4.3.b).
6.1.12.* В ответах можно использовать константы, заданные в виде
суммы рядов. Найдите асимптотику для
(a) количества линейных подпространств в Zn2 (см. задачу 1.4.7 и
определение перед ней);
(b) количества унициклических графов с n вершинами (см. задачу
2.2.5.b и определение перед ней).
6.2
Независимость и доказательства существования
Введение
Цель этого раздела — продемонстрировать метод доказательства некоторых интересных комбинаторных результатов (пункты
156
Элементы дискретной математики в задачах
(b) задач 6.2.1-6.2.4 и задачи 6.2.16-6.2.26), заключающийся в применении локальной леммы Ловаса 6.2.15.b.
Следующие две части введения важны, но формально не используются далее.
Об открытии леммы Ловаса и ее роли в математике. Локальная лемма Ловаса была доказана в 1973 году выдающимся венгерским математиком Ласло Ловасом. Впрочем, тогда Ловасу было
всего 25 лет и, хотя яркие результаты у него уже к тому времени
были, все-таки на тот момент его воспринимали не как классика,
но как восходящую звезду. Он уже был трехкратным победителем
международных математических олимпиад (1964, 1965 и 1966 годов). Классиком Ловас станет позже, и весьма серьезную роль в
этом сыграет доказанная им Локальная лемма. Разумеется, не только она: будет и топологический метод в комбинаторике, и мощные
результаты в теории алгоритмов, и значительный вклад в науку о
графовых пределах, и многое другое. Тем не менее, Локальная лемма — это замечательный инструмент вероятностной комбинаторики, благодаря которому были получены и продолжают получаться
многочисленные яркие результаты в области дискретной математики и теории алгоритмов.
Работа, в которой Ловас формулирует и доказывает свою Локальную лемму, написана в соавторстве с Полом Эрдешем — еще
одним великим специалистом по комбинаторике, основателем большой научной школы, автором множества задач и идей. Среди прочего, Эрдеш был одним из самых активных пропагандистов вероятностного метода в комбинаторике. Поэтому, несмотря на то, что
Локальную лемму доказал именно Ловас, роль Эрдеша во всем этом
не стоит недооценивать. В статье Эрдеша и Ловаса [EL] речь шла о
раскрасках гиперграфов (т.е. наборов подмножеств конечного множества). Как раз ради доказательства существования некоторой
раскраски Локальная лемма и придумывалась (т.е. ради обобщения задач 6.2.1, 6.2.16 и 6.2.17; не бойтесь, они формулируются и
решаются без слова ‘гиперграф’). Однако очень быстро стало ясно, насколько это мощный и плодотворный инструмент. Например,
почти сразу же с его помощью Дж. Спенсер улучшил нижнюю оценку числа Рамсея (см. определение в п. 4.1 и задачу 6.2.21), которая
6. АНАЛИТИЧЕСКИЕ И ВЕРОЯТНОСТНЫЕ МЕТОДЫ
157
не поддавалась улучшению в течение сорока лет. Сейчас диапазон
применения леммы становится все шире. Здесь теория графов и
гиперграфов, здесь экстремальные задачи комбинаторики, теория
алгоритмов и даже комбинаторная геометрия и теория диофантовых приближений.
За прошедшие десятилетия появились разнообразные усовершенствования Локальной леммы, многие из которых уже лишь отдаленно напоминают первоначальный вариант. И это еще одно свидетельство исключительной плодотворности идеи Ловаса.
Как устроено изложение в этом разделе. Основные идеи демонстрируются по одной и на ‘олимпиадных’ примерах, т.е. на простейших частных случаях, свободных от технических деталей. Мы
показываем, как можно придумать лемму Ловаса. Путь к ее доказательству и применениям намечен в виде задач (всех задач этого
и следующего разделов, кроме задач 6.2.6 и 6.2.7, которые просто
поясняют понятие независимости). Обучение путем решения задач
не только характерно для серьезного изучения математики, но и
продолжает древнюю культурную традицию. 4
К важнейшим задачам приводятся указания и решения.
Обычно лемму Ловаса излагают на вероятностном языке. Однако, по нашему мнению, приводимое комбинаторное изложение более
доступно и полезно для начинающего. Важно излагать вероятностные идеи (например, независимости) и развивать вероятностную
интуицию, но при этом сохранять строгость изложения. Разумнее
делать это, не определяя понятия вероятностного пространства. 5
Это как раз подготовит начинающего к введению этого довольно
абстрактного понятия, ср. с [Z, философски-методическое отступление]. Кроме того, вероятностной интуиции начинающего противоречит получение вероятностными методами абсолютно (а не с некоторой вероятностью) верного результата. 6 (Впрочем, для человека,
4
Например, послушники дзенских монастырей обучаются, размышляя над
загадками, данными им наставниками. Впрочем, эти загадки являются скорее
парадоксами, а не задачами. См. подробнее [S].
5
Отличие элементарной теории вероятностей от перечислительной комбинаторики скорее в том, что речь идет о долях вместо чисел, и интерес часто
представляют оценки, а не равенства.
6
Объяснять, как с помощью вероятностных методов можно получить абсо-
158
Элементы дискретной математики в задачах
уже владеющего понятием вероятностного пространства, изложение на вероятностном языке не хуже комбинаторного.)
Приведем интересные факты, которые можно доказать при
помощи леммы Ловаса и вряд ли можно доказать без нее! Видимо, из задач 6.2.1-6.2.4 вы сможете решить сейчас только пункты
(a). К пунктам (b) разумно вернуться после изучения следующего
раздела. Более того, задача 6.2.2.b естественнее по формулировке,
но сложнее двух следующих.
6.2.1. (a) По каждому из 100 видов работ в фирме имеется ровно 8 специалистов. Каждому сотруднику нужно дать выходной в
субботу или в воскресенье. Докажите, что это можно сделать так,
чтобы и в субботу, и в воскресенье для каждого вида работ присутствовал специалист по нему. (Сотрудник может быть специалистом
по нескольким видам работ; распределение специалистов по видам
работ известно тому, кто назначает выходные. Это задача 1.6.7.)
(b) По каждому из нескольких видов работ в фирме имеется
ровно 8 специалистов. (Теперь видов работ не обязательно 100.)
Каждый вид работ имеет общих специалистов не более чем с 30 другими видами. Каждому сотруднику нужно дать выходной в субботу
или в воскресенье. Докажите, что это можно сделать так, чтобы и
в субботу, и в воскресенье для каждого вида работ присутствовал
специалист по нему.
Замечание. Для каждого вида работ x обозначим через Ax множество распределений выходных, при которых и в субботу, и в
воскресенье на работе есть специалист по x. Нужно доказать, что
∩x Ax ̸= ∅. В п. (a) это делается путем подсчета количества элементов. В п. (b) этого уже не хватает, нужна идея из следующего
раздела. Там мы покажем, как независимость (определенную там)
можно применять для оценки количества элементов в пересечении
множеств.
Описанную идею можно сформулировать так. Нужное условие
мы представляем в виде пересечения некоторого числа условий.
лютно верный результат, лучше на более простых примерах. См., например,
задачи 6.2.5, 6.2.10, 6.2.11 и 6.3.3.ab. Мы хотели бы сделать этот текст доступным даже для тех, кто не разбирал таких примеров.
6. АНАЛИТИЧЕСКИЕ И ВЕРОЯТНОСТНЫЕ МЕТОДЫ
159
При этом ясно, что для каждого из них есть конструкция, ему удовлетворяющая. Иногда отсюда можно вывести, что есть конструкция, удовлетворяющая всем этим условиям одновременно! Эта идея
часто применяется в математике. (Для читателя, знакомого с соответствующими понятиями, напомним, что в анализе так доказывается существование решения дифференциального уравнения, в топологии — вложимость n-мерного компакта в R2n+1 , ср. [S6, §2].)
Число условий может быть бесконечно, поэтому идея пересечения
‘равносильна’ идее итерационного процесса. А в настоящей заметке
мы покажем, как применять эту идею в комбинаторике. Несмотря
на конечность числа условий, ее применение весьма нетривиально.
6.2.2. (a) По кругу стоит 200 студентов из 10 групп, в каждой из
которых 20 студентов. Докажите, что можно в каждой группе выбрать старосту так, чтобы никакие два старосты не стояли рядом.
(b) То же для 1600 студентов из 100 групп, в каждой из которых
16 студентов.
6.2.3. (a) Докажите, что можно раскрасить первые 8 натуральных
чисел в 2 цвета так, чтобы не было одноцветной арифметической
прогрессии длины 3. (Это задача 4.4.5.b.)
(b) Докажите, что можно раскрасить первые 15 миллионов натуральных чисел в 2 цвета так, чтобы не было одноцветной арифметической прогрессии длины 32.
6.2.4. (a) Докажите, что для любого M ∈ R можно раскрасить все
вещественные числа в 2 цвета так, чтобы для любого x ∈ R числа
x и x + M были не одного цвета.
(b) Докажите, что для любых 25 чисел M1 , . . . , M25 ∈ R можно
раскрасить все вещественные числа в 3 цвета так, чтобы для любого
x ∈ R среди чисел x, x + M1 , . . . , x + M25 были числа каждого из
трех цветов.
Решения пунктов (b) вышеприведенных задач основаны на идее,
аналогичной решению задачи 6.2.1.b.
Для удобства читателя этот раздел структурирован более тонко,
чем остальные. В частности, некоторые указания и решения приведены прямо в нем (а не в конце параграфа).
160
Элементы дискретной математики в задачах
Независимость и лемма Ловаса
Приведем задачи, которые подведут нас к лемме Ловаса 6.2.15.b
(почему она интересна, написано в предыдущем разделе).
6.2.5. Каждый житель города либо здоров, либо болен, а также
либо богат, либо беден. Богатство и здоровье независимы, т.е. доля
богатых здоровых среди богатых равна доле здоровых среди всех
жителей. Известно, что есть богатый горожанин и есть здоровый горожанин. Обязательно ли найдется богатый здоровый горожанин?
Подмножества A и B конечного множества M называются независимыми, если
|A ∩ B| · |M | = |A| · |B|.
При B ̸= ∅ это равносильно тому, что доля множества A ∩ B в B
равна доле множества A в M .
6.2.6. Зависимы ли следующие подмножества? (Мы называем зависимыми подмножества, не являющиеся независимыми.)
(a) В множестве всех клеток шахматной доски подмножество
клеток в первых трех ее строках и подмножество клеток в последних четырех ее столбцах.
(b) Подмножества {1, 2} ⊂ {1, 2, 3, 4} и {1, 3} ⊂ {1, 2, 3, 4}.
(c) Подмножества {1, 2} ⊂ {1, 2, 3, 4, 5, 6} и {1, 3} ⊂ {1, 2, 3, 4, 5, 6}.
6.2.7. Зависимы ли следующие подмножества множества целых чисел от 1 до 105?
(a) Подмножество чисел, делящихся на 5, и подмножество чисел,
делящихся на 7.
(b) Подмножество чисел, делящихся на 15, и подмножество чисел, делящихся на 21.
(c) Подмножество чисел, делящихся на 15, и подмножество чисел, делящихся на 5.
(d) Подмножество чисел, делящихся на 10, и подмножество чисел, делящихся на 7.
6.2.8. (Ср. с замечанием после задачи 6.2.1.b.) Зависимы ли следующие подмножества множества всех раскрасок чисел 1, 2, . . . , 400 в
два цвета?
6. АНАЛИТИЧЕСКИЕ И ВЕРОЯТНОСТНЫЕ МЕТОДЫ
161
(a) Подмножество раскрасок, для которых {1, 2, . . . , 8} одноцветно, и подмножество раскрасок, для которых {11, 12, . . . , 18} одноцветно.
(b) Подмножество раскрасок, для которых {1, 2, . . . , 8} неодноцветно, и подмножество раскрасок, для которых {11, 12, . . . , 18} неодноцветно (ср. с задачей 6.2.1.b).
(c) Подмножество раскрасок, для которых {1, 2, . . . , 8} одноцветно, и подмножество раскрасок, для которых {6, 7, . . . , 13} одноцветно.
6.2.9. Подмножества A и B конечного множества независимы тогда
и только тогда, когда A и B независимы.
6.2.10. (a) Обязательно ли найдется богатый здоровый умный горожанин, если в городе доля богатых горожан больше 2/3, доля
здоровых больше 2/3 и доля умных больше 2/3?
(b) Тот же вопрос, если в городе есть богатый горожанин, есть
здоровый горожанин и есть умный горожанин, богатство, здоровье
и ум попарно независимы, и доля богатых здоровых умных среди
богатых здоровых такая же, как и доля умных среди всех жителей.
(Вместе с условием попарной независимости последнее условие называется независимостью в совокупности.)
(c) Тот же вопрос, если в городе богатых горожан больше половины, здоровых больше половины, умных больше половины, богатство и ум независимы, здоровье и ум независимы.
Задача 6.2.10 показывает, что чем сильнее условие, характеризующее независимость нескольких множеств, тем меньшей доли каждого множества достаточно, чтобы гарантировать непустоту пересечения. Причем наиболее интересный результат (6.2.10.c) получается «посередине» между крайними условиями — полного отсутствия
независимости (6.2.10.a) и независимости в совокупности (6.2.10.b).
Так часто бывает: наиболее полезные соображения находятся между «крайними» точками зрения.
6.2.11. (a) Пусть A1 , A2 , A3 , A4 — подмножества 720-элементного
множества, в каждом из которых более 480 элементов. Если Ak и
Ak+1 независимы для любого k = 1, 2, 3, то A1 ∩ A2 ∩ A3 ∩ A4 ̸= ∅.
162
Элементы дискретной математики в задачах
(b) Пусть n > 2 и A1 , A2 , . . . , An — подмножества конечного мно1
жества, доля каждого из которых больше 1−
. Если Ak и Ak+1
n−1
независимы для любого k = 1, 2, . . . , n − 1, то A1 ∩ A2 ∩ · · · ∩ An ̸= ∅.
Подробнее о независимости см. [KZP].
Для формулировки леммы Ловаса нужно еще более «хитрое»
условие независимости на несколько множеств, чем рассмотренные
ранее.
Подмножество A конечного множества M называется независимым от набора подмножеств B1 , . . . , Bk ⊂ M , если A независимо с любым подмножеством, являющимся пересечением нескольких
(возможно, одного) множеств из B1 , . . . , Bk .
6.2.12. Приведите пример подмножеств A, B1 , B2 конечного множества:
(a) попарно независимых, но для которых A не является независимым от набора B1 , B2 ;
(b) не являющихся попарно независимыми, но для которых A
независимо от набора B1 , B2 .
6.2.13. Обозначим через M семейство всех раскрасок множества
{1, 2, . . . , 400} в два цвета. Для подмножества α ⊂ {1, 2, . . . , 400}
обозначим через Aα ⊂ M подмножество тех раскрасок, для которых
α одноцветно. Тогда A{1,2,...,8} не зависит от набора {Aα : α ⊂
{9, 10, . . . , 400}}. (Ср. с замечанием после задачи 6.2.1.b.)
6.2.14. Следующие условия на подмножества A, B1 , . . . , Bk равносильны:
• A независимо от набора B1 , . . . , Bk
• A независимо от набора B1 , . . . , Bk .
• A независимо от набора B 1 , . . . , B k .
6.2.15. (a) Локальная лемма Ловаса в симметричной форме.
Пусть A1 , . . . , An — подмножества конечного множества. Если
для некоторого d и любого k доля подмножества Ak не меньше
1
и существует набор из не менее чем n − d подмножеств
1−
4d
6. АНАЛИТИЧЕСКИЕ И ВЕРОЯТНОСТНЫЕ МЕТОДЫ
163
Aj , от которого Ak не зависит, то A1 ∩ · · · ∩ An ̸= ∅. 7
(b) При d > 2 утверждение пункта (b) верно, если заменить
1
1
1−
на 1 −
, где e — основание натуральных логарифмов.
4d
e(d + 1)
(c) Если ak = 1 при любом k 6 0 и для некоторого d > 2 выak−d
полнено ak+1 > ak −
при любом k > 0, то ak > 0 при любом
4d
k.
Читатель может перед доказательством этой леммы применить
ее к решению задачи 6.2.1.b. Доказательство леммы нетривиально
обобщает идеи решения задач 6.2.10 и 6.2.11. Из этих задач ясно,
что нужно оценивать снизу количество элементов в пересечении s
из данных множеств, начиная с s = 1 и заканчивая s = n, при
помощи индукции по s. Как часто бывает, наиболее трудная часть
— догадаться, какое конкретно утверждение нужно доказывать по
индукции (а также, по каким параметрам вести индукцию). Вот
оно:
(
)
1 t
|A1 ∩ · · · ∩ Ak+t | > |A1 ∩ · · · ∩ Ak | 1 −
для любых k, t > 0.
d
Указания и решения к задачам 6.2.1-6.2.15 (кроме 6.2.2.b,
6.2.3.b и 6.2.4.b)
6.2.1. (a) См. указание к задаче 1.6.7.
(b) Обозначим через A множество распределений выходных. Для
каждого вида работ x обозначим через x
b множество специалистов
по нему, а через Ax — множество распределений выходных, при которых и в субботу, и в воскресенье на работе есть специалист по
x. Тогда |Ax |/|A| = 2−7 . Подмножество Ax не зависит от набора
{Ay | yb ∩ x
b = ∅}. Так как каждый вид работ имеет общих специалистов не более чем с 30 другими видами, то вне этого набора не
7
Вот формулировка на вероятностном языке, которая не используется в
дальнейшем. Пусть дано вероятностное пространство и A1 , . . . , An — события. Пусть для некоторого d и любого k вероятность события Ak не меньше
1
1 − 4d
и существует набор из не менее чем n − d событий Aj , от которого
Ak не зависит. Тогда вероятность события A1 ∩ . . . ∩ An положительна.
168
Элементы дискретной математики в задачах
тов чисел x, x + M1 , . . . , x + M25 не все цвета присутствуют. Тогда
|Ax |/|A| 6 3(2/3)26 . Каждое множество Ax ‘зависимо не более чем
с 25 · 26 = 650 другими’ (т.е. независимо от набора всех множеств,
кроме некоторых 650). Применим локальную лемму Ловаса в симметричной форме (задачу 6.2.15.b) к дополнениям множеств Ax и
d = 650. Это возможно ввиду утверждения задачи 6.2.9 и неравенства (3/2)26 > 213 > 8000 > 7800 = 3 · 2600 = 3 · 4 · 650. Получим
∩x Ax ̸= ∅.
Задачи для самостоятельного решения
6.2.16. Дано число
(a) k > 10; (b) k = 9
и семейство k-элементных подмножеств конечного множества
M . Если каждый элемент множества M содержится ровно в k подмножествах семейства, то существует раскраска множества M в
два цвета, для которой каждое подмножество семейства содержит
элементы обоих цветов.
(Т.е. хроматическое число любого k-однородного k-регулярного
гиперграфа равно двум при k > 9. Ср. с задачей 6.2.1.b.)
6.2.17. В конечном множестве выбрано несколько подмножеств. В
каждом из них не менее 3 элементов. Каждое из них пересекается не
более
чем с ai выбранными i-элементными подмножествами. Если
∑
−i 6 1/8, то можно покрасить элементы данного множества
i ai 2
в два цвета так, чтобы каждое выбранное подмножество содержало
элементы обоих цветов.
6.2.18. (a) Для любого разбиения множества вершин цикла длины
11n на n множеств по 11 вершин можно выбрать по вершине из
каждого множества так, что между выбранными n вершинами нет
ребер.
(b) В графе степень каждой вершины не превосходит ∆. Все
вершины раскрашены в r цветов. Вершин каждого цвета не менее
2e∆ + 1. Тогда можно выбрать r вершин разных цветов, никакие
две из которых не соединены ребром.
6. АНАЛИТИЧЕСКИЕ И ВЕРОЯТНОСТНЫЕ МЕТОДЫ
169
6.2.19. (a) Каждую k-элементную арифметическую прогрессию в
n
] других таких
множестве {1, 2, . . . , n} пересекает не более k 2 [
k−1
прогрессий.
(b) Для любого натурального k существует раскраска первых
k−3
[2 (k − 1)/k 2 ] натуральных чисел в 2 цвета, для которой нет одноцветной k-элементной арифметической прогрессии.
(c) Каждую k-элементную арифметическую прогрессию в множестве {1, 2, . . . , n} пересекает не более nk других таких прогрессий.
(d) Для любого натурального k существует раскраска первых
k−3
[2 /k] натуральных чисел в 2 цвета, для которой нет одноцветной
k-элементной арифметической прогрессии.
6.2.20. (a) Если X ⊂ R — конечное множество
(
)mи m, r — натураль1
ные числа, для которых 4rm(m − 1) 1 −
< 1, то для люr
бого m-элементного подмножества M ⊂ R существует раскраска
множества R в r цветов такая, что для любого x ∈ X множество
x + M := {x + a : a ∈ M } содержит точки каждого из r цветов.
(b) То же для X = Z.
(c) То же для X = R.
( )(
)
n
n
r
6.2.21. (a) Если
+ 1 < 2( 2 )−1 /e, то R(n, n) > r.
2
√ 2 −1 n −
(b) R(n, n) & 2e n2n/2 . (Ср. с задачей 4.1.5.)
6.2.22. Имеется несколько цветов. Каждой вершине некоторого графа сопоставлен список из не менее чем 10d этих цветов, где d > 1.
Для любых вершины v и цвета из ее списка имеется не более d соседей вершины v, в списке которых есть этот цвет. Тогда можно так
раскрасить каждую вершину графа в некоторый цвет из ее списка,
чтобы концы любого ребра были разного цвета.
6.2.23. В ориентированном графе в каждую вершину входит не
больше ∆ ребер и из каждой вершины выходит не меньше δ ребер.
1
Тогда для любого натурального k 6
в графе найдется
1 − (4δ∆)−1/δ
ориентированный цикл длины, кратной k.
170
Элементы дискретной математики в задачах
6.2.24. Клетки доски n×n раскрашены в несколько цветов. Клеток
каждого цвета не больше чем (n − 1)/16. Тогда можно поставить на
доску n попарно не бьющих друг друга ладей, чтобы они стояли на
клетках разных цветов.
6.2.25. КНФ-формула или формула в конъюнктивной нормальной
форме — конъюнкция набора дизъюнкций нескольких из переменных x1 , . . . , xn и их отрицаний. Если в каждом ‘сомножителе’ КНФформулы ровно k ‘слагаемых’ и у каждого ‘сомножителя’ есть общие переменные не более чем с 2k−2 другими, то булева функция,
определяемая формулой, не является тождественным нулем.
Замечание. Одной из центральных в информатике является проблема k-выполнимости (k-SAT problem): для даннной КНФ-формулы,
в каждой дизъюнкции которой ровно k переменных, установить, является ли задаваемая ею функция тождественным нулем. При k = 2
есть полиномиальный алгоритм её решения. При бо́льших k быстрых алгоритмов, отвечающих на этот вопрос, неизвестно. Построение такого быстрого алгоритма, либо доказательство его несуществования, эквивалентно решению знаменитой открытой проблемы
«P̸=NP».
6.2.26. (a) Локальная лемма Ловаса. Пусть A1 , . . . , An — подмножества конечного множества, J1 , . . . , Jn ⊂ {1, . . . , n} и γ1 , . . . , γn ∈
(0, 1). Пусть для любого k
∏
• доля подмножества Ak не меньше 1 − (1 − γk ) j̸∈Jk γj ;
• множество Ak не зависит
∩n от набора {Aj∏:n j ∈ Jk }. 9
Тогда доля пересечения k=1 Ak не меньше k=1 γk > 0.
√
(b) Существует такое c > 0, что R(3, n) > cn n для любого n.
Замечание. При помощи более сложных вычислений из локальной леммы Ловаса выводится, что R(3, n) > c1 n2 / ln2 n. Более того,
это «лучшее», что можно выжать из локальной леммы Ловаса. Известно также неравенство R(3, n) > c2 n2 / ln n (теорема Кима). Его
9
Вот формулировка на вероятностном языке. Пусть дано вероятностное
пространство, A1 , . . . , An — события, J1 , . . . , Jn ⊂ {1, . . . , n} и γ1 , . . . , γn ∈
(0, 1).
∏ Пусть для любого k вероятность события Ak не меньше 1 − (1 −
γk ) j̸∈Jk γj и событие Ak не зависит от набора {Aj , j ∈ Jk }. Тогда вероят∏
ность события A1 ∩ . . . ∩ An не меньше m
j=1 γj .
6. АНАЛИТИЧЕСКИЕ И ВЕРОЯТНОСТНЫЕ МЕТОДЫ
171
доказательство вместо локальной леммы Ловаса использует квазислучайные графы, неравенства плотной концентрации и пр.
6.3
Случайные графы
Начнем с интересных задач, которые можно решить при помощи
случайных графов. (Более простые решения без случайных графов
неизвестны. Известны такие же или более сложные, и то не для
всех задач.)
6.3.1. Если в графе G = (V, E) с n вершинами минимальная степень вершины равна δ, то
(a) Для любого p ∈ (0, 1) существует такое множество вершин A ⊂
V , что в объединении A и множества всех вершин, не соединённых
ни с какой вершиной из A, имеется не более np+n(1−p)δ+1 вершин.
(b) Существует такое множество вершин D ⊂ V , что любая вершина из V \D соединена ребром с некоторой вершиной из D, и
1 + ln(δ + 1)
|D| 6 n
.
δ+1
Для решения следующих задач 6.3.2 и 6.3.3.c нужна приведенная ниже теория. К их решению разумно вернуться после задачи
6.3.9.
( )
( )
n
k (m)
k
6.3.2. (a) Eсли
p 2 +
(1 − p)( 2 ) < 1 для некоторого p ∈
m
n
(0, 1), то R(m, n)(> k. )
n2
(b)* R(4, n) > Ω
(мы пишем g > Ω(f ), если f = O(g)).
ln2 n
6.3.3. (a) Cherchez la femme. На русско-французской встрече не
было представителей других стран. Суммарное количество денег у
французов оказалось больше суммарного количества денег у русских, и суммарное количество денег у женщин оказалось больше
суммарного количества денег у мужчин. Обязательно ли на встрече была француженка?
(b) Денежные купюры разного достоинства и разных стран упакованы в два чемодана. Средняя стоимость купюры равна 100 рублей.
172
Элементы дискретной математики в задачах
Общее число купюр в левом чемодане больше, чем в правом. Обязательно ли в левом чемодане найдется купюра стоимостью не более
200 рублей? (Ср. с неравенством Маркова 6.3.9.a.)
(c) Для любых целых l, q > 0 существует граф, не содержащий обходов длины менее l и который невозможно правильно раскрасить
в q цветов. (См. определение правильности раскраски в §3.1.)
Зафиксируем p ∈ (0, 1) и назовем вероятностью графа (в модели, или в вероятностном пространстве, Эрдеша-Реньи) c n вершинаn(n−1)
ми {1, 2, . . . , n} и e рёбрами число P(G) = Pp (G) := pe (1−p) 2 −e .
Вероятностью семейства (или, что то же самое, свойства) графов
с вершинами 1, 2, . . . , n называется сумма вероятностей входящих в
него графов.
Случайной величиной называется функция, определённая на множестве графов c вершинами 1, 2, . . . , n.
Например, количество рёбер графа — случайная величина.
Пусть случайная величина Y принимает k различных значений y1 , . . . , yk . Тогда математическим ожиданием (мат. ожиданием) случайной величины Y называется её «взвешенное среднее»
k
∑
ys P(Y −1 (ys )), где Y −1 (ys ) – множество всех графов G, для
EY :=
s=1
которых Y (G) = ys . Последнюю вероятность обозначают P(Y = ys ).
6.3.4. Для данных n и p вероятность наличия k вершин, между
которыми нет рёбер, меньше ek ln n−pk(k−1)/2 .
6.3.5. Для данных n и p найдите мат. ожидание количества
(a) изолированных вершин;
(b) треугольников;
(c) k-клик;
(d) k-клик, являющихся компонентами связности;
(e) гамильтоновых циклов;
(f) обходов длины k;
(g) обходов длины k, являющихся компонентами связности;
(h) деревьев с k вершинами;
(i) древесных компонент данного размера k, т.е. деревьев с k вершинами, являющихся компонентами связности.
6. АНАЛИТИЧЕСКИЕ И ВЕРОЯТНОСТНЫЕ МЕТОДЫ
173
6.3.6. Для данного p найдите асимптотику (при постоянном k и
n → ∞) функции E(k) (Y ) := E (Y (Y − 1) . . . (Y − k + 1)) (т. е. k-го
факториального момента), если Y — число изолированных вершин.
Дисперсией случайной величины X называется число DX :=
E(X − EX)2 .
6.3.7. Для данных n и p найдите дисперсию количества (a) изолированных вершин; (b) треугольников.
6.3.8. (a) E(ξ + η) = Eξ + Eη;
(b) D(ξ + η) = Dξ + Dη, если ξ и η независимы.
6.3.9. Пусть X — случайная величина (определенная выше) и a >
0.
(a) Неравенство Маркова. P(|X| > a) 6 E|X|/a. (Ср. с задачей
6.3.3.a.)
(b) Неравенство Чебышева. P(|X − EX| > a) 6 DX/a2 .
Событие An происходит асимптотически почти наверное (или
с асимптотической вероятностью 1) относительно последовательности f (n), если Pf (n) (An ) → 1. Общепринятое сокращение: при
p(n) = f (n) событие An происходит а.п.н. (формально, эта фраза
не имеет смысла, поскольку означает «если p(n) = f (n), то событие An происходит а.п.н.», а без указания последовательности f (n)
фраза «событие An происходит а.п.н.» не может быть определена
как надо). Напомним, что здесь n — число вершин графа.
6.3.10. При p(n) = 1/(2n)
(a) а.п.н. имеется более n/2 изолированных вершин.
(b) для некоторого C > 0 а.п.н. каждая компонента связности
имеет менее C ln n вершин (специалисты говорят: менее O(ln n) вершин).
(c)* а.п.н. каждая компонента связности является деревом или унициклическим графом.
(d)* для некоторого C > 0 а.п.н. имеется менее C унициклических
компонент.
174
Элементы дискретной математики в задачах
(
)
6.3.11. (a) При p(n) = o n−3/2 а.п.н. рёбра попарно не пересекаются.
(
)
(b) При p = p(n) = o n−3/2 и pn2 → ∞ существует такая функция r = r(n) = o(pn2 ), что а.п.н. число вершин степени 1 больше
pn2 − r и меньше pn2 + r, и остальные степени равны нулю.
6.3.12. Теорема о связности случайного графа. Если c > 1 (0 < c <
1), то при p(n) = c ln n/n а.п.н. случайный граф связен (несвязен).
6.3.13. (a) Найдите хотя бы одну такую функцию p∗ (n), что
• при p(n)/p∗ (n) → 0 а.п.н. граф не содержит треугольника, и
• при p(n)/p∗ (n) → +∞ а.п.н. граф содержит треугольник.
(b) То же с заменой треугольника на подграф, изоморфный K4 .
Замечание. Такая функция p∗ называется пороговой вероятностью. Пороговая вероятность существует для любого монотонного семейства. Монотонно возрастающим (убывающим) семейством
графов называется такое семейство графов, которое вместе с каждым графом содержит любой его надграф (подграф).
6.3.14. Хроматическое число графа а.п.н. не больше
(a) одного при p(n) = o(1/n2 );
(b) двух при p(n) = o(1/n);
(c) трёх при p(n) = c/n, где c < 1.
6.3.15. *
(a) Жадный алгоритм раскраски (см. задачу 3.2.3) для любого
положительного ε а.п.н. ошибается не более чем в 2 + ε раз.
(b) Для любых ε, δ > 0 существует такая последовательность Gn
графов с n вершинами, что при случайной нумерации вершин графа Gn вероятность того, что отношение числа цветов в жадной раскраске к χ(Gn ) больше n1−ϵ , больше δ. (Иными словами, с одной
стороны, почти для любого графа в любой нумерация жадная раскраска хороша, но, с другой стороны, есть графы, которые почти
как ни нумеруй, а все дрянь получится!)
(Приведите аккуратные формулировки самостоятельно.)
Замечание. См. подробнее [R3, R4, R5]. В частности, в [R4] доказаны следующие результаты.
6. АНАЛИТИЧЕСКИЕ И ВЕРОЯТНОСТНЫЕ МЕТОДЫ
175
fn =
( Первая)теорема Боллобаша. Существует последовательность
n
n <
o
, для которой при p(n) = 1/2 а.п.н. χ(G) −
2 log2 n
2 log2 n fn .
(Эта теорема обобщается на практически любые значения p [Ja].)
Вторая теорема Боллобаша. Для любого α > 2/3 существуют
последовательности an и bn , для которых при p(n) = n−α а.п.н.
χ(G) ∈ {an , bn }.
(В этой теореме для некоторых α последовательности an и bn
"могут быть выбраны а "устроены"
могут быть выбраны так, что limn→∞ an = limn→∞ bn = ∞.)
Замечание. Приведем результат [Bo, стр. 100, теорема 5.4]. Обозначим pn := p(n). Для k > 2 обозначим через Tk = Tk,pn число
компонент связности в случайном графе, являющихся деревьями с
k вершинами.
(a) Если pn = o(n−k/(k−1) ), то а.п.н. Tk = 0.
(b) Если limn→∞ pn nk/(k−1) = c > 0, то последовательность случайных величин Tk = Tk,pn сходится при n → ∞ к случайной
величине, имеющей распределение Пуассона с параметром λ :=
s
ck−1 k k−2 /k!, т.е. limn→∞ P(Tk = s) = λs! e−λ для любого s ∈ Z, s > 0.
(c) Если nk/(k−1) = o(pn ) и limn→∞ (pn kn − ln n − (k − 1) ln ln n) =
−∞, то limn→∞ P(Tk > L) = 1 для любого L > 0.
(d) Если limn→∞ (pn kn−ln n−(k −1) ln ln n) = x ∈ R, то последовательность случайных величин Tk = Tk,pn сходится при n → ∞ к
случайной величине, имеющей распределение Пуассона с параметром λ := e−x /(k · k!).
(e) Если limn→∞ (pn kn − ln n − (k − 1) ln ln n) = +∞, то а.п.н.
Tk = 0.
6.4
Случайные графы. А.М. Райгородский
Зафиксируем целое положительное число n, а также p ∈ [0, 1]
и q = 1 − p. Вероятностью графа G = (V, E) назовем P (G) :=
2
p|E| q Cn −|E| . Вероятностью произвольного семейства (или, что то
176
Элементы дискретной математики в задачах
же самое, свойства10 ) графов с множеством вершин V называется
сумма вероятностей входящих в него графов.
(Говоря научно, ребра графа выбираются с помощью ”схемы
испытаний Бернулли”, т.е. каждое ребро независимо от остальных
имеется в графе с вероятностью p ∈ [0, 1]; соответственно, с вероятностью q = 1−p его нет в графе. В результате возникает случайный
граф, множество вершин которого фиксировано, а множество ребер
случайно. Всякое событие строится из элементарных, как из кирпичиков, и считается произошедшим, коль скоро случайный граф
ему — как множеству — принадлежит. Имея понятие случайного
графа, можно судить о том, с какой степенью достоверности граф
обладает тем или иным свойством. Например, можно определить
вероятность связности графа или вероятность того, что граф имеет какое-то хроматическое число, и т.д.)
Если Ω — множество всех графов с n вершинами, то P (Ω) = 1.
При вышеприведенном формальном определении это теорема. Вот
2
Cn
∑
2
2
CCk 2 pk (1−p)Cn −k = (p+(1−p))Cn = 1.
ее доказательство: P (Ω) =
k=0
n
Случайная величина — функция, определенная на множестве
графов. Например, количество ребер графа. Если случайная величина принимает k различных значений y1 , . . . , yk , то математическим ожиданием величины X называется ее ”взвешенное среднее”
k
∑
MX =
yi P (X −1 (yi )), где X −1 (yi ) – семейство таких графов G,
i=1
для которых X(G) = yi . Для краткости можно последнюю вероятность можно обозначать P (X = yi ).
1. (a) Докажите линейность математического ожидания: M (c1 X1 +
· · · + cs Xs ) = c1 M X1 + · · · + cs M Xs для любых случайных величин
X1 , . . . , Xs и констант c1 , . . . , cs ∈ R.
(b) Докажите, что если X принимает неотрицательные целые
значения, то P (X = 0) > 1 − M X.
2. Найдите M X для графов G с n вершинами и случайной величины X, равной
10
Сказать, что для графа выполнено некоторое свойство, это все равно, что
отнести граф к множеству графов, указанным свойством обладающих.
6. АНАЛИТИЧЕСКИЕ И ВЕРОЯТНОСТНЫЕ МЕТОДЫ
177
(a) числу треугольников в G;
(b) числу циклов длины k в G;
(c) числу полных подграфов на k вершинах (k - клик) в G;
(d) числу различных k - вершинных деревьев в G;
(e) числу изолированных k - вершинных деревьев в G;
(f) числу вершин на древесных компонентах G;
(g) числу вершин на циклических компонентах G.
Пусть f и g - некоторые функции натурального аргумента n,
причем g(n) ̸= 0 для всех n. Скажем, что f = o(g) (читается ”f - о
√
(n)
малое от g”), если fg(n)
→ 0 при n → ∞. Например 1 = o( n) или
(1)
1
=
o
2
n .
n
3. Пусть Pa и Pb – многочлены степеней a и b соответственно,
a < b. Докажите, что Pa = o(Pb ).
Полезно бывает изучать различные вероятности при n → ∞.
Пусть теперь n меняется и величина pn (вероятность появления ребра случайного графа) зависит от n. (В этом есть глубокий смысл,
который мы ниже пронаблюдаем.) Свойство B называется исключительно достоверным, если P (Bn ) → 1, при n → ∞, где Bn –
семейство графов с n вершинами, удовлетворяющих свойству B.
Говорят даже, что в таком случае это свойство выполнено почти
наверное.
( )
4. Докажите, что если pn = o n1 , то в случайном графе почти
наверное нет треугольников.
(
)
k
− k−1
5. Пусть k > 2 - константа. Докажите, что если pn = o n
,
то в случайном графе почти наверное нет компонент связности,
являющихся деревьями на k вершинах.
( )
6. Докажите, что если pn = o n1 , то случайный граф почти
наверное является лесом.
( )
7. Докажите, что если pn = o n1 , то случайный граф почти
наверное двудолен.
8. Определим число независимости α(G) графа G как размер
максимального подмножества вершин в G, которые попарно не со-
178
Элементы дискретной математики в задачах
единены ребрами. Пусть pn = 12 . Докажите, что P (α(G) < 2 log2 n + 10 log2 log2 n)
1 при n → ∞.
Хроматическое число χ(G) графа G – минимальное количество
цветов, в которое можно раскрасить вершины графа G правильным
образом. (Раскраска вершин графа в несколько цветов называется
правильной, если любые две вершины, соединенные ребром, окрашены в разные цвета.)
9. Докажите, что если pn = 21 , то хроматическое(число)случай-
ного графа почти наверное больше, чем 2 logn n + o logn n . Более
2)
2
(
строго: для некоторой функции fn = o logn n почти наверно вы2
n
полнено свойство χ(G) > 2 log n + fn (зависящее от n).
2
10. Докажите, что если pn >
граф связен.
3ln n
n ,
то почти наверное случайный
11∗ . Пусть pn = n−α , где α > 65 . Докажите, что
)
(
√
P ∀ S ⊂ V, |S| 6 n : χ(G|S ) 6 3 → 1, n → ∞.
Здесь G|S - порожденный или индуцированный подграф графа G =
(V, E), т.е. граф H = (S, F ), у которого e = (x, y) ∈ F тогда и только
тогда, когда x, y ∈ S и e ∈ E. Иными словами, H получается из
G удалением всех вершин, не принадлежащих S, вместе со всеми
выходящими из них ребрами.
12∗ . Граф расстояний на плоскости – граф, вершины которого
являются точками плоскости, и ребрами соединены пары вершин,
( )
находящиеся на расстоянии 1. Докажите, что если pn = o n1 , то
случайный граф почти наверное может быть реализован как граф
расстояний на плоскости, а если pn > 1000
n , то почти наверное выполнено обратное свойство.
6.5
Подсказки
6.1.1 . (d,e) См. задачу 1.1.3.
6.1.2 . См. задачу 2.7.1.
6. АНАЛИТИЧЕСКИЕ И ВЕРОЯТНОСТНЫЕ МЕТОДЫ
179
6.2.19 . (c) Оцените количество пересекающих прогрессий, содержащих данный элемент x ∈ {1, 2, . . . , n}, и просуммируйте по всем
x.
6.2.20 . (a) Обозначим через A семейство раскрасок множества M +
X в r цветов. Для любого x ∈ X обозначим через Ax семейство раскрасок множества M + X в r цветов, для которых множество x + M
содержит не все цвета.
(b) Используйте компактность.
6.2.21 . (a) Для любого S ⊂ {1, 2, . . . , r} обозначим через AS множество графов с r вершинами, для которых индуцированный на S
подграф пуст.
6.2.22 . Сначала надо уменьшить множество цветов, чтобы каждой
вершине было поставлено в соответствие ровно 10d цветов. Покрасим множество вершин случайно равновероятно в те цвета, которые
им сопоставлены.
6.2.25 . Примените локальную лемму Ловаса в симметричной форме (задачу 6.2.15.b).
6.3.1 . (a) Возьмём произвольное 0 < p < 1. Неформально говоря,
будем считать, что P (v ∈ A) = p для каждой вершины v и подмножества A ⊂ V . Формально, поставим в соответствие каждому
подмножеству A ⊂ V число p|A| (1 − p)n−|A| . Иными словами, рассмотрим вероятностное пространство всех подмножеств множества
V с вероятностью подмножества A, равной p|A| (1−p)n−|A| . (Это возможно, поскольку сумма всех 2n таких вероятностей
равна 1.) Тогда
∑
|A|
для каждой вершины v имеем P (v ∈ A) = A∋v p (1 − p)n−|A| = p.
Определим случайную
{ величину χA (индикатор события v ∈ A)
1, v ∈ A;
как χA (v) = 1v∈A :=
0, v ̸∈ A.
6.3.2 . (a) Рассмотрите модель G(k, p).
180
Элементы дискретной математики в задачах
6.3.3 . (c) Набор вершин называется независимым, если между ними нет рёбер. По задаче 3.1.3 достаточно для некоторого достаточно
большого n построить граф, в котором длина каждого несамопересекающегося цикла больше l и любые k = k(n) вершин зависимы,
m
для некоторой функции k такой, что limm→∞
= ∞. Такой
k(m) − 1
граф легко получить из графа с n вершинами, в котором любые
k = k(n) вершин зависимы и в котором меньше n/2 несамопересекающихся циклов длины менее l.
Назовем французским граф, в котором меньше n/2 несамопересекающихся циклов длины менее l. Назовем женским граф, любые
k = k(n) вершин в котором зависимы. Придумайте, как раздать
графам деньги, чтобы существование француженки вытекало из
пункта (a). Иными словами, как поставить графам в соответствие
веса, при помощи которых можно будет доказать существование
графа с обоими свойствами.
6.3.6 . k-й факториальный момент для числа того-то — это мат.
ожидание числа упорядоченных последовательностей длины k из
того-то, в которых все элементы различны.
6.3.10 . (a) Посчитайте мат. ожидание и дисперсию количества изолированных вершин.
(b) Найдите мат. ожидание количества связных компонент размера более C ln n.
(c) Введите случайные величины аналогично решению пункта (b)
и посчитайте их мат. ожидания, используя (b).
(d) То же самое.
6.3.11 . (a) Рёбра попарно не пересекаются тогда и только тогда,
когда степень каждой вершины равна 0 или 1.
(b) Число вершин степени 1 равно половине количества ребер.
Оцените мат. ожидание и дисперсию количества ребер.
6.3.13 . Обозначим через ξn количество подграфов, изоморфных данному. Оцените Eξn и Dξn . Подберите функцию p∗ (n), для которой
Eξn → 0 при
p(n)
→ 0 и Dξn = o(E2 ξn ) при
∗
p (n)
p(n)
→ +∞.
p∗ (n)
192
Элементы дискретной математики в задачах
7
Алгебраические методы
7.1
Линейно-алгебраический метод в комбинаторике
Напомним, что для множества F
F n := {(a1 , a2 , . . . , an ) : a1 , a2 , . . . , an ∈ F }.
Элементы этого множества называются векторами (или наборами
или точками). Если F ∈ {Z2 , Z, Q, R}, то векторы можно покомпонентно складывать:
(x1 , . . . , xn ) + (y1 , . . . , yn ) := (x1 + y1 , . . . , xn + yn ).
Если F ∈ {Z, Q, R}, то вектор можно покомпонентно умножить на
число λ ∈ F :
λ(x1 , . . . , xn ) := (λx1 , . . . , λxn ).
(Это можно делать и для F = Z2 , но не интересно.)
7.1.1. Теорема о линейной зависимости.
(Z2 ) Среди любых n + 1 наборов длины n из нулей и единиц
найдется несколько (не ноль) наборов, покомпонентная сумма по
модулю два которых есть нулевой набор.
(Q) Для любых n + 1 векторов v1 , . . . , vn+1 ∈ Qn найдутся рациональные числа λ1 , . . . , λn+1 , не все равные нулю, для которых
λ1 v1 + . . . + λn+1 vn+1 = (0, . . . , 0).
(R) Аналог теоремы (Q) справедлив для вещественных, комплексных и целых чисел.
Наборы из задач 7.1.1.(Z2 ),(Q) называются линейно зависимыми — над Z2 и над Q соответственно. Линейная независимость
— отрицание линейной зависимости. Аналогично определяется линейная (не)зависимость многочленов над Z2 и Q соответственно.
(Эти и следующие понятия используются в формулировках задач
7.1.4.c, 7.1.5.c, 7.1.7.b и в решениях некоторых задач.)
Для F ∈ {Z2 , Z, Q, R} скалярное произведение F n × F n → F
определяется формулой
(x1 , . . . , xn ) · (y1 , . . . , yn ) := x1 y1 + . . . + xn yn .
7. АЛГЕБРАИЧЕСКИЕ МЕТОДЫ
193
Векторы x, y ∈ F n называются ортогональными, если x · y = 0.
Расстояние между точками пространства Rn определяется формулой
√
|(x1 , . . . , xn ), (y1 , . . . , yn )| := (x1 − y1 )2 + . . . + (xn − yn )2 .
Линейным подпространством называется подмножество L ⊂
замкнутое относительно сложения векторов и умножения на
рациональные числа. Линейное подпространство L называется nмерным, если найдутся такие линейно независимые векторы
v1 , . . . , vn ∈ L, что любой вектор v ∈ L линейно выражается через
данные векторы, т. е. найдутся числа λ1 , . . . , λn ∈ Q, для которых
v = λ1 v1 +. . .+λn vn . Число n называют размерностью пространства
L. Ср. с определением перед задачей 1.4.7.
Замечание. Аналогичные определения можно дать и в более общей ситуации — это приводит к понятию кольца и модуля над ним.
Попытка доказать (и использовать!) аналог теоремы о линейной
зависимости (задачи 7.1.1.(Z2 ),(Q)) приводит к понятиям поля и
линейного пространства над ним. (Для случая целых чисел уже
не все обобщения проходят.) Подробности можно найти в учебнике
по линейной алгебре.
Qn ,
7.1.2. Дано семейство F подмножеств множества Rn .
(a) Если в каждом подмножестве из F нечётное число элементов,
а в пересечении любых двух подмножеств из F чётное число элементов, то |F| 6 n.
(b) Постройте пример, когда эта оценка достигается.
(c) Если в пересечении любых двух подмножеств из F ровно q
элементов и в каждом подмножестве из F более q элементов, то
|F| 6 n.
(d) Если q > 0 и в пересечении любых двух подмножеств из F
ровно q элементов, то |F| 6 n.
7.1.3. (a) Существуют 2k подмножеств 2k-элементного множества,
в каждом из которых чётное число элементов и в пересечении любых двух из которых чётное число элементов.
(b) Больше, чем 2k подмножеств, в условиях пункта (a) быть не
может.
194
Элементы дискретной математики в задачах
7.1.4. (a) Наибольшее число точек в Rn с равными попарными
расстояниями равно n + 1.
(b) Постройте n(n − 1)/2 точек в Rn , попарные расстояния между
которыми принимают только два различных значения.
(c) Для a ∈ R и точек v = (v1 , . . . , vn ) ∈ Rn , x = (x1 , . . . , xn ) ∈ Rn
обозначим Pv (x) := |x − v|2 − a2 . Если попарные расстояния между k точками u1 , . . . , uk ∈ Rn равны a, то многочлены Pu1 , . . . , Puk
линейно независимы над Q.
(d) Если попарные расстояния между k точками в Rn принимают
только два различных значения, то k 6 (n + 1)(n + 4)/2.
7.1.5. (a) Среди любых 327 попарно пересекающихся 9-элементных
подмножеств 25-элементного множества найдутся два подмножества, в пересечении которых ровно 3 или ровно 6 элементов.
(b) Для n, k ∈ Z обозначим
∑
n
Vn,k := {(x1 , . . . , xn ) ∈ {0, 1} :
xs = k}.
s
Среди любых 327 точек в V25,9 есть две, скалярное произведение
которых лежит в {0, 3, 6}.
(c) Для любого ⃗a ∈ V25,9 раскроем скобки в произведении
(⃗a · (x1 , x2 , . . . , x25 ) − 1)(⃗a · (x1 , x2 , . . . , x25 ) − 2),
где x1 , x2 , . . . , x25 — переменные. С каждым из полученных одночленов проведём следующую операцию: для каждого i если в одночлене есть множитель xni , то заменим этот множитель на xi . Полученный многочлен обозначим F⃗a (x1 , . . . , x25 ). Докажите, что если
скалярное произведение никаких двух векторов среди ⃗a1 , . . . , ⃗as ∈
V25,9 не делится на 3, то многочлены F⃗a1 , . . . , F⃗as линейно независимы над Q.
(d) Укажите 326 многочленов, линейными комбинациями которых
с рациональными коэффициентами можно получить каждый многочлен F⃗a , ⃗a ∈ V25,9 .
7.1.6. (a) Среди любых 107 пятиэлементных подмножеств 14-элементного множества найдутся два подмножества, в пересечении которых ровно 2 элемента.
7. АЛГЕБРАИЧЕСКИЕ МЕТОДЫ
195
(b) То же для 93 подмножеств.
(c) То же для 92 подмножеств.
(d) Невозможно раскрасить в 21 цвет все пятиэлементные подмножества 14-элементного множества так, чтобы любые два пятиэлементные подмножества, пересекающиеся ровно по двум элементам,
были разноцветны.
(Ср. с замечанием в задаче 5.1.4. Вот эквивалентная формулировка. Вершинами графа являются все пятиэлементные подмножества
14-элементного множества. Его ребрами являются пары подмножеств, пересекающиеся ровно по двум элементам. Докажите, что
этот граф нельзя правильно раскрасить в 21 цвет.)
7.1.7. (a) Для простого p и целого t число G(t) := (t − 1)(t −
2) . . . (t−p+1) делится на p тогда и только тогда, когда t не делится
на p.
(b) Пусть p простое и n = 4p. Обозначим
M = {(1, y2 , y3 . . . , yn ) |
| yk ∈ {1, −1} и среди y2 , . . . , yn число минус единиц чётно}.
Обозначим G(t) := (t − 1)(t − 2) . . . (t − p + 1). Для любого ⃗a ∈ M
раскроем скобки в произведении G(⃗a · (1, x2 , . . . , xn )), где x2 , . . . , xn
— переменные. В каждом из полученных одночленов для каждого i
будем заменять x2i на 1, пока это возможно. Полученный многочлен
обозначим F⃗a (x2 , . . . , xn ).
Докажите, что если скалярное произведение никаких векторов среди ⃗a1 , . . . , ⃗as ∈ M не равно нулю, то многочлены F⃗a1 , . . . , F⃗as линейно независимы над Q.
(c) Существуют n и ограниченное подмножество в Rn , которое
невозможно разбить на n + 1 непустых частей меньшего диаметра.
7.1.8. (a) Теорема Франкла-Уилсона. Если t простое, то cреди люt−1 ( )
∑
n
бых различных 1+
подмножеств n-элементного множества,
j
j=0
в каждом из которых k-элементов, найдутся два подмножества, число элементов в пересечении которых делится на t.
(b) То же, только задано целое q и ‘делится на t’ заменено на
‘сравнимо с q по модулю t’.
196
Элементы дискретной математики в задачах
7.1.9. (a) Если множество рёбер графа Kn является объединением множеств рёбер s полных двудольных графов, не пересекающихся по рёбрам, то s > n − 1.
(b) Постройте набор двудольных графов, на котором эта оценка
достигается.
Более подробное и развернутое изложение можно найти в [M,
R1], а более продвинутое — в [B].
7.2
Матрицы Адамара
7.2.1. Теорема Адамара. Если у матрицы A размера n × n все элементы по модулю меньше 1, то | det A| 6 nn/2 .
Квадратная матрица H называется матрицей Адамара, если все
её элементы равны ±1 и H · H T = nEn , где n — порядок матрицы
H и En — единичная матрица.
7.2.2. Постройте матрицу Адамара n × n для n =
(2) 2;
(4) 4;
(8) 8;
(16) 16;
(12) 12.
7.2.3. (a) У матрицы Адамара любые два столбца ортогональны.
(b) Матрица является матрицей Адамара тогда и только тогда,
когда её элементы равны ±1 и любые две строки ортогональны.
(c) Для матриц Адамара достигается верхняя оценка в теореме
Адамара. (Название матрицы Адамара получили благодаря этому
результату.)
(d) Если существует матрица Адамара n × n и n > 2, то n делится
на 4.
Гипотеза. Матрица Адамара n × n существует для любого числа n, делящегося на 4.
Гипотеза не доказана даже для некоторых чисел, меньших 1000;
а именно, для 668, 716, 892.
Для решения двух следующих задач потребуются простейшие
свойства квадратичных вычетов; см. [O, §9], [Vi, §5], [Z, 3.8].
(
)
∑ j(j + d)
7.2.4. Для простого числа p обозначим Sd = Sp,d :=
p
j∈Zp
(это сумма символов Лежандра).
7. АЛГЕБРАИЧЕСКИЕ МЕТОДЫ
197
(a) Докажите, что Sd не зависит от d ̸= 0.
(b) Найдите Sd для каждого d ∈ Zp .
7.2.5. Постройте матрицу Адамара n × n для n, равного
(2a) 2a, если существует матрица Адамара a × a;
(ab) ab, если существуют матрицы Адамара a × a и b × b;
(4k) p + 1, где p — простое число вида 4k − 1;
(8k+4) 2p + 2, где p — простое число вида 4k + 1.
Матрица Адамара называется нормализованной, если у неё первая строчка и первый столбец состоят из одних единиц.
7.2.6. Нарисуйте все нормализованные матрицы Адамара порядков
1; 2; 4.
7.2.7. Адамаровость матрицы сохраняется при следующих преобразованиях:
(1) умножение строчки или столбца на −1;
(2) перестановка строчек или столбцов местами.
Матрицы Адамара, получаемые друг их друга применением некоторого числа преобразований (1) и (2), называются эквивалентными.
7.2.8. (a) Какие из матриц из задачи 7.2.6 эквивалентны?
(b) Любая матрица Адамара эквивалентна некоторой нормализованной.
Количество классов эквивалентности: для порядков 1,2,4,8,12 —
1, 16 — 5, 20 — 3, 24 — 60, 28 — 487, 32 — больше миллиона.
7.2.9. Для любых ли матриц Адамара H и H ′ матрицы H ⊗ H ′ и
H ′ ⊗ H эквивалентны? Здесь тензорное произведение ⊗ определено
в указании к задаче 7.2.5.(ab).
7.2.10. Матрица Адамара H, построенная при помощи конструкции (Пэйли) из задачи 7.2.5.(8k + 4), эквивалентна матрице H T .
7.2.11.* Существует ли матрица Адамара H, не эквивалентная матрице H T ?
198
7.3
Элементы дискретной математики в задачах
Подсказки
7.1.2 . (a) Если подмножеств больше n, то по теореме о корректности определения размерности одно из них равно симметрической
разности некоторых других: A = B1 ⊕ B2 ⊕ . . . ⊕ Bs .
(c) Возьмём векторы v1 , . . . , vs ∈ {0, 1}n , соответствующие подмножествам {A1 , . . . , As } = F. По теореме о корректности определения размерности достаточно показать линейную независимость
этих векторов над Q.
7.1.3 . (b) Для семейства U подмножеств множества {1, 2, . . . , n},
замкнутого относительно суммы по модулю 2 (т. е. для линейного
подпространства в Zn2 ), обозначим
U ⊥ := {X ⊂ {1, 2, . . . , n} : |X ∩ Y | чётно для любого Y ∈ U }.
Докажите, что dim U ⊥ 6 n − dim U .
7.1.4 . (d) Докажем, что если попарные расстояния между k точками в Rn равны, то k 6 n+2. Это сложное доказательство более слабой верхней оценки n+2 интересно тем, что его обобщение работает
для двух расстояний. Обозначим через a данное расстояние. Многочлен Pv (x1 , . . . , xn ) является линейной комбинацией (с коэффициентами, не зависящими от x1 , . . . , xn ) многочленов |x|2 , x1 , . . . , xn и
1. В этом списке n + 2 многочлена. По (c) многочлены, соответствующие данным точкам в Rn , линейно независимы над Q. Поэтому
количество точек не превосходит n + 2.
7.1.5 . (a) Следует из (b).
(b) Следует из (c).
(c) Пусть, напротив,
λ1 F⃗a1 + . . . + λs F⃗as = 0
для некоторых ⃗a1 , . . . , ⃗as ∈ V25,9 и рациональных λ1 , . . . , λs , причем не все λk равны нулю. Умножим и поделим это равенство на
некоторые целые числа, чтобы сделать λ1 , . . . , λs целыми, не все из
которых делятся на 3. Не уменьшая общности, можно считать, что
λ1 не делится на 3. Подставим в полученное равенство значения
x1 = (⃗a1 )1 , . . . , xn = (⃗a1 )n .
204
8
Элементы дискретной математики в задачах
Теоремы об инцидентностях в геометрии
8.1
Задачи
8.1.1. Теорема Сильвестра-Галлаи. Пусть даны n точек на плоскости, не все точки лежат на одной прямой. Тогда найдётся прямая,
которая пройдёт ровно через две точки.
e изображения G
e графа на плоскоЧислом скрещиваний cr(G)
сти называется число пар таких пересекающихся ребер, которые не
имеют общих вершин.
Числом скрещиваний cr(G) графа G называется минимальное
число скрещиваний среди всех изображений графа на плоскости.
Замечание. Планарные графы составляют ничтожную долю от
всех графов: для планарности в графе должно быть «мало» рёбер,
в то время, как в «типичном» графе число рёбер квадратично по
числу вершин. Поэтому вместо того, чтобы делить мир на чёрное и
белое, планарные и непланарные графы, часто хочется классифицировать графы более тонко, по степени их «удалённости от планарных». Соответствующих характеристик непланарности несколько.
Одна из главных — это число скрещиваний.
8.1.2. Докажите, что для любого графа G с n вершинами и e рёбрами выполнено неравенство cr(G) > e − 3n.
8.1.3.* Дан граф G с n вершинами и e рёбрами.
e3
(a) Если e > 4n, то cr(G) > 64·n
2.
3
e
(b) cr(G) > 64·n
2 − n.
Пусть P — множество некоторых точек на плоскости, L — множество некоторых прямых на плоскости. Числом инциденций I(P, L) :=
|{(p, l) ∈ P × L : p ∈ l}| называется количество пар вида (точка на
прямой, прямая), где прямая и точка взяты из соответствующих
множеств. Обозначим через I(n, m) максимальное число инциденций для всех конфигураций из n различных точек и m различных
прямых на плоскости.
8.1.4.
∗
Для произвольного n верно неравенство I(n, n) > 2n4/3 .
8. ТЕОРЕМЫ ОБ ИНЦИДЕНТНОСТЯХ В ГЕОМЕТРИИ
205
8.1.5. Пусть P — множество из n различных точек на плоскости,
L — множество из m различных прямых. Пусть G — граф, определяемый следующим образом. Вершины G соответствуют точкам из
множества P , а ребро между двумя вершинами G проводится если
и только если две соответствующие точки из множества P лежат
на какой-либо прямой из множества L рядом, т. е. не разделены
другой точкой из множества P . Таким образом, рёбрам графа G
соответствуют отрезки прямых из множества L. Пусть e — число
рёбер в графе G.
(a) cr(G) 6 m2 .
(b) e > I(P, L) − m.
(c) Теорема Семереди-Троттера. I(P, L) 6 4(mn)2/3 + m + 4n.
Замечание. Из задач 8.1.4 и 8.1.5.c следует, что I(n, n) ∼ cn4/3 .
Важность этого результата, в частности, в том, что он показывает
комбинаторные различия между плоскостью R2 и конечными проективными плоскостями. (См. указание к задаче 5.8.3, а также [J,
12.4] Для них соответствующая формула выглядела бы как cn3/2 .
Из этой задачи выросла целая область, которая изучает различные
обобщения данного вопроса, например, на случай полиномиальных
кривых, пространств бо́льших размерностей и т.п. Кроме того, она
тесно связана с задачами о расстояниях, которые мы обсудим ниже,
и с вопросами о сложности геометрических конфигураций. Стоит
отметить, что исторически первый вопрос в духе вопроса Эрдеша об
инциденциях был поставлен Сильвестром ещё в 19 веке (см. задачу
8.1.1). Однако, он не получил должного внимания.
8.1.6. Существует такое число c, что для любых n точек плоскости
√
верно следующее утверждение. Для 2 6 k 6 n число прямых,
каждая из которых содержит по крайней мере k из этих точек, не
превосходит cn2 /k 3 .
8.1.7.* Теорема Спенсера—Семереди—Троттера. Существует такое
c, что для любых n точек плоскости количество неупорядоченных
пар точек, находящихся на расстоянии 1, не превосходит cn4/3 .
9. АДДИТИВНАЯ КОМБИНАТОРИКА
9
209
Аддитивная комбинаторика
9.1
Задачи
Суммой A+B подмножеств A, B ⊂ R или A, B ⊂ Zm называется
множество
A + B := {a + b : a ∈ A, b ∈ B}.
9.1.1. (a) Если A, B ⊂ Z4 , |A + B| = |A| ̸= 0 и |B| = 2, то |A| ∈
{2, 4}.
(b) Если A, B ⊂ Z6 , |A + B| = |A| ̸= 0 и |B| = 2, то |A| ∈ {3, 6}.
Базовые свойства суммы множеств даются следующей задачей.
9.1.2. Для любых конечных подмножеств A, B ⊂ R
(a) max{|A|, |B|} 6 |A + B| 6 |A||B|;
(b) |A| 6 |A + A| 6 |A|(|A|+1)
.
2
9.1.3. (a) Если A = {2j : j = 0, 1, . . . , n − 1}, то |A + A| = n(n+1)
.
2
(b) Для любой конечной арифметической прогрессии P ⊂ R верно
неравенство |P + P | = 2|P | − 1.
Замечание. Задача 9.1.3.a даёт пример подмножества, для которого достигается верхняя оценка в задаче 9.1.2.b. Задача 9.1.3.b даёт
пример подмножества, для которого «почти достигается» нижняя
оценка в задаче 9.1.2.b. Пример подмножества, для которого эта
оценка достигается — произвольное одноэлементное подмножество.
9.1.4. Теорема Кнезера для R. Для любых конечных подмножеств
A, B ⊂ R верно неравенство |A + B| > |A| + |B| − 1.
Следующий Zp -аналог можно использовать в дальнейшем без
доказательства.
Теорема Коши–Давенпорта Для любого простого числа p и
для произвольных двух подмножеств A, B ⊂ Zp верно неравенство
|A + B| > min{|A| + |B| − 1, p}.
210
Элементы дискретной математики в задачах
9.1.5. (a) Если A, B ⊂ R и |A + B| = |A|, то |B| = 1.
(b) Если p простое, A, B ⊂ Zp и |A + B| = |A|, то либо |B| 6 1,
либо A = Zp .
Аналогично сумме множеств можно определить их разность:
A − B := {a − b : a ∈ A, b ∈ B}.
9.1.6. Неравенство Ружи. Для любых конечных подмножеств A, B, C ⊂
R
|C||A − B| 6 |A − C||B − C|.
9.1.7. (a) Для любого непустого конечного подмножества X ⊂ R
|X + X|2
выполнено неравенство |X − X| 6
.
|X|
(b) Для любых непустых конечных подмножеств X, Y ⊂ R выпол|X + Y |2
нено неравенство |X + X| 6
.
|Y |
(c) Для любых конечных подмножеств X, Y ⊂ R таких, что X ∩
|X + X||Y + Y |
Y ̸= ∅ выполнено неравенство |X + Y | 6
.
|X ∩ Y |
9.1.8. Для любых непустых подмножеств A, B ⊂ R следующие
утверждения эквивалентны:
(1) |A + B| = |A||B|;
(2) |A − B| = |A||B|;
(3) |{(a1 , a2 , b1 , b2 ) ∈ A × A × B × B : a1 + b1 = a2 + b2 }| = |A||B|;
(4) |{(a1 , a2 , b1 , b2 ) ∈ A × A × B × B : a1 − b1 = a2 − b2 }| = |A||B|;
(5) |A ∩ ({x} − B)| = 1 для любого x ∈ A + B;
(6) |A ∩ (B + {y})| = 1 для любого y ∈ A − B;
(7) (A − A) ∩ (B − B) = {0}.
Сумма произвольных подмножеств X1 , X2 , . . . , Xn ⊂ R определяется как
X1 +X2 +. . .+Xn := {x1 +x2 +. . .+xn : xi ∈ Xi для любого 1 6 i 6 n}.
Для произвольного подмножества X ⊂ R и любого целого k > 0
положим
kX := X
{z . . . + X} .
| +X +
k раз
9. АДДИТИВНАЯ КОМБИНАТОРИКА
211
Неравенство Ружи (задача 9.1.6) можно обобщить на случай
суммы n множеств.
Неравенство Плюннеке–Ружи. Для любых непустых конечных подмножеств A1 , A2 , . . . , An , B ⊂ R
|A1 + A2 + . . . + An | 6
|A1 + B||A2 + B| . . . |An + B|
.
|B|n−1
9.1.9. Для любого конечного подмножества
A )⊂ R и произвольного
(|A|+n−1
n ∈ N верны неравенства |A| 6 |nA| 6
.
n
Аналогично сумме множеств можно определить их произведение:
AB := {ab : a ∈ A, b ∈ B}.
9.1.10. Если A, B ⊂ Z5 \ {0}, |AB| = |A| =
̸ 0 и |B| = 2, то |A| ∈
{2, 4}.
9.1.11. (a) Существует такое c > 0, что для любого конечного
множества A ⊂ R выполняется неравенство max{|A + A|, |A · A|} >
c|A|5/4 .
(b) Существует такое c > 0, что для любой арифметической прогрессии P ⊂ R длины более 3 верно неравенство |P · P | > c|P |5/4 .
Замечание. На самом деле, если P — арифметическая прогрессия длины n во множестве целых чисел, то можно улучшить оценку
из задачи 9.1.11.b. А именно, для любого ε ∈ (0, 2) существует такое
c > 0, что |P · P | > c|P |ε . Доказательство этого факта использует
нетривиальные теоремы из теории чисел, поэтому выходит за рамки этой книги.
9.1.12. Дано произвольное конечное непустое подмножество A ⊆ R.
Обозначим через L семейство прямых на плоскости, задаваемых
уравнением y = a(x − b), где a, b ∈ A. Обозначим P := (A + A) ×
(A · A). Докажите, что общее количество инциденций (определение
смотри в §8) между прямыми из L и точками из P не меньше, чем
|A|3 .
212
9.2
Элементы дискретной математики в задачах
Подсказки
9.1.1 . Задача решается перебором.
9.1.2 . (b) Для любых двух a1 , a2 ∈ A верно равенство a1 + a2 =
a2 + a1 . Дальше используйте определение.
9.1.3 . (a) Рассмотрите четыре целых числа 0 6 i, j, k, l 6 n − 1
таких, что i 6 j, k 6 l, i =
̸ k, и покажите, что 2i + 2j ̸= 2k + 2l .
9.1.4 . Упорядочим каждое множество и рассмотрим |A| + |B| − 1
заведомо разных сумм.
9.1.5 . (a) Используйте теорему Кнезера.
9.1.6 . Используйте формулу a − b = (a − c) + (c − b).
9.1.8 . Докажите и используйте следующий факт: если |A + B| =
|A||B|, то для любого элемента z ∈ A + B существует единственная
пара (a, b) ∈ A × B, для которой z = a + b.
9.1.9 . Используйте определение.
9.1.11 . (a) Рассмотрите множество точек P и множество прямых
L, определённые в задаче 9.1.12. Запишите оценку количества инциденций между прямыми из L и точками из P, которую даёт теорема
Семереди-Троттера (задача 8.1.5). Сравните её с оценкой из задачи
9.1.12. Из получившегося двойного неравенства выведите требуемое
неравенство.
9.1.12 . Посчитайте |P| и |L|. Потом докажите, что каждая прямая
из L содержит по крайней мере |A| точек из P.
10. ГРАФЫ: ЗАДАЧИ ДЛЯ ИССЛЕДОВАНИЯ
10
215
Графы: задачи для исследования
10.1
Степенные последовательности. А. Скопенков
0.* (a) Можно ли опустить какие-нибудь неравенства из задачи 5 так, чтобы достаточность (т.е. утверждение задачи 10) осталась верной? Если да, попробуйте найти минимальный набор неравенств.
(b) Если слить две степенные последовательности, то обязательно получится степенная последовательность. А какие степенные последовательности обладают тем свойством, что их нельзя разбить
на две степенные последовательности?
Граф называется планарным, если его можно нарисовать на
плоскости так, чтобы внутренности ребер (то есть ребра без их концов) не пересекались и не самопересекались. Подробнее см. [?, ?, ?].
1. (abcd*) То же, что в задаче 2 предыдущей темы, для планарных графов.
2. (abcd*) То же, что в задаче 2 предыдущей темы, для связных
планарных графов.
Тор, лист Мебиуса и сфера с ручками изображены на рис.
Тор, лист Мебиуса, цилиндр и сфера с ручками
Пусть на торе (или на сфере с ручками, или на диске с листами Мебиуса) нарисован без самопересечений граф. Назовем гранью
замкнутую область на торе, ограниченную ребрами этого графа.
Реализация графа на торе (или на сфере с ручками) называется клеточной, если каждая грань разбивается любой ломаной с
концами на границе этой грани (т.е. топологически эквивалентна
замкнутому диску на плоскости).
3. (abcd*) То же, что в задаче 2 предыдущей части, для связных
клеточно реализуемых на торе графов.
4. (abcd*) То же, что в задаче 2 предыдущей части, для связных
клеточно реализуемых на сфере с g ручками графов (g дано вместе
с n и d1 , . . . , dn ).
Для получения необходимых условий в задачах 3 и 4 нужен следующий факт.
216
Элементы дискретной математики в задачах
Формула Эйлера для сферы с g ручками. Пусть на сфере
с g ручками нарисован без самопересечений клеточно связный граф
с V вершинами и E ребрами. Пусть F — число граней. Тогда V −
E + F = 2 − 2g.
Для решения следующих задач 5 и 6 полезен аналог этого факта
для диска с m листами Мебиуса.
Реализация графа на листе Мебиуса (или на диске с листами Мебиуса, нарисованном на лекции) называется клеточной, если одна
грань топологически эквивалентна кольцу, а каждая из оставшихся
остальных — замкнутому диску на плоскости.
5. (abcd) То же, что в задаче 2 предыдущей части, для связных
клеточно реализуемых на листе Мебиуса графов.
6. (abcd) То же, что в задаче 2 предыдущей части, для связных
клеточно реализуемых на диске с m листами Мебиуса графов (m
дано вместе с n и d1 , . . . , dn ).
7-11. (abcd) То же, что в задачах 3-6, если требуется построить
граф с n гранями, в границе которых d1 , . . . , dn ребер, соответственно.
Пункты (a,b) задач 3-6 известны [Mo]. По-видимому, пункты (c)
этих задач неизвестны Пункты (d) этих задач неизвестны.
[Mo] B. Mohar, 2-cell embeddings with prescribed face lengths and
genus,
Ann.
Combin.
14
(2010)
525-532.
http://www.fmf.uni-lj.si/~mohar/Reprints/Inprint/BM06_AC_Mohar_
2cellEmbeddings.pdf.
10.2
Гамильтоновость (А. Веснин, А. Скопенков)
Цикл (путь) называется гамильтоновым, если он проходит через каждую вершину графа ровно по одному разу. Граф называется
гамильтоновым, если в нем есть гамильтонов цикл. См. подробнее
§2.6.
1. (a) Нарисуйте гамильтоновы циклы в графах правильных
многогранников.
(b) Никакой граф, гомеоморфный букве θ (т.е. графу K3,2 ), не
является гамильтоновым.
10. ГРАФЫ: ЗАДАЧИ ДЛЯ ИССЛЕДОВАНИЯ
217
Пусть H — граф. Граф X называется H-гамильтоновым, если в X существует подграф, содержащий все вершины графа X и
гомеоморфный графу H.
Следующая задача (кроме (f)) проста и приводится для того,
чтобы помочь решателю войти в курс дела. Задачи, отмеченные
звездочкой, являются нерешенными.
2. (a) Любой гамильтонов граф, отличный от окружности, является θ-гамильтоновым.
(b) Существует θ-гамильтонов граф, не являющийся гамильтоновым.
(c) Существует гамильтонов граф, отличный от окружности и
θ, не являющийся K4 -гамильтоновым.
(d) Существует K4 -гамильтонов граф, не являющийся θ-гамильтоновым.
(e) Для любых графов G и H существует G-гамильтонов граф,
не являющийся H-гамильтоновым.
(f)* Опишите "иерархию"графов по их гамильтоновости: когда
H-гамильтонов граф является G-гамильтоновым?
2’. (b,d*,e*,f*) То же, что в 2, для графов многогранников.
Граф Погорелова — граф выпуклого многогранника в трехмерном пространстве,
(1) из каждой вершины которого исходит три ребра,
(2) каждая замкнутая несамопересекающаяся ломаная на поверхности многогранника, разделяющая какие-либо две его грани,
пересекает по крайней мере пять ребер многогранника.
3. (Greenbergs) Постройте не гамильтонов граф Погорелова.
4. (a) Негамильтонов граф Погорелова из [Boll, Kokseter, Matematicheskie
esse i razvlecheniya, M, Mir, 1986, str. 285] является θ-гамильтоновым.
(b)* (гипотеза) Существует K4 -гамильтонов граф Погорелова,
не являющийся θ-гамильтоновым.
Идея доказательства: udalit’ trehvalentnuyu vershiny i na ee mesto
vkleit’ graf Greenbergsa s udalennoi vershinoi.
5.* (a) Опишите "иерархию"графов Погорелова по их гамильтоновости: когда H-гамильтонов гиперболический граф является
G-гамильтоновым?
218
Элементы дискретной математики в задачах
(b) (гипотеза) Для любого графа G существует граф Погорелова, не являющийся G-гамильтоновым. (Идея доказательства: "vkleivanie"grafa
Greenbergsa.)
(c) Для любых ли графов G и H существует G-гамильтонов граф
Погорелова, не являющийся H-гамильтоновым?
ва,
6.* Постройте минимальный (по числу граней) граф Погорело-
(a) являющийся K4 -гамильтоновым, но не θ-гамильтоновым.
(b) не являющийся K4 -гамильтоновым.
(c) не являющийся H-гамильтоновыми ни для какого подграфа
H данного графа G. (Например, G = K4 .)
10.3
Изоморфизмы графов. И.Н. Шнурников
Используемое здесь определение изоморфизма графов напомнено в [?].
1. (a) Сколько существует изоморфизмов K5 → K5 ? А K3,3 →
K3,3 ?
(b) Изоморфны ли графы G2 и G3 , вершины каждого из которых занумерованы числами от 1 до 7, вершины графа Gk соединены
ребром, если либо i − j ≡ 1 mod 7, либо i − j ≡ k mod 7.
(c) Постройте граф с наименьшим числом n > 1 вершин такой,
что никакая не тождественная перестановка его вершин не является
изоморфизмом.
2. Симметричные графы. Мы будем работать со связными ориентированными графами, из каждой вершины графа выходят два
ребра и в каждую входят два ребра. Такой граф назовем симметричным, если для любой пары ребер a, b существует перестановка вершин графа (а если есть кратные ребра, то и перестановка
их между собой), при которой все ребра графа переходят в ребра
этого же графа, а ребро a переходит в ребро b (направления всех
ребрах сохраняются). При этом никакое ребро не должно остаться
на месте.
(a) Для каждого натурального n придумайте два (неизоморфных) симметричных графа с n вершинами каждый.
10. ГРАФЫ: ЗАДАЧИ ДЛЯ ИССЛЕДОВАНИЯ
219
(b) Придумайте симметричные графы с 6, 12 и 30 вершинами,
не изоморфные графам из (a).
(c) Найдите все симметричные графы, которые имеют хотя бы
одну петлю или хотя бы одно кратное ребро.
(d) Найдите все симметричные графы с p-вершинами (p — простое число).
(e) Найдите все симметричные графы с не более, чем 8 вершинами.
(f) Найдите все симметричные графы, которые можно нарисовать (без самопересечений) на плоскости так, что для каждой вершины входящие ребра чередуются с выходящими.
(g)* Найдите все плоские симметричные графы.
3. У Васи есть несвязный граф. Он всеми возможными способами удалил из этого графа по одной вершине и каждый из полученных графов нарисовал на отдельном листочке бумаге, после чего
все листочки отдал Коле. Докажите, что Коля может восстановить
исходный граф.
4*. Нерешенные задачи о вершинной и реберной реконструируемости.
(a) Пусть G и G̃ — связные графы без петель и кратных ребер
с V > 3 занумерованными вершинами. Для каждого k ∈ {1, . . . , V }
рассмотрим графы G − k и G̃ − k, полученные из графов G и G̃
удалением в каждом из них вершины с номером k и всех выходящих из нее ребер. Пусть для всех k ∈ {1, . . . , V } графы Gk и G̃k
изоморфны. Верно ли, что графы G и G̃ изоморфны?
(b) Пусть G и G̃ — простые графы с E > 5 ребрами. Для каждого k ∈ {1, . . . , E} рассмотрим графы Gk и G̃k , полученные из графов
G и G̃ соответственно путем удаления в каждом из них ребра с номером k. Пусть для всех k ∈ {1, . . . , E} графы Gk и G̃k изоморфны.
Верно ли, что графы G и G̃ изоморфны?
10.4
Турниры. Д. Пермяков
Для решения основной задачи этого пункта потребуются некоторые навыки работы с графами. Перед этой серией полезно (но не
обязательно) прорешать пункт ’Пути в графах’.
220
Элементы дискретной математики в задачах
Ориентированный граф – граф, каждое ребро которого является стрелкой от одной вершины ребра к другой. Турнир — ориентированный граф, между любыми двумя вершинами которого есть
ровно одно ребро. Ориентированный граф называется сильносвязным, если от любой его вершины можно добраться до любой другой,
двигаясь по направлению стрелок на ребрах.
1. (Основная) Какое минимальное количество несамопересекающихся циклов длины k может быть в сильносвязном турнире с
n вершинами?
Эта задача сложная, к ней непросто подступиться. Обычно при
решении сложной задачи полезно рассмотреть частные случаи, попытаться решить близкие задачи. Это позволяет заметить закономерности, которые можно сформулировать в виде гипотез и затем
доказать. Мы не будем подсказывать эти гипотезы, а предлагаем
вам самим исследовать эту задачу и высказывать ваши предположения. Сформулируйте и докажите какую-нибудь лемму, которая,
по вашему мнению, поможет в решении задачи 1. После того, как
вы попробовали найти путь к решению самостоятельно, предлагаем
вам доказать следующие утверждения. Они могут оказаться полезными в решении Задачи 1 и, возможно, помогут довести его до
конца.
2. (a) Турнир сильносвязен тогда и только тогда, когда в нем
есть несамопересекающийся ориентированный цикл (т.е., цикл, идущий по направлениям стрелок на ребрах), проходящий по всем вершинам.
(b) В сильносвязном турнире через любую вершину проходит
несамопересекающийся ориентированный цикл любой длины от трех
до количества вершин турнира.
Ответы, указания и решения
10.1.1, 2. (a,b,c) То же, что в 11abc.
(d) Не знаем.
10.1.4. (a) e целое и e > n − 1 + 2g [Mo].
max di и e > n − 1 + 2g [Mo].
10.1.6. (a) e целое и e > n−1+m [Mo].
и e > n − 1 + m [Mo].
(b) e целое, e >
(b) e целое, e > max di
Предметный указатель
Локальная Лемма Ловаса, 145, 152 эквивалентность матриц Адамара, 175
антиклика, 55
формула
асимптотика функции, 136
Эйлера, 62
бином Ньютона, 10
Кэли, 58
булев куб, 18
включений и исключений, 13
цикл, 55
формула Стирлинга, 137
эйлеров, 67
функция
гамильтонов, 69
Эйлера, 12
несамопересекающийся, 55
гомеоморфность графов, 64
простой, 55
граф, 54
в графе, 55
Петерсена, 94
в смысле теории гомологий,
59
дистанционный, 77
числа
двудольный, 55
Стирлинга второго рода, 10
двусвязный, 73
число
несвязный, 56
инциденций, 180
ориентированный, 57
скрещиваний графа, 180
планарный, 64
число Рамсея
плоский, 62
для гиперграфов, 104
полный, 55
для подграфов, 106
ребёрный, 71
двухцветное, 101
с петлями и кратными рёбрами, 66
многоцветное, 103
связный, 56
дерево, 57
трёхсвязный, 74
диаметр множества, 72, 173
унициклический, 59
дисперсия, 154
грань графа, 62
длина цикла, 55
длина пути, 55
хроматический индекс, 94
221
222
Элементы дискретной математики в задачах
хроматический многочлен, 95
хроматическое число, 93
изоморфность графов, 60
карта
на плоскости, 64
на торе, 65
клика, 55
код Прюфера, 58
кратность
ребра, 66
лемма
Дирака, 69
лента Мёбиуса, 65
лес, 96
остовный, 96
линейная зависимость, 170
линейное подпространство, 171
лист, 57
математическое ожидание, 154
матрица Адамара, 174
нормализованная, 175
многочлен
Татта, 96
многогранник
Гринбергса, 70
мост, 96
мультиграф, 66
де Брёйна, 76
двусвязный, 73
ориентированный, 66
неравенства
Бонферрони, 13
неравенство
Чебышёва, 155
Маркова, 155
независимое множество, 55
независимость, 143
от набора, 145
в совокупности, 144
обход, 55
ортогональность, 171
остов графа, 57
перманент, 123
подграф, 55
подразделение ребра графа, 64
пороговая вероятность, 156
последовательность
де Брёйна, 68
правило
Паскаля, 10
правило «0 лучше 1», 68
правильная раскраска
карты, 64
рёбер графа, 93
вершин графа, 91
правильные многогранники, 63
принцип
Дирихле, 15
путь, 55
эйлеров, 67
гамильтонов, 69
ориентированный, 57
в графе, 55
размерность
Вапника-Червоненикиса, 124
линейного пространства, 171
ребро графа, 54
инцидентное вершине, 54
кратное, 66
система общих представителей, 119
система различных представителей, 121
ПРЕДМЕТНЫЙ УКАЗАТЕЛЬ
223
скалярное произведение, 171
инцидентная ребру, 54
изолированная, 55
случайная величина, 154
жадный алгоритм, 119
случайный граф, 154
раскраски вершин графа, 93
степень вершины, 55
исходящая, 67
входящая, 67
стягивание ребра графа, 56
сумма множеств, 185
связная компонента графа, 56
теорема
Биркгофа–Уитни, 95
Брукса, 92
Дирихле, 15
Эрдеша, 102
Эрдеша–Стоуна–Шимоновица,
136
Эрдеша-Секереша, 104
Фари, 65
Хопкрофта-Тарджана, 65
Коши-Давенпорта, 185
Куратовского, 64
Менгера
рёберная, 74
вершинная, 74
Семереди-Троттера, 181
Шура, 104
Татта, 96
Турана, 72
Уитни
вершинная, 74
Ван-дер-Вардена, 106
тор, 65
треугольник в графе, 72
турнир, 57
вектор, 170
вершина графа, 54
224
11
Элементы дискретной математики в задачах
Программа курса ДА 2014-15 уч. года
Нужно уметь решать задачи, аналогичные пп. 2.1-2.7, 3.1, 3.2,
4.1, 4.2, 4.5, 5.1, 5.2, 5.5, 6.1-6.3, 7.1, 7.2 книги
Элементы дискретной математики в задачах, А.А. Глибичук,
А.Б. Дайняк, Д.Г. Ильинский, А.Б. Купавский, А.М. Райгородский,
А.Б. Скопенков, А.А. Чернов, Изд-во МЦНМО, в печати,
http://www.mccme.ru/circles/oim/discrbook.pdf
В скобках указана ориентировочная сложность пункта (или части пункта) программы. Формального смысла эти баллы не имеют.
Но мы надеемся, что они помогут студентам разумно организовать
подготовку к экзамену: не изучать более ‘сложных’ пунктов программы, пока не изучены более ‘простые’. Пунктами ‘на 5 и меньше’ студенты (и преподаватели) могут пользоваться в дальнейших
курсах (без повторения материала).
«Без доказательства» сокращается до «б/д». В пунктах программы приводятся ссылки на вышеуказанную книгу (или на имеющийся в ней список литературы или на другую литературу).
Образцы вопросов приведены после программы.
Глава 2. Графы (1-й семестр)
1. (4) Определение графа, графов с петлями и кратными ребрами. Ориентированые графы. Соотношение между числом вершин и
ребер дерева. (П. 2.1 и задачи 2.2.1.)
2. (5) Код Прюфера. Формула Кэли. (Задачи 2.2.3.a и 2.2.4.c.)
3. (6) Точная формула для числа унициклических графов. (Задача
2.2.5.b.)
4. (5) Определение плоских и планарных графов. Формула Эйлера (б/д). Примеры непланарных графов. Критерий Куратовского
планарности графов (б/д). (П. 2.4 и задачи 2.4.2.)
5. (6) Классификация правильных многогранников с точностью до
изоморфизма их графов. (Задачи 2.4.3.cdefg.) 6-раскрашиваемость
любой карты на плоскости. (Задача 2.4.4.a.)
11. ПРОГРАММА КУРСА ДА 2014-15 УЧ. ГОДА
225
6. (3) Пути и циклы. Простые пути и циклы (обходы). (П. 2.1.)
Критерии эйлеровости графа и ориентированного графа. (Задачи
2.5.3.ac.)
7. (5) Последовательности и графы де Брёйна. Правило «ноль лучше единицы». (Задачи 2.5.5–2.5.8.)
8. (5) Гамильтоновы пути и циклы. Достаточное условие Дирака
гамильтоновости графа. (Задача 2.6.2.b.)
9. (6) Вершинная связность и число независимости графа. Достаточное условие гамильтоновости в их терминах. Гамильтоновость
графа 1-пересечений 3-элементных подмножеств n-элементного множества. (Задачи 2.6.3, 2.6.4 и 2.6.5.c.)
10. (3) Гамильтоновы цепи в турнирах. Нижняя оценка с доказательством, верхняя — без. (Задача 2.6.7.)
11. (5) Теорема Турана о числе ребер в графе с данным числом
вершин и числом независимости. Асимптотика наибольшего числа
ребер в графе с n вершинами без k-клик. (Задачи 2.7.1 и 6.1.2.)
12. (6) Оценка числа ребер у дистанционного графа на плоскости
и в пространстве произвольной размерности. Сравнение с теоремой
Турана. (Задачи 2.7.2, 2.7.4, 2.7.5.)
Глава 3. Раскраски графов (1-й семестр)
13. (5) Число независимости и кликовое число. Хроматическое число. Соотношения между хроматическим числом, числом независимости и кликовым числом. (Задача 3.1.3.)
Глава 4. Основы теории Рамсея (2-й семестр)
14. (3) Числа Рамсея R(s, t): точные значения для s + t 6 7. Рекуррентная верхняя оценка Эрдеша–Секереша. (Задачи 4.1.1, 4.1.2.ac.)
15. (4) Следствие рекуррентной верхней оценки Эрдеша–Секереша
для недиагональных и диагональных чисел Рамсея. Уточнение Конлона (б/д). Нижняя оценка диагональных чисел Рамсея с помощью
простого вероятностного метода. (Задачи 4.1.2.b, 4.1.5.)
226
Элементы дискретной математики в задачах
16. (5) Многоцветные числа Рамсея Rk (l1 , . . . , lr ) и их рекуррентная верхняя оценка. Следствие для R3 (s, t). Нижняя вероятностная
оценка для R3 (s, s). (Задачи 4.2.2.b, 4.2.7.)
17. (8) Верхняя оценка Конлона для двудольных чисел Рамсея:
лемма с конкретными l, m, r, s и ее аналог с последовательностями
(б/д); доказательство оценки с использованием леммы. (Есть оригинальная статья Conlon’а, скачивается с его домашней страницы.)
18. (8) Конструктивная нижняя оценка Франкла–Уилсона для R(s, s).
[R3]
19. (6) Замечание о распределении простых в натуральном ряде
и его роли в аккуратном доказательстве нижней оценки Франкла–
Уилсона для R(s, s). [R3]
Глава 5. Системы множеств (гиперграфы) (20-23 1-й семестр, 24-31 2-й семестр)
20. (5) Гиперграфы. Гиперграфы t-пересечений. Теорема Эрдеша–
Ко–Радо (о максимальном числе ребер в гиперграфе 1-пересечений);
б/д. (Задача 5.1.3.)
21. (7) Теорема Эрдеша–Ко–Радо. (Задача 5.1.3.)
( )
22. (7) Пример n−t
k−t подмножеств n-элементного множества, в каждом из которых ровно k элементов и любые два из которых пересекаются не менее чем по t элементам.
23. (7) История последовательных продвижений в задаче: теорема Эрдеша–Ко–Радо (общий случай), теорема Франкла, теорема
Уилсона, теорема Алсведе–Хачатряна (все б/д, но с подробными
комментариями). (Задачи 5.1.2, 5.1.3.)
24. (2) Системы общих представителей (с.о.п.). «Тривиальные» нижние и верхние оценки.
25. (5) Верхняя оценка (размера минимальной) с.о.п. с помощью
жадного алгоритма. (Задачи 5.2.1, 5.2.5, 5.2.6.a.)
26. (9) Конструктивная нижняя оценка с.о.п. (Задача 5.2.6.b.)
11. ПРОГРАММА КУРСА ДА 2014-15 УЧ. ГОДА
227
27. (7) Нижняя оценка с.о.п. с помощью обобщенных с.о.п. (Задача
5.2.6.c.)
28. (10) Вероятностная нижняя оценка с.о.п. (Задача 5.2.6.def.) Следствие из нее.
29. (6) С.о.п. в геометрии (теорема о треугольниках на плоскости).
Размерность Вапника–Червоненкиса. (Задача 5.5.1.) Теорема Радона (б/д). Подсчет размерности семейства полупространств (Задача
5.5.2.bc.) Оценка числа подмножеств в семействе заданной размерности на n-элементном множестве. (Задача 5.5.9.) Лемма о размерности измельчения (с не очень подробными выкладками).
30. (8) Эпсилон-сети. Теорема Вапника–Червоненкиса об эпсилонсетях и теорема о треугольниках как частный случай.
31. (6) Теорема Вапника–Червоненкиса (б/д). Приложения в статистике: равномерная сходимость в ЗБЧ (УЗБЧ) и теорема Гливенко–
Кантелли как частный случай.
Глава 6. Аналитические и вероятностные методы (32-36
1-й семестр, 37-49 2-й семестр)
32. (3) Оценки для факториалов
( n )и биномиальных коэффициентов.
(Задача 6.1.7.a.) Оценки для n/2 с помощью тождества. (Задача
6.1.5.ab.)
√
33. (5) Асимптотика ln n! и n n! с доказательством без использования формулы Стирлинга. Формула Стирлинга (б/д). (Задача 6.1.6.)
( n )
34. (5) Оценка биномиальных коэффициентов вида [an]
, a ∈ (0, 1).
Аналогичный результат для полиномиальных коэффициентов. (Задача 6.1.5.cdef.)
( )
35. (5) Асимптотика для nk при k 2 = o(n).
( nОценки
) ( n той
) же величины при больших k. Асимптотики для n/2 / n/2−x . (Задача
6.1.7.bcde.)
36. (7) Асимптотика числа унициклических графов. (Задача 6.1.12.b.)
228
Элементы дискретной математики в задачах
37. (5) Симметричный и несимметричный случай ЛЛЛ (б/д). Вывод оценки диагонального числа Рамсея (теорема Спенсера). (Задачи 6.2.15, 6.2.21.b и 6.2.26.а.)
38. (6) Симметричный и несимметричный случай ЛЛЛ (с доказательством симметричного — либо напрямую, либо с доказательством несимметричного и выводом из него). (Задачи 6.2.15 и 6.2.26.а.)
39. (9) Вывод из несимметричного случая ЛЛЛ нижней оценки для
R(3, t): Самые точные известные оценки для R(3, t) (б/д). (Задача
6.2.26.b и замечание после нее.)
40. (5) Двудольные числа Рамсея: нижние оценки простым вероятностным методом и с помощью ЛЛЛ. Отличие нижних оценок
для двудольных чисел Рамсея от аналогичных нижних оценок для
R(s, t). (Литературы нет; делается совершенно аналогично тому,
как то же самое делается для обычных чисел Рамсея.)
41. (5) Случайные графы. Неравенства Маркова и Чебышёва. Неравенство для случайного блуждания. (п. 6.3, [R4, п. 1.11 и 1.12])
42. (5) Связность случайного графа (б/д). Случайные величины и
вероятностные неравенства, необходимые для доказательства. [R4,
п. 2.5]
43. (7) Связность случайного графа: случай p = c ln n/n при c < 1.
ln n + γ + o(1)
Теоремы о
и о гигантской компоненте (б/д). [R4, п.
n
2.5]
44. (8) Связность случайного графа: случай p = c ln n/n при c > 1.
[R4, п. 2.5]
45. (3) Теоремы Боллобаша о хроматическом числе случайного графа (б/д). Пояснения к ним: случаи p = o(1/n2 ) и p = o(1/n). [R4,
п. 2.6]
46. (9) Пояснения к теоремам Боллобаша о хроматическом числе
случайного графа: случай p = c/n, c < 1 (б/д) и случай, когда
функция из второй теоремы Боллобаша может стремиться к бесконечности. [R4, п. 2.6]
11. ПРОГРАММА КУРСА ДА 2014-15 УЧ. ГОДА
229
47. (7) Сравнение оценок хроматического числа через кликовое
число и число независимости в терминах случайных графов: одна
«почти всегда» значительно лучше другой (распределение кликового числа и числа независимости). [R4, п. 2.7]
48. (10) Теорема о том, что почти наверное жадный алгоритм найдет множество, размер которого лишь, как максимум, в 2 раза отличается от реального. Теорема Кучеры о слабости жадного алгоритма на специальных графах (б/д). (А. Райгородский, "Экстремальные задачи теории графов и Интернет" , Интеллект.)
49. (9) Теорема Эрдеша о графе с большим обхватом и большим
хроматическим числом. (Задача 6.3.3.c.)
Глава 7. Алгебраические методы (50-58 1-й семестр, 5964 2-й семестр)
50. (4) Граф пересечений для полного однородного гиперграфа.
Его кликовое число и число независимости (б/д). (Задача 5.1.3.)
51. (5) Кнезеровский граф. Верхняя оценка его хроматического
числа. Простые нижние оценки. Примеры конкретных кнезеровских графов. Кликовое число и число независимости кнезеровского
графа. ([R7] := А. Райгородский, "Гипотеза Кнезера и топологический метод в комбинаторике МЦНМО.)
52. (6) Верхняя оценка хроматического числа кнезеровского графа.
Теорема Ловаса о хроматическом числе кнезеровского графа (б/д;
с доказательством — на ‘8’). [R7]
53. (7) Теорема Борсука–Улама–Люстерника–Шнирельмана в разных формулировках, но с доказательством только в случае плоскости и трехмерного пространства. [R7]
54. (6) Максимальное число m(n, k, t) подмножеств n-элементного
множества, в каждом из которых ровно k элементов и среди которых любые два множества пересекаются не по t элементам. Точное
значение для m(n, 3, 1): явная конструкция и оценка по индукции.
Линейно-алгебраическая оценка для m(n, 3, 1). Аналогичная оценка
для m(n, 5, 2) и ее асимптотическая неулучшаемость. [R1]
230
Элементы дискретной математики в задачах
55. Общая теорема Франкла–Уилсона для m(n, k, k − p) при k < 2p
(6). Замечание о непростом «модуле» (10). (Задача 7.1.8 и [R1].)
56. (9) Теорема Франкла–Уилсона для m(n, k, k − p) при k > 2p.
[R1]
57. (7) Точность обеих теорем Франкла–Уилсона при постоянных k,
t. Максимальное число k-элементных подмножеств n-элементного
множества, из которых любые два множества пересекаются не более
чем по t элементам. Связь с теорией кодирования, теорема Редля
(б/д). [R1]
58. (6) Хроматические числа пространств. Историческая справка.
Интерпретация величины m(n, k, t) как числа независимости дистанционного графа. Нижняя оценка хроматического числа пространства с помощью результатов для m(n, k, t). Возможные улучшения. ([R1] и А. Райгородский, "Хроматические числа".)
59. (3) Матрицы Адамара. Необходимое условие существования.
Гипотеза Адамара. Нормализация. (Задачи 7.2.1-7.2.3, определения
и гипотеза в §7.2, [Hal].)
60. (5) Неудачная попытка построить матрицу Адамара «строчка
за строчкой».
61. (5) Теорема о плотности порядков матрицы Адамара в натуральном ряде (б/д). [Hal, AS]
62. (6) Приложения матриц Адамара к задаче о раскраске
гипер√
графа: определение уклонения, верхняя оценка вида 2n ln(2s) с
√
доказательством и оценка вида 6 n (б/д). [R3]
63. (8) Приложения матриц Адамара к задаче о раскраске гиперграфа: нижняя оценка с помощью матриц Адамара. [R3]
64. (6) Интерпретация в терминах дистанционного графа, возникающего в теореме Франкла–Уилсона (клики и независимые множества).
11. ПРОГРАММА КУРСА ДА 2014-15 УЧ. ГОДА
231
ОБРАЗЦЫ ВОПРОСОВ НА ЭКЗАМЕНЕ
Предварительная часть (вариант 2014 года). Нужен только ответ/формулировка; доказательства приводить не нужно.
( )
1. Найдите асимптотику биномиального коэффициента nk при
k 2 = o(n).
2. Найдите количество деревьев с данными n вершинами, с точностью до изоморфизма.
3. Дайте определение гамильтонова цикла в графе. (Можно использовать только определение графа. Если Вы используете другие
определения — например, цикла — то их тоже нужно дать.)
4. Сформулируйте теорему о хроматическом числе случайного
графа в модели G(n, p) при p = o(1/n) и n → ∞.
5. У дистанционного графа на плоскости 4n вершин, и среди
любых n+1 вершин есть ребро. Сформулируйте наилучшую оценку
на количество ребер такого графа, доказанную в курсе.
6. Найдите кликовое число графа, вершины которого — все 5элементные подмножества 20-элементного множества, и ребро между вершинами проводится в том и только в том случае, когда множества не пересекаются?
7. Найдите R4 (15, 4, 4, 4).
8. Найдите максимальную VC-размерность семейства подмножеств множества {1, . . . , 10} в каждом подмножестве которого более 5 элементов.
Основная часть (точно таких вопросов на экзамене не
будет). Здесь главное — не ответы, а доказательства. В частности, формулировки и доказательства всех используемых студентом результатов из курса ДА (в частности, всех результатов из
курса ДА, используемых для доказательства других результатов
из курса ДА). При этом можно пользоваться без доказательства
результатами из других курсов.
Вопрос из билета. Существует ли 57 подмножеств 60-элементного
множества, в каждом из которых 30 элементов, и любые два из которых пересекаются по 15 элементам?
(Если используется задача 7.2.4, то ее нужно доказать.)
Доп. вопрос попроще.
Каково наибольшее число ребер в
232
Элементы дискретной математики в задачах
графе с 52 вершинами, в котором среди любых 5 вершин есть 2, не
соединенные ребром?
(Если используется теорема Турана, то ее нужно доказать.)
Доп. вопрос посложней. Укажите функцию f (n), для которой R(n, n) & f (n). (Чем больше функция, тем выше Ваша оценка.)
(Если используется неравенство n! > (n/e)n (6.1.6.с) или ЛЛЛ
(6.2.15.b), то их нужно доказать; это неравенство доказывается без
использования формулы Стирлинга.)
Литература
[AM] Акопян А. В. , Мусин О. Р. О множествах с двумя расстояниями. Мат. Просвещение, 17 (2013), 136–151.
URL: http://www.mccme.ru/free-books/matpros/mph.pdf
[A] Арнольд В.И. Обыкновенные дифференциальные уравнения.
М.: Наука, 1984.
[AS] Алон Н., Спенсер Дж. Вероятностный метод. М.: Бином, 2011.
[B] Babai L., Frankl P. Linear algebra methods in combinatorics, Part
1. Department of Computer Science, The University of Chicago,
1992.
[BM] I. Bogdanov and A. Matushkin. Algebraic proofs of linear versions
of the Conway–Gordon–Sachs theorem and the van Kampen–
Flores theorem, http://arxiv.org/abs/1508.03185
[Bo] Bollobás B. Random Graphs. Cambridge studies in advanced
mathematics, 2001.
[BKS] Брагин В., Клячко А., Скопенков А. Когда любая группа из
n элементов циклическая?
URL: http://arxiv.org/abs/1108.5406.
[BKKSS] Баранов Д., Клячко А., Кохась K., Скопенков А., Скопенков М. Когда любая группа из n элементов циклическая?
URL: http://olympiads.mccme.ru/lktg/2011/6/index.htm
[EL] P. Erdős, L. Lovász, “Problems and results on 3-chromatic
hypergraphs and some related questions”, Infinite and Finite Sets,
233
234
Элементы дискретной математики в задачах
Colloquia Mathematica Societatis Janos Bolyai, 10, Amsterdam:
North Holland, 1973, 609–627.
[Ga] Гарднер М. Рамсеевская теория графов. // Квант, 1988, N4, с.
15–20, 82.
URL: http://kvant.mccme.ru/1988/04/ramseevskaya_teoriya_grafov.htm
[Gr] Григорьев И. Порождение перестановок «восьмёркой».
URL: http://www.mccme.ru/mmks/dec10/grigoryev_report.pdf
[G] Грэхем Р. Начала теории Рамсея. М.: Мир, 1984.
[GIF] С. А. Генкин, И. В. Итенберг и Д. В. Фомин, Ленинградские
математические кружки, Киров, 1994.
[GKP] Грэхем Р., Кнут Д., Паташник А. Конкретная математика.
М.: Мир, 1998.
[Har] Харари Ф. Теория графов. М.: УРСС, 2003.
[Hal] Холл М. Комбинаторика. М.: Мир, 1970.
[IRS] Ильинский Д., Райгородский А., Скопенков А. Независимость
и доказательства существования в комбинаторике. Мат. Просвещение, 19 (2015).
URL: http://arxiv.org/abs/1411.3171.
[IKRS] Ильинский Д., Купавский А., Райгородский А., Скопенков
А. Дискретный анализ для математиков и программистов (подборка задач). Мат. Просвещение, 17 (2013), с. 162–181.
URL: http://www.mccme.ru/free-books/matpros/matprosi.html.
[I]
Игнатьев М.В. Квантовая комбинаторика. Мат. Просвещение.
18 (2014), с. 66–111.
[Ja] Janson S., Luczak T., Rucinski A. Random Graphs. John Wiley,
2000.
[J] Jukna S. Extremal Combinatorics With Applications in Computer
Science. Springer-Verlag, XVII (2001).
ЛИТЕРАТУРА
235
[K] Калужнин Л. А., Сущанский В. И. Преобразования и перестановки. М.: Физматлит, 1985.
[KZP] Колмогоров А. Н., Журбенко И. Г., Прохоров А. В., Введение в теорию вероятностей. Серия «Библиотечка «Квант» »,
выпуск 23. М.: Наука, 1982.
URL: http://ilib.mccme.ru/djvu/bib-kvant/teorver.htm
[KR] Курант Р., Роббинс Г. Что такое математика? М.: МЦНМО,
2001.
URL: http://ilib.mccme.ru/pdf/kurant.htm .
[L] Lovász L. Combinatorial Problems and Exercises. North-Holland,
Amsterdam, 1979.
[M] J. Matoušek. Thirty-three Miniatures: Mathematical and
Algorithmic Applications of Linear Algebra, Amer. Math.
Soc., 2010
[Ma] А. Матушкин, Непустота пересечения цепочки множеств,
Мат. Просвещение, представлено к публикации.
[MS] Медников Л. Э., Шаповалов А.В. Турнир городов: мир математики в задачах. МЦНМО, 2012.
[NPP] Ф. Нилов, A. Полянский, Н. Полянский. Инциденции точек
и прямых,
URL: http://www.turgor.ru/lktg/2014/7/index.htm
[O] Задачник по ОКТЧ./ Глибичук А.А. [и др.] Готовится к печати.
[P] Прасолов В. В. Элементы комбинаторной и дифференциальной
топологии. М.: МЦНМО, 2004.
URL: http://www.mccme.ru/prasolov
[PS05] В.В. Прасолов и М.Б. Скопенков. Рамсеевская теория зацеплений // Мат. Просвещение. 2005. 9. С. 108-115.
[R1] Райгородский А.М. Линейно-алгебраический метод в комбинаторике. М.: МЦНМО, 2015.
236
Элементы дискретной математики в задачах
[R2] Райгородский А.М. Проблема Борсука. М.: МЦНМО, 2014.
[R3] Райгородский А.М. Вероятность и алгебра в комбинаторике.
М.: МЦНМО, 2010.
[R4] Райгородский А.М. Модели случайных графов и их применения. М.: МЦНМО
URL:
http://ium.mccme.ru/postscript/s12/gasnikovraigorodskii.pdf
[R5] Райгородский А.М., Комбинаторика и теория вероятностей.
М.: Изд-во МФТИ, 2012.
[R6] Райгородский А.М., Системы общих представителей в комбинаторике и их приложения в геометрии. М.: МЦНМО, 2013.
URL: http://www.mccme.ru/free-books/dubna/raigor-2.pdf
[S] Д. Судзуки, Основы дзэн-буддизма. Наука дзэн — ум дзэн. Киев: Преса Украiни. 1992.
[S1] Skopenkov A. On the Kuratowski graph planarity criterion.
URL: http://arxiv.org/abs/0802.3820 , v3.
Русскоязычная версия: Скопенков А. Вокруг критерия Куратовского планарности графов, Мат. Просвещение, 9 (2005),
116–128 и 10 (2006), 276–277.
URL: http://www.mccme.ru/free-books/matprosa.html
[S2] Skopenkov A. A two-page disproof of the Borsuk partition
conjecture.
URL: http://arxiv.org/abs/0712.4009 , v2.
Русскоязычная версия: Скопенков А. Короткое опровержение
гипотезы Борсука. Мат. Просвещение, 17 (2013), с. 88–92.
URL: http://www.mccme.ru/free-books/matpros/matprosi.html
[S3] Скопенков А. Алгебраическая топология с геометрической
точки зрения. М.: МЦНМО,в печати.
URL:
http://arxiv.org/abs/0808.1395,
http://www.mccme.ru/circles/oim/obstruct.pdf
ЛИТЕРАТУРА
237
[S4] Скопенков А. Олимпиады и математика. Мат. Просвещение, 10
(2006), с. 57–63.
URL: http://www.mccme.ru/free-books/matprosb.html
[S5] Скопенков А. Простое доказательство теоремы РуффиниАбеля, Мат. Просвещение, 15 (2011), с. 113–126.
URL: http://arxiv.org/abs/1102.2100
[S6] Скопенков А. Объемлемая однородность, МЦНМО, Москва,
2012.
URL: http://arxiv.org/abs/1003.5278
[Sk] A. Skopenkov. Realizability of hypergraphs and Ramsey link
theory, http://arxiv.org/abs/1402.0658
[Sk15] A. Skopenkov. A short elementary proof of the Ruffini-Abel
Theorem, http://arxiv.org/abs/1508.03317
[S7] Скопенков А. Алгоритмы распознавания реализуемости гиперграфов.
URL: http://www.mccme.ru/circles/oim/algor.pdf
[So] Соловьева Ф. И. Введение в теорию кодирования. Новосибирск,
2006.
URL: http://tc.nsu.ru/uploads/codingtheory.pdf
[Ve] Веснин А.Ю. Гамильтоновы графы и остовные подграфы: задачи для исследования, материалы Московской математической конференции школьников.
URL: http://www.mccme.ru/mmks/mar08/vesnin3.pdf
[Vi] Виноградов И.М. Основы теории чисел. М.; Ижевск: НИЦ «Регулярная и хаотическая динамика», 2003.
[VS] Волков М., Силкин Н. Кого послать на Марс? // Квант (1988)
N8, с. 51–57.
URL: http://kvant.mccme.ru/1988/08/kogo_poslat_na_mars.htm
[Z] Математика в задачах. Сборник материалов московских выездных математических школ. Под редакцией А. Заславского
238
Элементы дискретной математики в задачах
и др. М.: МЦНМО, 2009.
URL: http://www.mccme.ru/free-books/olymp/matprob.pdf
[1] https://www.dpmms.cam.ac.uk/~dc340/EGT3.pdf
[2] http://www.cs.rit.edu/~spr/ElJC/ejcram14.pdf
[3] http://www.math.upenn.edu/~wilf/website/Three\
%20problems.pdf
[4] http://www.unn.ru/math/no/5/_nom5_001_ilyin.pdf
[5] http://www.sosmath.com/calculus/sequence/stirling/
stirling.html
[6] http://www.spbstu.ru/publications/m_v/n_002/Polischook/
Stirling.pdf
[7] http://arxiv.org/pdf/1109.2546.pdf
[8] http://dainiak.blogspot.ru
Download